ML15175A443

From kanterella
Revision as of 02:18, 9 July 2018 by StriderTol (talk | contribs) (Created page by program invented by StriderTol)
Jump to navigation Jump to search
Hatch Initial Exam 2015-301 Draft RO Written Exam
ML15175A443
Person / Time
Site: Hatch  Southern Nuclear icon.png
Issue date: 06/22/2015
From:
NRC/RGN-II
To:
Southern Nuclear Operating Co
References
50-321/15-301, 50-366/15-301 50-321/15-301, 50-366/15-301
Download: ML15175A443 (247)


Text

Name: ________________________________ ILT-09 RO NRC EXAM Form: 0 Version: 0 1.201002K3.03 001/05401RMCS/001.010.A.04/NEW/SYS-B/BOTH/201002K3.03/2/2/H/3/ARB/ELJUnit 2 is performing a shutdown with reactor power at 13% RTP. At 13:00, a FAILURE of the MASTER Timer occurs in the Reactor Manual Control System(RMCS).oThe MASTER Timer will NOT energizeAt 13:30, a RWM "Insert" ROD BLOCK exists on the currently selected control rod. oRMCS / RWM ROD BLOCK OR SYSTEM TROUBLE, (603-239) is ILLUMINATEDAt 13:05, if the "Rod Movement Control" switch is placed to the "IN" position, the selected control rod __________ INSERT. At 13:35, if the "Emergency In/Notch Override" switch is placed to the "Emergency In"position, RMCS __________ INSERT the selected control rod. will; will will; will NOT will NOT; will will NOT; will NOT A.B.C.D.Description:RMCS provides the electrical components and logic circuitry required to monitor andmanipulate, in precise fashion, the control rods used in the Reactor Core. RMCS imposes theoperating constraints (rod blocks) and permissives related to control rod movement that areappropriate to equipment conditions and the flux density at circumscribed regions about eachcontrol rod in the fuel matrix. A rod is selected for movement by depressing a pushbutton for the desired rod on the reactorcontrol bench board. The arrangement of control rod selection pushbuttons and circuitry permits the selection of only one control rod at a time for movement. The rod selection circuitry isarranged so that a rod selection is sustained until either another rod is selected or automaticaction occurs which reverts the selection circuitry to a no-rod-selected condition. Once a rod has been selected, the direction for travel is determined by the Rod Movement Control Switch(RMCS). A rod insert signal is generated by placing the RMCS to the ROD IN position. Turning the RMCS to ROD IN and releasing (spring return to OFF) initiates the Master Timer ifWednesday, January 21, 2015 2:26:47 PM 1

ILT-09 RO NRC EXAMthere are no rod inhibit signals from the following:

a. RWM Insert blockb.Control Rod Withdraw signal presentThe Master Timer energizes the appropriate buses in the required sequence for a rod insertion. Initiating movement of the selected rod prevents the selection of any other rod until the insertcycle of the selected rod is completed. The sequence involves energizing the Insert Bus (valves121 & 123) for a preset amount of time. The Insert Bus is then automatically deenergized byenergizing the Settle Bus (valve 120). The Settle Bus remains energized for a period of time toallow the control rod to settle into the next notch position. The Rod Movement Control Switch spring returns to the "OFF" position. It indicates notch inand notch out cycles. If held in "NOTCH OUT" the rod will complete one notch out cycle andstop. If held in "ROD IN" position the rod will continuously insert until the switch is released oruntil a RWM rod block is initiated, or control rod is fully inserted. The "EMERGENCY IN" position bypasses all the interlocks to insert the rod except the RWMInsert blocks. It directly energizes the directional control valves by bypassing the timer. Itallows continuous rod motion inward with no settle function; water is forced past the seals in theCRD while settling into a notch. With the Master Timer failure, RMCS will still prevent control rod movement if a RWM insertblock occurs. K/A JUSTIFICATION:The second part of this question satisfies the K/A statement by asking the applicant to determineif RMCS will still process a control rod block using the EMERGENCY IN switch if a failure ofRMCS occurs, which in this case is the Master Timer failing. The "A" distractor is plausible if the applicant thinks about the operation of the "Rod Out/NotchOverride" switch and remembers the "Emergency In"switch bypasses the Master Timer and directly energizes the Insert bus allowing control rod insertion. The second part is plausible ifthe applicant remembers that during the performance of 31EO-EOP-103 that ALL inward rodmotion is not prohibited. This is allowed since the RWM is bypassed.The "B" distractor is plausible if the applicant thinks about the operation of the "Rod Out/NotchOverride" switch and remembers the "Emergency In"switch bypasses the Master Timer and directly energizes the Insert bus allowing control rod insertion. The second part is plausiblesince it is correct. The "C" distractor is plausible since the first part is correct. The second part is plausible if theapplicant remembers that during the performance of 31EO-EOP-103 that ALL inward rodmotion is not prohibited. This is allowed since the RWM is bypassed.A. Incorrect - See description above. B. Incorrect - See description above. C. Incorrect - See description above. Wednesday, January 21, 2015 2:26:47 PM 2

ILT-09 RO NRC EXAMD. Correct - See description above.

References:

NONEK/A:201002 Reactor Manual Control SystemK3. Knowledge of the effect that a loss or malfunction of the REACTOR MANUALCONTROL SYSTEM will have on following: (CFR: 41.7 / 45.4)K3.03 Ability to process rod block signals . . . . . . . . . . . . . . . 2.9 3.0LESSON PLAN/OBJECTIVE:C11-RMCS-LP-05401, Reactor Manual Control System (RMCS), Ver. 5.0, EO 001.010.A.04 &EO 001.026.A.01 References used to develop this question:34GO-OPS-065-0, Control Rod Movement, Ver. 12.4 Wednesday, January 21, 2015 2:26:47 PM 3

ILT-09 RO NRC EXAM 2.201003A1.02 001/00101C11/001.005.A.10/MOD/SYS-B/BOTH/201003A1.02/2/2/H/2/JSC/ELJUnit 2 is operating at 25% RTP while performing a Reactor Startup. The next reactivitymanipulation is to withdraw control rods in the selected rod group from position 12 to 20.The OATC observes the following indications: Based on the above conditions, If a Control Rod is moved, the Control Rod will travel at __________ speed.Placing the control switch for 2C11-F003, Drive Press Cntl Valve, to OPEN for one (1) second will cause Drive Water dp, 2C11-R602, indication to __________ .faster than NORMAL;lowerfaster than NORMAL;riseNORMAL;lowerNORMAL;rise A.B.C.D.Description:Normal Drive water differential pressure is maintained 220-280 psid IAW 34SO-C11-005-2 inorder to maintain rod speeds constant. Drive water dp is based on the differential between CRD pressure and RPV pressure. Drive water is used to insert or withdraw control rods. Drive wateris applied to the CRDM piston area therefore the higher the differential pressure the faster theWednesday, January 21, 2015 2:26:47 PM 4

ILT-09 RO NRC EXAMcontrol rod speeds will be.

Drive water dp is controlled by positioning 2C11-F003 fromH11-P603.

To increase dp, the operator has to take 2C11-F003 control switch to the close position.

To lower dp, the operator has to take 2C11-F003 control switch to the open position. IAW 334AB-C11-003-2, Caution 1 states:OPERATIONS MANAGEMENT APPROVAL IS REQUIRED PRIOR TO INCREASINGDRIVE WATER PRESSURE ABOVE 350 PSID FOR CONTROL RODS WHICH ARE NOTFULLY INSERTED.K/A JUSTIFICATION:This question satisfies the K/A statement by requiring the applicant to predict how the CRDdrive water DP affects rod speeds.The "B" distractor is plausible since the first part is correct. The second part is plausible if theapplicant thinks differential pressure is maintained for drive flow and is measured downstream of 2C11-F003. In this case, opening 2C11-F003 will direct more flow/pressure causing Drivewater dp to rise. The "C" distractor is plausible if the applicant thinks that normal drive water dp ranges up to 350psid since this differential pressure is used in 34AB-C11-003-2, Inability to Move a Control Rod,for freeing a control rod. During the performance of this abonormal, Operations Management permission is required increase differntial pressure above 350 psid. The normal range for drivewater d/p is 220-280 psid. The second part is plausible since it is correct.The "D" distractor is plausibleif the applicant thinks that normal drive water dp ranges up to 350psid since this differential pressure is used in 34AB-C11-003-2, Inability to Move a Control Rod,for freeing a control rod. During the performance of this abonormal, Operations Management permission is required increase differntial pressure above 350 psid. The normal range for drivewater d/p is 220-280 psid. The second part is plausible if the applicant thinks differentialpressure is maintained for drive flow and is measured downstream of 2C11-F003. In this case, opening 2C11-F003 will direct more flow/pressure causing Drive water dp to rise. A. Correct - See description above. B. Incorrect - See description above. C. Incorrect - See description above. D. Incorrect - See description above.

References:

NONEK/A:Wednesday, January 21, 2015 2:26:47 PM 5

ILT-09 RO NRC EXAM201003 Control Rod and Drive MechanismA1. Ability to predict and/or monitor changes in parameters associated with operating theCONTROL ROD AND DRIVE MECHANISM controls including: (CFR: 41.5 / 45.5)A1.02 CRD drive pressure . . . . . . . . . . . . . . . . . . 2.8 2.8LESSON PLAN/OBJECTIVE:C11-CRD-LP-00101, Control Rod Drive System, Ver 8.0, EO 001.005.A.10References used to develop this question:34SO-C11-005-2, Control Rod Drive Hydraulic System, Ver 32.034AB-C11-003-2, Inability to Move a Control Rod, Ver 10.1Modified from HLT Database Q#201001A1.03-001 Original QuestionThe Unit 2 CRD system is being started per 34SO-C11-005-2, "Control Rod Drive HydraulicSystem", with the following plant conditions: o"2A" CRD pump........................................Runningo2C11-F002A, FCV.....................................AUTOoCRD system flow.......................................50 gpm o2C11-F003, Drive Press Cntl Valve..........Full OPENWhich ONE of the choices below completes the following statement? As the operator throttles 2C11-F003 in the CLOSED direction, the Drive Water dP will__________; and, 2C11-F002A, FCV, will throttle __________. A. decrease; open B. decrease; closed C. increase; open D. increase; closed Wednesday, January 21, 2015 2:26:47 PM 6

ILT-09 RO NRC EXAM 3.201006A1.03 001/05403RWM/H-OP-90000.003/NEW/SYS-I/BOTH/201006A1.03/2/2/F/2/ARB/ELJUnit 2 is starting up with the following conditions:oReactor power is 3.5 % RTPoControl Rod density equals 60%oThe currently "Latched" rod step (step 12) has an Insert limit of 24 and aWithdraw limit of 48oAll Control rods in the currently "Latched" step are at position 48 oStep 13 Control rods are at position 00The RWM "Display" will FIRST indicate that Step 13 is "Latched" when the __________ . After RWM is "Latched" to Step 13, ALL of the Control rods "Latched" in this step will beindicated ____________ . first Control rod in step 13 is selected; on the "List Rods" display of RWM first Control rod in step 13 is selected; by backlighting on the Control Rod Select Matrix last Control rod in step 12 reached position 48; on the "List Rods" display of RWM last Control rod in step 12 reached position 48; by backlighting on the Control Rod Select Matrix A.B.C.D.Description:Rod Selection Matrix (RSM),The RSM contains 137 pushbuttons, corresponding to the 137 control rods, used by the operatorto select a control rod to be moved. Each pushbutton contains backlights which will illuminatebrightly if that pushbutton is selected by the operator and pushed. If greater than 50% roddensity, it will dimly illuminate if that pushbutton is part of the selected rod group, or not at all ifthere is a rod select inhibit associated with that control rod. If less than 50% rod density, once apushbutton of a control rod group is selected and pushed, that particular pushbutton willbacklight brightly and the rest of the pushbuttons belonging to the group will not backlight untilthe selected rod in the group is moved one notch away from the other rods. This will cause the rest of the group to dimly backlight. When all rods in the group are at the same position, thedimly lit pushbuttons will extinguish. In either condition, since only one rod may be selected ata time, only one pushbutton will be backlighted brightly in a group.RWM evaluates all control rod positions to determine the current rod step, per the prescribed rodsequence, ten times a second. This evaluation is used to determine the "Latched" Step. The Latched Step is the group of rods that contain the rod that is currently being or has just beenWednesday, January 21, 2015 2:26:47 PM 7

ILT-09 RO NRC EXAMmoved per the prescribed sequence.

When all the rods in a currently latched step are at theirwithdraw (insert if shutting down) limit, the RWM will latch into the next higher (lower ifshutting down) step when the first rod in the next step is selected. An example would be:

Rods are being withdrawn in Step 6.

When all the rods in step 6 are at their withdraw limit,RWM will still show Step 6 as the Latched Step (this is the step number on the RWM display).

As soon as a Step 7 rod is selected, the Step number on the display will change to 7, and theLatched Step will be 7.

This is true unless the rods in the higher step are the same rods in thecurrent step, i.e. the rods in Step 4 that are moved from position 04 to 08 are the same rods as inStep 3 that were moved from 00 to 04 (BPWS).

If this is the case, the Step Number display willnot change until the first rod in the higher step is moved, i.e.

the first rod in Step 4 from theprevious example is withdrawn past position 04.

"ALL RODS IN" is determined by all rods being at or inserted past position "00". If all rods arefully inserted then ALL RODS IN: YES is displayed. If all rods are not full in then ALL RODSIN: NO and RODS NOT FULL-IN: # (number of rods not full-in) will be displayed. A LISTRODS softkey will then be available to determine which rods are not full-in and their position. If more than 16 rods are not full-in, then the softkeys NEXT PAGE and PREVIOUS PAGE will be available to view the remainder of the rods not full-in. These displays will not updateautomatically as the rod's current position changes. K/A JUSTIFICATION:This question satisfies the K/A statement by requiring the applicant to use RWM and specificplant conditions to determine the "Latched" Group.The "A" distractor is plausible since the first part is correct. The second part is plausible if theapplicant remembers that the "List Rods" function will indicate all of the control rods that are notfull-in and does not recognize Step 13 Control rods are at position 00. Since Step 13 Controlrods are full-in, they will not be displayed on the "List Rods" screen. The "C" distractor is plausible if the applicant remembers that a withdraw block will occur whenposition 48 is reached and thinks since all the rods in the current group are now at their withdraw position that RWM will then transition to the next step of control rods to be withdrawn. Thesecond part is plausible if the applicant remembers that the "List Rods" function will indicate allof the control rods that are not full-in and does not recognize Step 13 Control rods are at position 00.Since Step 13 Control rods are full-in, they will not be displayed on the "List Rods" screen.The "D" distractor is plausible if the applicant remembers that a withdraw block will occur whenposition 48 is reached and thinks since all the rods in the current group are now at their withdrawposition that RWM will then transition to the next step of control rods to be withdrawn. Thesecond part is plausible since it is correct. A. Incorrect - See description above. B. Correct - See description above. C. Incorrect - See description above. D. Incorrect - See description above. Wednesday, January 21, 2015 2:26:47 PM 8

ILT-09 RO NRC EXAM

References:

NONEK/A:201006 Rod Worth Minimizer System (RWM) (Plant Specific)A1. Ability to predict and/or monitor changes in parameters associated with operating theROD WORTH MINIMIZER SYSTEM (RWM) (PLANT SPECIFIC) controls including:

(CFR: 41.5 / 45.5)A1.03 Latched group indication: P-Spec (Not-BWR6) . . . . . . . . . . . . . . . . . . . . 2.9 3.0LESSON PLAN/OBJECTIVE:C11-RWM-LP-05403, Rod Worth Minimizer, Ver. 6.2, H-OP-90000.003References used to develop this question:34GO-OPS-065-0, Control Rod Movement, Ver. 12.4 Wednesday, January 21, 2015 2:26:47 PM 9

ILT-09 RO NRC EXAM 4.203000A2.14 001/00701E11/006.005.A.01/MOD/SYS-I/BOTH/203000A2.14/2/1/H/2/ARB/ELJUnit 2 is operating at 25% RTP with the following alarm status:RHR RELAY LOGIC BPOWER FAILUREIlluminatedExtinguishedRHR RELAY LOGIC APOWER FAILURESubsequently, an event occurs resulting in the following conditions:oDrywell pressure to stabilize at 4.5 psigoRPV pressure decreases to 160 psigWith the current status of the above alarms and plant conditions, RHR pumps 2B & 2D __________ . IAW 34AR-601-207-2, RHR Relay Logic B Power Failure, the NPO will confirm Circuitbreaker #4 on __________ , is in the CLOSED position. must be manually started; 2R25-S002, 125 VDC Distribution Cabinet 2B must be manually started; 2R25-S006, 125 VDC Distribution Cabinet 2F will automatically start; 2R25-S002, 125 VDC Distribution Cabinet 2B will automatically start; 2R25-S006, 125 VDC Distribution Cabinet 2F A.B.C.D.Description:Control power for RHR pumps "A" & "D" are in Electrical Division 1 (2R25-S004) with controlpower for RHR pumps "B" & "C" in Electrical Division 2 (2R25-S006) . If either Division 1 or 2 Control power is lost then the respective two (2) RHR pumps will not have control power foroperation of the breaker closure. Initiation Logic "A" is operated from 2R25-S001, 125 VDC Bus "A" and Initiation Logic "B" isoperated from 2R25-S002, 125 VDC Bus "B". If 2R25-S001 or 2R25-S002 fail prior to a LOCAsignal being received, then either Div I or Div II logic, will de-energize. If a LOCA signal isthen received, the division with power still available will start ALL RHR pumps in both Div IWednesday, January 21, 2015 2:26:47 PM 1 0 ILT-09 RO NRC EXAMThe RHR pumps will initiate from either divisions logic on: Low Reactor Water Level -101 inches High Drywell Pressure 1.85 psig IAW 34SO-E11-010-2, RHR System, contains the following steps explaining logic power to theRHR pumps:

2.2 Restarting an RHR pump following abnormal conditions affected by anti-pump logic andDCR 98-011 (4160V Breaker Operability Indication):2.2.1 An auto initiation signal is present and then the pump is manually TRIPPED: oThe operator is alerted by illumination of the white start disagreement light above thesecured pump control switch. This white light remains sealed in illuminated until:- The initiation signal clears and is reset.OR- Power is removed from the applicable 4160V bus.OR- DC logic power is lost.o"The operator can manually restart the pump using the control switch (as long asDC logic power OR 4160V bus power has not been lost).o"The pump will auto start if the initiation signal clears, is reset, and a subsequentinitiation signal occurs.o"The pump will auto start if a loss of relay logic power occurs for the divisionassociated with the secured pump (Division I for A and D pumps, Division II for Band C pumps) AND an initiation signal is still present from the opposite divisionrelay logic.o"Loss of the opposite division relay logic power does not affect the ability tostart/stop the pump with the control switch.2.2.2 A loss of associated relay logic power exists AND an auto initiation signal is present,and then the pump is manually TRIPPED: The breaker will TRIP using the control switch.The white start disagreement light above the control switch will NOT illuminate due toloss of relay logic power.The green breaker open/pump stopped indicating light will NOT illuminate due to thebreaker anti-pump feature being activated because the breaker close signal is stillpresent from the other division relay logic.To restart the pump, the pump restart pushbutton must be depressed AND held until thegreen breaker open/pump stopped light illuminates. This removes the breaker closesignal from the opposite division and allows the breaker to recharge. When the greenlight is illuminated, the push button can be released and the pump will restart from theopposite division start signal.Wednesday, January 21, 2015 2:26:47 PM 1 1 ILT-09 RO NRC EXAM2.2.3 The pump has been manually TRIPPED and then a loss of associated relay logic power occurs: The green breaker open/pump stopped light is illuminated.The pump can be started with the control switch.2.2.4 The pump has been manually TRIPPED, and a loss of associated relay logic power hasoccurred, and then an initiation signal is received. The pump will start upon receipt of an automatic initiation signal.K/A JUSTIFICATION:This question satisfies the K/A statement by requiring the applicant to predict the impact onRHR pumps (manual start versus auto start) with the failure of Division B initiation logic andthen use 34AR-601-207-2 to determine which cabinet power supply must be investigated for theassociated power failure. The "A" distractor is plausible if the applicant thinks about the control power logic instead of theinitiation logic and since Div II initiation logic is lost, thinks RHR 2B & 2D must be manuallystarted. This would also be true if an initiation signal is present, the initiation logic is then lostand then the RHR pump is secured, resulting in the RHR pump not auto starting and must be manually started. The second part is plausible since it is correct. The "B" distractor is plausible if the applicant thinks about the control power logic instead ofthe initiation logic and since Div II initiation logic is lost, thinks RHR 2B & 2D must bemanually started. This would also be true if an initiation signal is present, the initiation logic isthen lost and then the RHR pump is secured, resulting in the RHR pump not auto starting and must be manually started. The second part is plausible since this power supply provides controlpower for RHR 2C, PSW 2D, and CRD 2B, and the applicant can think this is the initiation logicpower supply. The "D" distractor is plausible since the first part is correct. The second part is plausible sincethis power supply provides control power for RHR 2C, PSW 2D, and CRD 2B, and the applicant can think this is the initiation logic power supply. A. Incorrect - See description above. B. Incorrect - See description above. C. Correct - See description above. D. Incorrect - See description above.

References:

NONEWednesday, January 21, 2015 2:26:47 PM 1 2 ILT-09 RO NRC EXAMK/A:203000 RHR/LPCI: Injection Mode (Plant Specific)A2. Ability to (a) predict the impacts of the following on the RHR/LPCI: INJECTIONMODE (PLANT SPECIFIC) ; and (b) based on those predictions, use procedures to correct, control, or mitigate the consequences of those abnormal conditions or operations: (CFR: 41.5 / 45.6)A2.14 Initiating logic failure . . . . . . . . . . . . . . . . . . . . . . . . . . . 3.8 3.9*REPLACED THE BELOW K/A AFTER PHONE CONVERSATION WITH CHIEFEXAMINER PHIL CAPEHART ON 3/27/2014.A2. Ability to (a) predict the impacts of the following on the RHR/LPCI: INJECTIONMODE (PLANT SPECIFIC) ; and (b) based on those predictions, use procedures tocorrect, control, or mitigate the consequences of those abnormal conditions or operations:

(CFR: 41.5 / 45.6)A2.15 Loop selection logic failure: Plant-Specific . . . . . . . . . . . . . 4.2* 4.2*LESSON PLAN/OBJECTIVE:E11-RHR-LP-00701, Residual Heat Removal System, Ver. 7.0, EO 006.005.A.01 & EO 006.007.A.02 References used to develop this question:34AB-R22-001-2, Loss Of DC Buses, Ver. 4.3 34AR-601-207-2, RHR Relay Logic B Power Failure, Ver. 2.2 34SO-E11-010-2, RHR System, Ver. 40.4 Modified from HLT Database Q#209001K3.03-002 Original QuestionUnit 2 was operating at 25% RTP when an event occurs causing Drywell pressure to stabilize at4.5 psig. An NPO reports the status of the following alarms: Wednesday, January 21, 2015 2:26:47 PM 1 3 ILT-09 RO NRC EXAMIlluminatedExtinguishedWith the current status of the above alarms, __________ will be operating and The 2C EDG frequency indication will be __________ . A. ONLY one (1) Core Spray pump; approximately 60 Hz B. ONLY one (1) Core Spray pump; greater than 65 Hz C. BOTH Core Spray pumps; approximately 60 Hz D. BOTH Core Spray pumps; greater than 65 Hz Wednesday, January 21, 2015 2:26:47 PM14 ILT-09 RO NRC EXAM 5.205000K3.02 001/03401E11/034.002.A.10/MOD/SYS-B/BOTH/205000K3.02/2/1/F/2/ARB/ELJUnit 1 is in Mode 4 with RHR Loop B Shutdown Cooling in service. The following conditions exist:oRecirculation Pumps .................. SecuredoRHR B Pump flow .................... 7700 gpmoAll other RHR Pumps ............... StandbySubsequently, a tube rupture occurs in RHR Heat Exchanger 1B. With the above conditions and NO operator actions,RWL will start __________ . IAW 34SO-E11-010-1, RHR System, the MINIMUM listed RHR to RHRSW differentialpressure allowed is __________ . going up;21 psid going up;31 psid lowering; 21 psid lowering; 31 psid A.B.C.D.Description:Each heat exchanger is a vertical, U tube, single pass heat exchanger. There are two heatexchangers, one per RHR loop. The heat exchangers capacity is based on the most severe modeof operation, which is the SDC mode. It is designed to maintain reactor temperature less than orequal to 125°F, 20 hrs after all rods are fully inserted. In order to prevent leakage from the RHR system into the RHRSW system, tube side pressure in the heat exchanger is maintained higher than shell side pressure. The differential pressure ismaintained by controlling the position of 1E11-F068A/B. If RHR (shell side) pressure was greater than RHRSW (inside tubes) pressure and a tube leakexisted, a possible radioactive release to the environment could occur (RHRSW discharge flowis directed to the flume). 34SO-E11-010 directs placing the INTERLOCK OVERRIDE keylock switch for 1E11-F068A/B to the override position prior to starting the RHRSW pump in its respective loop. This actionoverrides the 30 psi interlock on valve F068A/B, thus allowing opening the F068A/B valveprior to starting a RHRSW pump. After the pump is started, immediately return the keylock switch to its normal position. With the keylock switch in normal, 1E11-F068A/B will isolateshould the running RHRSW pump trip. When placing the RHRSW system in operation, the tubeWednesday, January 21, 2015 2:26:47 PM 1 5 ILT-09 RO NRC EXAMside heat exchanger outlet valve (F068A/B) is used to manually control RHRSW pressure higherthan RHR pressure by at least 20 psig. A RHRSW (tube) side leak will cause RWL to go up since RHR is taking a suction off the RPVand discharging back to the RPV but now with RHRSW leaking into the shell side. If RHR wasinside the tube, the leak would be flowing from RHR to the shell side causing RWL to startgoing down. K/A JUSTIFICATION:This question satisfies the K/A statement by asking the applicant the effect that a malfunction ofSDC (tube leak) will have on RWL. In this question, the effect on RWL is that with the SDCtube leak, RWL will start going up from its current value. If RHR was inside the tube, the leakwould be flowing from RHR to the shell side causing RWL to start going down. The "B" distractor is plausible since the first part is correct. The second part is plausible if theapplicant thinks about the 30 psig limit vice the 20 psid limit. Would also be correct if askingabout the 30 psig limit for F068 closure. The "C" distractor is plausible if the applicant thinks the high pressure side of the heat exchangeris on the RHR side therefore RWL would lower due to leaking to the RHRSW side. The secondpart is plausible since it is correct. The "D" distractor is plausible if the applicant thinks the high pressure side of the heatexchanger is on the RHR side therefore RWL would lower due to leaking to the RHRSW side.

The second part is plausible if the applicant thinks about the 30 psig limit vice the 20 psid limit.Would also be correct if asking about the 30 psig limit for F068 closure. A. Correct - See description above. B. Incorrect - See description above. C. Incorrect - See description above. D. Incorrect - See description above. Wednesday, January 21, 2015 2:26:47 PM 1 6 ILT-09 RO NRC EXAM

References:

NONEK/A:205000 Shutdown Cooling System (RHR Shutdown Cooling Mode)K3. Knowledge of the effect that a loss or malfunction of the SHUTDOWN COOLINGSYSTEM (RHR SHUTDOWN COOLING MODE) will have on following:

(CFR: 41.7 / 45.4)K3.02 Reactor water level: Plant-Specific . . . . . . . . . . . 3.2 3.3LESSON PLAN/OBJECTIVE:E11-RHRSW-LP-03401, Residual Heat Removal Service Water System (RHRSW), Ver. 5.0 ,EO 034.002.A.10 References used to develop this question:34SO-E11-010-1, "Residual Heat Removal System", Ver. 44.1Modified from HLT Database Q#LT-034002-017 Original QuestionA MINIMUM RHR to RHRSW differential pressure of __________ is procedurally requiredto be maintained to ensure that any RHR Heat Exchanger leaks result in the leaking watergoing into the __________ system. A. 8 psid; RHR B. 8 psid; RHRSW C. 20 psid; RHR D. 20 psid; RHRSW Wednesday, January 21, 2015 2:26:47 PM 1 7 ILT-09 RO NRC EXAM 6.206000A1.04 001/00501E41/005.002.A.03/NEW/SYS-B/BOTH/206000A1.04/2/1/F/2/JSC/ELJUnit 2 was operating at 100% RTP when a spurious scram occurred. RPV pressure control has been transferred to the HPCI System due to the MSIVs beingmanually closed. IAW 34SO-E41-001-2, HPCI System, HPCI is operating in the Pressure Control Mode at thefollowing times with the associated flows and 2E41-F008, Test To CST Valve, positions: Time HPCI flow 2E41-F00810:00 Rated gpm 75% open 10:20 Rated gpm 25% open Based on the above conditions;The HIGHEST rate of Torus water level increase will be occurring at __________ . If Torus water level reaches 151 inches, HPCI will be operating __________ . 10:00; on MINIMUM flow 10:00; at RATED flow 10:20; on MINIMUM flow 10:20; at RATED flow A.B.C.D.Description:With HPCI in the pressure control mode, the amount of exhaust to the suppression pool dependson how much work the HPCI turbine is performing. Since the flow is set at rated, the amount ofwork is inversely proportional to the F008 position. If F008 is throttled in close direction, theturbine will have to work harder to maintain rated flow since the flow path is being restricted.The turbine will have to increase speed to maintain flow therefore more steam is being exhausted to the suppression pool. The more steam that is exhausted means the level will rise at a fasterrate. HPCI Suppression Pool Suction Isolation Valves F041 and F042, isolate the SuppressionPool from the HPCI system. They also provide an automatic transfer of the HPCI pump suction source from the CST to the Suppression Pool. The automatic suction swap will occur when theCST lowers to < 34 inches or the Suppression pool increases to 152 inches.When either of the above conditions are met, the Suppression Pool Valves F041 and F042 will receive an open signal. When BOTH of these valves are full open, the CST Suction Valve F004will receive a close signal. This ensures a suction path is maintained for the HPCI pump andprevents draining the CST to the Suppression Pool. 2E41-F008 and F011, Test to CST valves, will receive a close signal when either HPCI Suppression Pool Suction Isolation Valves F041and F042 are fully open.Wednesday, January 21, 2015 2:26:47 PM 1 8 ILT-09 RO NRC EXAMK/A JUSTIFICATION:This question satisfies the K/A statement by requiring the applicant to predict/monitor changesin the suppression pool level based upon the increase in HPCI exhaust.The "A" distractor is plausible if the applicant thinks about the operation of 2E41-F008. Theopening of a valve in a system would usually mean more flow (work) however the Turbine is in automatic control. If HPCI was in manual mode with the F008 opened more (75% vice 25%), theturbine would be performing more work thus level would be increasing faster. The second part isplausible if the applicant thinks about the RCIC system setpt (150.5 inches) instead of the HPCI system setpt (152 inches). If the applicant thinks about the RCIC setpt, the HPCI system wouldshift suction sources and the test valves (F008/F011) would close causing HPCI to be onminimum flow.The "B" distractor is plausible if the applicant thinks about the operation of 2E41-F008. Theopening of a valve in a system would usually mean more flow (work) however the Turbine is in automatic control. If HPCI was in manual mode with the F008 opened more (75% vice 25%), theturbine would be performing more work thus level would be increasing faster. The second part isplausible since it is correct. The "C" distractor is plausible since the first part is correct.The second part is plausible if theapplicant thinks about the RCIC system setpt (150.5 inches) instead of the HPCI system setpt (152 inches). If the applicant thinks about the RCIC setpt, the HPCI system would shift suctionsources and the test valves (F008/F011) would close causing HPCI to be on minimum flow.A. Incorrect - See description above. B. Incorrect - See description above. C. Incorrect - See description above. D. Correct - See description above. Wednesday, January 21, 2015 2:26:47 PM 1 9 ILT-09 RO NRC EXAM

References:

NONEK/A:206000 High Pressure Coolant Injection System A1. Ability to predict and/or monitor changes in parameters associated with operating theHIGH PRESSURE COOLANT INJECTION SYSTEM controls including:

(CFR: 41.5 / 45.5)A1.04 Suppression pool level: BWR-2,3,4 . . . . . . . . . . . 3.7 3.8LESSON PLAN/OBJECTIVE:E41-HPCI-LP-00501, High Pressure Coolant Injection (HPCI), Ver 6.0, EO 005.002.A.03References used to develop this question:34SO-E41-001-2, High Pressure Coolant Injection (HPCI) System, Ver 28.3 Wednesday, January 21, 2015 2:26:47 PM 2 0 ILT-09 RO NRC EXAM 7.206000A4.03 001/00501E41/005.003.A.04/MOD/SYS-B/BOTH/206000A4.03/2/1/F/3/ARB/ELJThe Unit 2 HPCI system was manually placed in service following a Feedwater transient. The following conditions exist: oRWL ............................ 38 inches (lowest level reached -15 inches)oRPV Pressure ............... 920 psigoDrywell pressure ......... 0.5 psig (highest pressure reached 0.6 psig)oHPCI Bearing temperatures are steadily increasingThe SRO directs the NPO to monitor HPCI Bearing temperatures. With the above conditions and IAW 34SO-E41-001-2, High Pressure Coolant Injection (HPCI)System, When HPCI was started, 2E41-F059, Lube Oil Clg Wtr valve, __________ opened.The NPO will monitor HPCI Bearing temperatures on Panel, __________ . was manually;2H11-P700 was manually; 2H11-P614 automatically; 2H11-P700 automatically; 2H11-P614 A.B.C.D.Description:HPCI Bearing temperatures are monitored on recorder 2E41-R605 on panel 2H11-P614 in theMain Control Room. 2H11-P700 panel, also located in the Main Control Room, is where HPCIparameters such as HPCI Area Coolers can be monitored. IAW 34SO-E41-001-2, High Pressure Coolant Injection (HPCI) System7.2.2 Control Room Manual Startup 7.2.2.1 IF required, depress the High Water Level Reset Pushbutton, 2H11-P601.

7.2.2.2 Open 2E41-F059, Lube Oil Clg Wtr Valve. 7.2.2.3 Start 2E41-C002-2, Barom Cndsr Vacuum Pump.

7.2.2.4 Confirm the necessary locations are posted as High Radiation areas, except in an emergency situation. 7.2.2.5 Perform wing steps, 7.2.2.5.1 AND 7.2.2.5.2, in rapid succession: 7.2.2.5.1 Place 2E41-F001, Turb Steam Supply Valve, control switch, to Open, AND Wednesday, January 21, 2015 2:26:47 PM21 ILT-09 RO NRC EXAM VERIFY the red light illuminates. 7.2.2.5.2 Take 2E41-C002-3, Aux Oil Pump, control switch, to the START position. 7.2.2.6 Open 2E41-F006, Pump Discharge Valve. 7.2.2.7 Confirm the following valves OPEN: Turbine Control Valve Turbine Stop Valve 7.2.2.8 Confirm the turbine comes up to speed as directed by 2E41-R612, Flow Control. 7.2.2.9 WHEN flow increases to 790 gpm, confirm 2E41-F012, Min Flow Valve, CLOSED. 7.2.2.10 Confirm closed OR close the following valves: 2E41-F028, Steam Line Drain Valve 2E41-F025, Barom Cndsr Disch To CRW 2E41-F029, Steam Line Drain Valve 2E41-F026, Barom Cndsr Disch To CRWK/A JUSTIFICATION:This question satisfies the K/A statement by requiring the applicant to know in the control roomwhere to monitor HPCI Turbine/Bearing temperatures. The "A" distractor is plausible since the first part is correct. The second part is plausible if theapplicant remembers there are HPCI System temperatures located on this panel and thinking this is where Turbine/Bearing temperatures are monitored. The "C" distractor is plausible if the applicant thinks about the manual startup of RCIC wherethe manual initiation pushbutton is depressed which then starts up RCIC automatically. Theoperator then confirms the automatic startup of RCIC. The HPCI system does not have a manualinitiation pushbutton, therefore the operator has to perform all actions manually to include the opening of E11-F059. The second part is plausible if the applicant remembers there are HPCISystem temperatures located on this panel and thinking this is where Turbine/Bearingtemperatures are monitored. The "D" distractor is plausible if the applicant thinks about the manual startup of RCIC wherethe manual initiation pushbutton is depressed which then starts up RCIC automatically. The operator then confirms the automatic startup of RCIC. The HPCI system does not have a manualinitiation pushbutton, therefore the operator has to perform all actions manually to include theopening of E11-F059. The second part is plausible since it is correct. A. Incorrect - See description above. B. Correct - See description above. C. Incorrect - See description above. D. Incorrect - See description above. Wednesday, January 21, 2015 2:26:47 PM22 ILT-09 RO NRC EXAM

References:

NONEK/A:206000 High Pressure Coolant Injection SystemA4. Ability to manually operate and/or monitor in the control room: (CFR: 41.7 / 45.5 to 45.8)A4.03 Turbine temperatures: BWR-2, 3,4 . . . . . . . . . . . . . 3.1 3.0LESSON PLAN/OBJECTIVE:E41-HPCI-LP-00501, High Pressure Coolant Injection (HPCI), Ver. 6.0, EO 005.003.A.04 References used to develop this question:34SO-E41-001-2, High Pressure Coolant Injection (HPCI) System, Ver. 28.3 Wednesday, January 21, 2015 2:26:47 PM 2 3 ILT-09 RO NRC EXAM 8.209001K2.01 001/00801E21/008.001.A.01/MOD/SYS-B/BOTH/209001K2.01/2/1/F/2/ARB/ELJWhich ONE of the choices below completes the following statements? The power supply for the 2A Core Spray pump is __________ . The power supply for the 2B Core Spray pump is __________ . 4160V 2E, 2R22-S005; 4160V 2F, 2R22-S006 4160V 2E, 2R22-S005; 4160V 2G, 2R22-S007 4160V 2F, 2R22-S006; 4160V 2E, 2R22-S005 4160V 2F, 2R22-S006; 4160V 2G, 2R22-S007 A.B.C.D.Description:The following Table 1 of R22-ELECT-LP-02702 lists the pumps powered from each 4160VEmergency Bus:4160V Bus 2E (2R22-S005) 4160V Bus 2F (2R22-S006)2A PSW Pump 2C PSW Pump2A RHR Pump 2D PSW Pump2A CRD Pump 2C RHR Pump2A Core Spray Pump 2D RHR Pump2A RHRSW Pump 2C RHRSW Pump2A Drywell Chiller Motor 2B CRD Pump4160V Bus 2G (2R22-S007) 2B PSW Pump2B RHR Pump2B Core Spray Pump2B RHRSW Pump 2D RHRSW Pump 2B Drywell Chiller MotorThe power supply for Core Spray pump 2A is 4160V 2E & for 2B is 4160V 2G. K/A JUSTIFICATION:This question satisfies the K/A statement by requiring the applicant to know the power supply toCore Spray pump 2A (4160V 2E) & 2B (4160V 2G).Wednesday, January 21, 2015 2:26:47 PM 2 4 ILT-09 RO NRC EXAMThe "A" distractor is plausible since the first part is correct. The second part is plausible if theapplicant remembers there is a combination of "A" & "B" pumps that are powered by 4160V "E"& "F" (CRD pumps "A" & "B") and thinks the Core Spay pumps are powered by thisarrangement. The "C" distractor is plausible if the applicant remembers that Core Spray 2A is powered by anEmergency Bus but does not remember which one and thinks this is the power supply for CoreSpray 2A. The second part is plausible if the applicant remembers there is a Core Spray pump powered from this bus and thinks it is Core Spray 2B. The "D" distractor is plausible if the applicant remembers that Core Spray 2A is powered by anEmergency Bus but does not remember which one and thinks Core Spray 2A is powered by thisbus. The second part is plausible since it is correct. A. Incorrect - See description above. B. Correct - See description above. C. Incorrect - See description above. D. Incorrect - See description above. Wednesday, January 21, 2015 2:26:47 PM 2 5 ILT-09 NRC EXAM

References:

NONEK/A:209001 Low Pressure Core Spray SystemK2. Knowledge of electrical power supplies to the following: (CFR: 41.7) K2.01 Pump power . . . . . . . . . . . . . . . . . . . . . . . . 3.0* 3.1*LESSON PLAN/OBJECTIVE:E21-CS-LP-00801, Core Spray System, Ver. 5.0, EO 008.001.A.01References used to develop this question:34SO-E21-001-2, Core Spray System, Attachment 2 Page 3 of 3, Ver. 24.0Modified from HLT Database Q#295003AK1.04-002 Original QuestionA concurrent LOSP and LOCA occurs on Unit 1. o4160 VAC "1E" AND "1F" are DE-ENERGIZED and can NOT be recovered.o4160 VAC "1G" is powered from it's associated EDG.Which of the choices below lists the pumps that are available for injection, if needed? A. "1A" RHR, "1A" Core Spray, "1A" CRDB. "1C" RHR, "1C" RHRSW, "1B" CRD C. "1B" RHR, "1B" Core Spray, "1B" RHRSW D. "1D" RHR, "1C" PSW, "1D" PSW Wednesday, January 21, 2015 2:26:47 PM 2 6 ILT-09 NRC EXAM 9.211000K1.05 001/01101C41/011.002.A.04/MOD/SYS-B/BOTH/211000K1.05/2/1/F/2/ARB/ELJUnit 1 was at 100% RTP when a reactor scram occurred.oSeveral control rods did NOT insert (ATWS)oBoron injection is requiredoRWL is 9 inches (lowest RWL reached -5 inches)On Panel 1H11-P603, the SBLC Pump Select switch is placed in the Start Sys "A" position. With the above conditions, 1G31-F001, RWCU Isolation valve, will __________ .1G31-F004, RWCU Isolation valve, will __________ .remain in the open position; remain in the open position remain in the open position; have received an isolation signal have received an isolation signal; remain in the open position have received an isolation signal; have received an isolation signal A.B.C.D.Wednesday, January 21, 2015 2:26:47 PM 2 7 ILT-09 NRC EXAMDescription:Primary control of the SBLC system is by a brass keylock switch which manually activates either pump and both squib valves. The switch is located on Control Room panel H11-P603. Whenthis switch is positioned to actuate either SBLC systems, a Reactor Water Clean-Up SystemIsolation will also be initiated to prevent the removal of the SBLC solution by the filter demin. Reactor Water Cleanup System Outboard Containment Isolation Valve (G31-F004) isinterlocked to the SBLC system to close upon initiation of SBLC. This feature is designed toensure that the SBLC solution enters the RPV and does not enter the RWCU system where theremoval of boron would occur when it is needed to shutdown the reactor. The closure of theRWCU suction valve will trip the RWCU pump. K/A JUSTIFICATION:This question satisfies the K/A statement by requiring the applicant to determine the effect onthe RWCU System (1G31-F004 closing) when initiating the SBLC System. The "A" distractor is plausible if the applicant remembers there is a time when RWCU isolationwill not isolate when SBLC is initiated (Local) and thinks both RWCU valves will remain open. The second part is plausible if the applicant remembers there is a time when RWCU isolationwill not isolate when SBLC is initiated (Local) and thinks both RWCU valves will remain open. The "C" distractor is plausible if the applicant remembers there is one RWCU isolation valvethat will isolate on SBLC initiation and thinks it is the 1G31-F001. The second part is plausible if the applicant remembers there is a time when RWCU isolation will not isolate (Local) whenSBLC is initiated and thinks this RWCU valve will remain open. The "D" distractor is plausible if the applicant remembers BOTH valves will isolate on any ofthe following signals: Low RWL, High Differential Flow, High Area Temperature, or High AreaDifferential Temperature. The second part is plausible since it is correct. A. Incorrect - See description above. B. Correct - See description above. C. Incorrect - See description above. D. Incorrect - See description above.

References:

NONEK/A:211000 Standby Liquid Control SystemWednesday, January 21, 2015 2:26:47 PM 2 8 ILT-09 NRC EXAMK1. Knowledge of the physical connections and/or cause-effect relationships betweenSTANDBY LIQUID CONTROL SYSTEM and the following:

(CFR: 41.2 to 41.9 / 45.7 to 45.8)K1.05 RWCU . . . . . . . . . . . . . . . . . . . . . . . . . . . . 3.4 3.6LESSON PLAN/OBJECTIVE:C41-SBLC-LP-01101, Standby Liquid Control, Ver. 6.1, EO 011.002.A.04References used to develop this question:34SO-C41-003-1, Standby Liquid Control System, Ver. 12.2 Modified from HLT Database Q#LT-011002-007 Original QuestionUnit 1 was at 100% rated power when a reactor scram occurred.

oSeveral control rods did NOT insert (ATWS) oBoron injection is requiredOn panel 1H11-P603, the SBLC Pump Select switch is placed in the Start Sys "A" position.__________ squib valve(s) will detonate; AND, RWCU Isolation valve, __________ will CLOSE.A. Both; 1G31-F001 B. Both; 1G31-F004 C. Only one;; 1G31-F001 D. Only one; 1G31-F004 Wednesday, January 21, 2015 2:26:47 PM 2 9 ILT-09 NRC EXAM 1 0.212000K6.05 001/01001C71/012.003.C.10/MOD/SYS-B/BOTH/212000K6.05/2/1/F/2/JSC/ELJUnit 1 is operating at 30% RTP. The following failure occurs on Turbine Control Valve (TCV) #1 Emergency Trip Supply (ETS)Oil Pressure Sensor:oThe transmitter for TCV #1, ETS Oil Pressure Sensor, begins drifting down and settlesbelow its trip setpointThe TCV Closure RPS Scram setpoint is __________ .Based on the above conditions, a RPS HALF (1/2) Scram __________ be received. 670 psig; will 670 psig; will NOT 1100 psig; will 1100 psig; will NOT A.B.C.D.Wednesday, January 21, 2015 2:26:47 PM 3 0 ILT-09 NRC EXAMDescription:Four independent pressure switches, one on each TCV, monitor the hydraulic trip oil pressure onthe disk dump valve. If trip oil (FASTC) pressure decreases below 670 psig, the pressureswitches open contacts to input a scram signal to RPS. The four channels input to RPS asfollows: CV#1-RPS channnel A1, CV#2-RPS channnel B1, CV#3-RPS channnel A2,CV#4-RPS channnel B2. Any one trip signal will cause a half scram on RPS. A full scram willrequire a trip signal on both RPS sides.K/A JUSTIFICATION:This question satisfies the K/A statement by requiring the applicant to know how a singlemalfunction in a Control Valve pressure sensor will affect RPS logic.The "B" distractor is plausible since the first part is correct. The second part is plausible if theapplicant thinks about the RPS actuation of the MSIVS or Stop Valves (means and extremes)instead of the RPS actuation of the Control Valves (1/4 taken twice). The "C" distractor is plausible if the applicant thinks about the automatic start feature of thestandby pump (1100 psig) instead of the scram setpt (670 psig). The second part is plausible since it is correct.The "D" distractor is plausible if the applicant thinks about the automatic start feature of thestandby pump (1100 psig) instead of the scram setpt (670 psig). The second part is plausible ifthe applicant thinks about the RPS actuation of the MSIVS or Stop Valves (means and extremes)instead of the RPS actuation of the Control Valves (1/4 taken twice).A. Correct - See description above. B. Incorrect - See description above. C. Incorrect - See description above. D. Incorrect - See description above.

References:

NONEK/A:212000 Reactor Protection SystemK6. Knowledge of the effect that a loss or malfunction of the following will have on theREACTOR PROTECTION SYSTEM :

(CFR: 41.7 / 45.7)K6.05 RPS sensor inputs . . . . . . . . . . . . . . . . . . . . . . . . . . 3.5 3.8Wednesday, January 21, 2015 2:26:47 PM 3 1 ILT-09 NRC EXAMLESSON PLAN/OBJECTIVE:C71-RPS-LP-01001, Reactor Protection System, Ver 6.0, EO 012.003.C.10 and 012.005.B.02References used to develop this question:LFD-1-RPS-15, Turbine Control Valve Trip Oil Pressure-Low, Ver 33LFD-1-RPS-14, Turbine Stop Valve-Closure, Ver 66 Modified from HLT Database Q#245000K1.04-001 Original QuestionWith Unit 2 at 35% RTP, which ONE of the below choices correctly completes the followingstatements? (Limit your response ONLY to direct valve input to RPS Logic, NOT plant integrated response.)The MINIMUM number of Turbine Control Valves (TCV) that will DIRECTLY cause at least a RPS HALF (1/2) Scram from a TCV Fast Closure Trip is __________ . The MINIMUM number of Turbine Stop Valves (TSV) that will DIRECTLY cause a RPS FULL Scram from a TSV Closure is __________ . A. two (2); three (3) B. two (2); two (2) C. one (1); three (3) D. one (1); two (2) Wednesday, January 21, 2015 2:26:47 PM 3 2 ILT-09 NRC EXAM 1 1.215003A2.05 001/01202C51/012.003.C.10/NEW/SYS-I/BOTH/215003A2.05/2/1/H/3/ARB/ELJA Unit 2 Reactor startup is in progress. At 10:00, the IRMs indicate as follows: oIRMs A, B, C & D24/125 on Range 6 oIRMs E, F, G & H10/40 on Range 5 IRMs A - D are increasing 3/125 per minute AND IRMs E - H are increasing 9/40 per minute. At 10:01, the voltage at the detector for IRM H decreases to one (1) VDC. With the above IRM conditions, The EARLIEST listed time that the IRMs will initiate a half scram signal is __________ .

IAW the associated ARPs on 2H11-P603, an OD-7 Option 2, Control Rod Position Check, _________ REQUIRED to be performed. 10:01; is 10:01; is NOT 10:03; is 10:03; is NOT A.B.C.D.Description:Phil, this was question 1 of 10 of the previously submitted questions. Changes wereincorporated based on your ES-401-9 comments. Any one of the eight IRMs reaching a downscale (10/125 or 3.2/40), upscale (80/125 or 25.6/40),upscale trip (115/125 or 36.8/40) or an INOP condition will generate a half scram signal, if notbypassed. The power supply output is 100 VDC for application to the detector. A voltagesensing circuit is used to detect decreases in the output voltage and provide a Low High VoltageTrip signal to the Drawer Inop circuit. With the current value on IRMs, all IRMs are below the RPS Scram setpoint of 36.8/40. When the voltage at IRM H drops to one (1) VDC, an IRM Inoptrip will be generated in Channel B making 10:01 the EARLIEST listed time that the IRMs willgenerate a half scram signal. At 10:03, IRMs E, F, G & H will be above their scram setpoint of 36.8/40 (10/40 + 27/40 = 37/40). IAW 34AR-603-118-2, REACTOR AUTO SCRAM SYSTEM B TRIP, 603-118, Note beforestep 5.2.4, "IF surveillance testing is in progress which initiates half scram signals, it isWednesday, January 21, 2015 2:26:48 PM 3 3 ILT-09 NRC EXAMacceptable to delay the performance of the OD-7 Option 2 UNTIL the applicable surveillance iscomplete.

The following step is NOT applicable IF all control rods are inserted.

"K/A JUSTIFICATION:This question satisfies the K/A statement by requiring the applicant to predict the earlist listedtime that the IRMs will initiate a scram signal and then asking the applicant if an OD-7 Option 2, Control Rod Position Check, is required to be performed to mitigate this event. 34AR-603-118-2, will require the Control Rod Position Check to be performed. The "B" distractor is plausible since the first part is correct. The second part is plausible if theapplicant thinks that a full scram is generated and remembers the Note which states that if a full scram is received the step for an OD-7 Option 2 is not required. Also plausible since changingIRM H to IRM A, C, E or G will make the second part correct. The "C" distractor is plausible if the applicant thinks the the voltage drop on IRM H will onlygenerate a rod block but calculates that at 10:03 that the scram setpoint of 36.8/40 on IRMs E, F,G & H has been exceeded at 10:03. The second part is plausible since it is correct. The "D" distractor is plausible if the applicant thinks the the voltage drop on IRM H will onlygenerate a rod block but calculates that at 10:03 that the scram setpoint of 36.8/40 on IRMs E, F, G & H has been exceeded at 10:03. The second part is plausible if the applicant thinks that a fullscram is generated and remembers the Note which states that if a full scram is received the stepfor an OD-7 Option 2 is not required. Also plausible since changing IRM H to IRM A, C, E or G will make the second part correct. A. Correct - See description above. B. Incorrect - See description above. C. Incorrect - See description above. D. Incorrect - See description above. Wednesday, January 21, 2015 2:26:48 PM 3 4 ILT-09 NRC EXAM

References:

NONEK/A:215003 Intermediate Range Monitor (IRM) SystemA2. Ability to (a) predict the impacts of the following on the INTERMEDIATE RANGEMONITOR (IRM) SYSTEM ; and (b) based on those predictions, use procedures to correct, control, or mitigate the consequences of those abnormal conditions or operations: (CFR: 41.5 / 45.6)A2.05 Faulty or erratic operation of detectors/system . . . . . . . . . 3.3 3.5LESSON PLAN/OBJECTIVE:C51-IRM-LP-01202, Intermediate Range Monitors, Ver 5.0, EO 012.003.C.10 References used to develop this question:34AR-603-109-2, Reactor Neutron Monitoring Sys Trip, Ver. 4.5 34AR-603-118-2, Reactor Auto Scram System B Trip, Ver. 4.0 34AR-603-203-2, IRM Bus A Upscale Trip Or Inop, Ver. 3.0 34AR-603-212-2, IRM Bus B Upscale Trip Or Inop, Ver. 3.0 34AR-603-221-2, IRM Upscale, Ver. 2.1 34AR-603-238-2, Rod Out Block, Ver. 3.2Wednesday, January 21, 2015 2:26:48 PM 3 5 ILT-09 NRC EXAM 1 2.215004A3.01 001/01201C51/012.003.A.10/MOD/SYS-B/BOTH/215004A3.01/2/1/H/2/JSC/ELJUnit 2 is performing a Reactor Startup IAW 34GO-OPS-001-2, Plant Startup. oIRMs are on Range 2oSRMs A, B, C, and D are fully inserted in the coreoSRMs C and D are currently selected for withdrawalThe following indications currently exist:Based on these indications, Annunicator ROD OUT BLOCK, (603-238) __________ be illuminated. If the DRIVE OUT pushbutton for SRM/IRM Drive Contol is depressed for 10 seconds andthen released, SRM C & D indications above will __________ . will; stop lowering as soon as the "Drive Out" push button is released will;continue to lower until SRM C & D are fully withdrawn will NOT;stop lowering as soon as the "Drive Out" push button is released will NOT;continue to lower until SRM C & D are fully withdrawn A.B.C.D.Wednesday, January 21, 2015 2:26:48 PM 3 6 ILT-09 NRC EXAMDescription:The SRM UPSCALE rod block prevents rod withdrawal unless all SRM detectors read less than 7 x 10 4 cps. The rod block for this condition will be bypassed when either one of the following:the associated IRMs are on Range 8 (and above), Mode Switch in RUN, or individual SRM isbypassed.The SRM drive mechanism positions the shuttle tube (containing SRM detector chamber) fromfully inserted position (18 inches above core midplane) to fully withdrawn position (30 inchesbelow the bottom of the active core). When inserting detectors, the IN pushbutton is notrequired to be held. Once depressed the insertion will seal in until the IN pushbutton isdepressed a second time. Depressing the OUT pushbutton while the detectors are inserting willnot cancel or stop the insertion. When withdrawing detectors, the OUT signal does not seal in,the OUT pushbutton must be continuously depressed.K/A JUSTIFICATION:This question satisfies the K/A statement by requiring the applicant to monitor the SRM cpsindication and determine the automatic plant response (Rod Block).The "B" distractor is plausible since the first part is correct. The second part is plausible since theDrive circuit has a seal in feature for driving the detectors into the core. When withdrawing the detectors the operator must continuously depress the Drive Out pushbutton for the detectors towithdraw. If the detector were to continuously withdraw, the CPS indication would lower untilthe detector is fully withdrawn. The "C" distractor is plausible if the applicant thinks about the SRM TRIP (3x10 5 cps) setpointinstead of the SRM UPSCALE alarm (7x10 4 cps). The second part is plausible since it iscorrect.The "D" distractor is plausible if the applicant thinks about the SRM TRIP (3x10 5 cps) setpointinstead of the SRM UPSCALE alarm (7x10 4 cps). The second part is plausible since the Drivecircuit has a seal in feature for driving the detectors into the core. When withdrawing thedetectors the operator must continuously depress the Drive Out pushbutton for the detectors towithdraw. If the detector were to continuously withdraw, the CPS indication would lower until the detector is fully withdrawn. A. Correct - See description above. B. Incorrect - See description above. C. Incorrect - See description above. D. Incorrect - See description above.

References:

Wednesday, January 21, 2015 2:26:48 PM 3 7 ILT-09 NRC EXAMK/A:215004 Source Range Monitor (SRM) SystemA3. Ability to monitor automatic operations of the SOURCE RANGE MONITOR (SRM)SYSTEM including:

CFR: 41.7 / 45.7) A3.01 Meters and recorders . . . . . . . . . . . . . . . . . . . . . . . . . . . . 3.2 3.2LESSON PLAN/OBJECTIVE:C51-SRM-LP-01201, Source Range Monitors, Ver 7.0, EO 012.003.A.10, 012.003.A.06References used to develop this question:34AR-603-204-2, SRM UPSCALE OR INOPERATIVE, Ver 1Modified from HLT Database Q#215004K5.03-003 Original QuestionUnit 1 is starting up per 34GO-OPS-001-1, "Plant Startup". The reactor is has already beendeclared CRITICAL.oAll IRMs are on Range 3oSRM detectors are being intermittently withdrawn as required by the procedureAs the SRM "A" detector is being withdrawn, SRM "A" count rate decreases to 150 cps.When SRM "A" count rate reaches 150 cps, a control rod block __________ have already occurred. If the "DRIVE OUT" pushbutton continues to be depressed, the SRM "A" detector

__________ continue to withdraw further.A. will; will B. will; will NOT C. will NOT; will D. will NOT; will NOT Wednesday, January 21, 2015 2:26:48 PM 3 8 ILT-09 NRC EXAM 1 3.215004K2.01 001/01201C51/012.003.A.09/MOD/SYS-B/BOTH/215004K2.01/2/1/F/2/ARB/ELJUnit 2 is starting up with Reactor power at 5% RTP.

2 4/48 VDC Cabinet 2B, 2R25-S016, de-energizes and can NOT be restored. SRM Channels __________ will have lost their power supply.2A & 2C 2A & 2D 2B & 2C 2B & 2D A.B.C.D.Wednesday, January 21, 2015 2:26:48 PM 4 0 ILT-09 NRC EXAMDescription:Source Range Monitor Power Supplies:Channels A & C - 24/48 VDC Cabinet 2A, 2R25-S015Channels B & D - 24/48 VDC Cabinet 2B, 2R25-S016Typically, power supplies are divisionalized where Div. 1 will be powered by the "A" side of a power system and Div. 2 will be powered by the "B" side of a power system. In some cases thisdoes not hold true, i.e. 2P41-F316 A & F316 D, PSW TB Isolation valves are powered by a Div. 1power supply (2R24-S025) and 2P41-F316 B & F316 C are powered by a Div. 2 power supply(2R24-S027) . K/A JUSTIFICATION:This question satisfies the K/A statement by requiring the applicant to know the power supply toSRM Channels B and D. The "A" distractor is plausible if the applicant remembers that this is a power supply to 2 of theSRMs and thinks 2A & 2C SRMs are the SRMs that remain de-energized. Also would becorrect if 2R25-S015, 24/48 VDC Cabinet 2A, is de-energized. The "B" distractor is plausible if the applicant remembers a power arrangement wherecomponents "A" & "D" are powered by the same power supply (PSW 2P41-F316A & F316D) and thinks 2A & 2D SRMs are the SRMs that will be de-energized. The "C" distractor is plausible if the applicant remembers a power arrangement wherecomponents "B" & "C" are powered by the same power supply (PSW 2P41-F316B & F316C)and thinks 2B & 2C SRMs are the SRMs that will be de-energized. A. Incorrect - See description above. B. Incorrect - See description above. C. Incorrect - See description above. D. Correct - See description above. Wednesday, January 21, 2015 2:26:48 PM 4 1 ILT-09 NRC EXAM

References:

NONEK/A:215004 Source Range Monitor (SRM) SystemK2. Knowledge of electrical power supplies to the following: (CFR: 41.7)K2.01 SRM channels/detectors . . . . . . . . . . . . . . . . . . . . . . 2.6 2.8LESSON PLAN/OBJECTIVE:C51-SRM-LP-01201, Source Range Monitors, Ver. 7.0, EO 012.003.A.09 References used to develop this question:A-20159, Plant Hatch Load List, 2R25-S015, Ver. 3.0 A-20160, Plant Hatch Load List, 2R25-S016, Ver. 2.0 Modified from HLT Database Q#215004-011Original QuestionWhich of the below electrical distribution cabinets correctly completes the following statement? The power supply to SRM channels B and D on Unit 2 is _____________.A. 24/48 VDC Cabinet 2A, 2R25-S015 B. 24/48 VDC Cabinet 2B, 2R25-S016 C. 125 VDC A Cabinet, 2R25-S001D. 125 VDC B Cabinet, 2R25-S002Wednesday, January 21, 2015 2:26:48 PM 4 2 ILT-09 NRC EXAM 1 4.215005K5.03 001/05401RMCS/300.010.C.01/MOD/P-AB/BOTH/215005K5.03/2/1/H/3/JSC/ELJUnit 2 is starting up from a refueling outage.oReactor power is 7% RTPThe NPO has just completed moving a Step of Control Rods (symmetrical pattern) to theirwithdraw limit of position 08. Before movement of the next group of control rods, the following occurs, oOne (1) Control Rod starts drifting out from position 08With the above Control Rod drifting out, Notch positions __________ will provide the HIGHEST change in magnitude of LPRMpower indication as the control rod drifts through this range of notch positions. IAW 34AB-C11-004-2, Mispositioned Control Rods, the OATC is required to __________ .20 to 24; select the drifting rod AND drive it to position 08 using the EMERGENCY IN switch 20 to 24; enter 34AB-C71-001-1, Scram Procedure, AND SCRAM the reactor 36 to 40; select the drifting rod AND drive it to position 08 using the EMERGENCY IN switch 36 to 40; enter 34AB-C71-001-1, Scram Procedure, AND SCRAM the reactor A.B.C.D.Description:Wednesday, January 21, 2015 2:26:48 PM 4 3 ILT-09 NRC EXAMThe graph above shows the cold, All Rods-In (ARI) axial power shape for beginning of cycle 24compared to the previous cycle. The strong mid-peaked power shape is the reason for theadditional banking between notches 20 and 30 for BPWS Groups 2-4. (NOTE: the results aboveare hypothetical and assume that the reactor is critical with all-rods-in. However, the results are representative of the axial flux shape during the approach to critical). Remember that the fluxprofiles change over cycle life. The mid-peaked flux/power profile at cold ARI conditionsgradually shifts to the top of the core as the cycle burns. K/A JUSTIFICATION:This question satisfies the K/A statement by asking the applicant to know the axial flux profileof the core which will determine where the higher worth notch positions will be. This in turn willbe indicated by the LPRMs as the rod is moved through this region. The applicant has tounderstand that at the BOC the flux profile is low/mid plane whereas at EOC it shifts to the upper plane region. The "A" distractor is plausible since the first part is correct. The second part is plausible if theapplicant takes the required actions for a Control Rod drifting out when reactor power is abovethe Low Power Set Point (LPCP) of 26% RTP. The "C" distractor is plausible if the applicant thinks about the axial profile for the EOC vice theBOC (startup after refueling outage). The second part is plausible if the applicant takes the required actions for a Control Rod drifting out when reactor power is above the Low Power SetPoint (LPCP) of 26% RTP. The "D" distractor is plausible if the applicant thinks about the axial profile for the EOC vice theBOC (startup after refueling outage). The second part is plausible since it is correct.A. Incorrect - See description above. B. Correct - See description above. C. Incorrect - See description above. D. Incorrect - See description above.

References:

Wednesday, January 21, 2015 2:26:48 PM44 ILT-09 NRC EXAMNONEK/A:215005 Average Power Range Monitor/Local Power Range Monitor SystemK5. Knowledge of the operational implications of the following concepts as they apply toAVERAGE POWER RANGE MONITOR/LOCAL POWER RANGE MONITOR SYSTEM : (CFR: 41.5 / 45.3)K5.03 Control rod symmetrical patterns . . . . . . . . . 2.9 3.3LESSON PLAN/OBJECTIVE:C11-RMCS-LP-05401, Reactor Manual Control System, Ver 6.0, EO 200.091.A. 03References used to develop this question:34AB-C11-004-2, Mispositioned Control Rods, Ver 4.0Modified from HLT Database Q#201001A1.03-001 Original QuestionUnit 1 is at 15% power and is performing a reactor startup.All control rods in the currently latched RWM step are at their Insert Limit.oThe Withdraw Limit for the latched step is position 08oControl rod 30-31 is currently selectedWhen control rod 30-31 is withdrawn, the rod begins DRIFTING OUT. Wednesday, January 21, 2015 2:26:48 PM 4 5 ILT-09 NRC EXAMWhich ONE of the choices below completes the following statements? When control rod 30-31 reaches position 10, the Rod Movement Control "Rod Out" whitelight will be __________; and, Per 34AB-C11-004-1, "Mispositioned Control Rods", a reactor scram __________ required.A. extinquished; is NOT B. illuminated; is NOT C. extinquished; is D. illuminated; is Wednesday, January 21, 2015 2:26:48 PM 4 6 ILT-09 NRC EXAM 1 5.217000K3.03 001/03901E51/039.013.A.01/NEW/SYS-I/BOTH/217000K3.03/2/1/H/3/ARB/ELJAfter a loss of Main Condenser vacuum transient on Unit 2, RCIC is operating in Pressure Control Mode. The RCIC flow controller, 2E51-R612, is in AUTOMATIC with an output of 75%. Subsequently, a malfunction causes 2E51-R612 controller output to drift from 75% to 95%resulting in the following RCIC indications: As the RCIC controller output drifts up to 95%, the reactor Cooldown Rate will _________ .Based on the above indications, RCIC should have __________ . decrease; ONLY automatically tripped decrease; automatically tripped AND isolated increase; ONLY automatically tripped increase; automatically tripped AND isolated A.B.C.D.Description:Phil, this was question 2 of 10 of the previously submitted questions. Changes wereincorporated based on your ES-401-9 comments. RCIC automatically operating in pressure control mode during a scram due to loss of maincondenser vacuum (MSIVs shut) will cause a constant work load to be performed with theturbine. As the failure of the controller output failure occurs which causes its setpt to increasefrom 75% to 95%, the turbine will need more steam to be converted to work thus cooldown rateWednesday, January 21, 2015 2:26:48 PM 4 7 ILT-09 NRC EXAMwill increase.RCIC indications show a high turbine exhaust pressure of 45.7 psig. This causes RCIC to trip on high turbine exhaust pressure (setpoint 40 psig). RCIC System Rupture Disks (D001 and D002) provide protection for the RCIC Turbine casingfrom excessive exhaust pressure. The two diaphragms are in series and are designed to ruptureat 150 psig. High pressure between the diaphragms will cause a RCIC System Isolation at 10psig. K/A JUSTIFICATION:This question satisfies the K/A statement by asking the applicant to determine the effect of aRCIC malfunction has on decay heat removal. With RCIC in operation, the reactor cooldown rate is 7°F/hr. After RCIC flow increases, the turbine will have to perform more work thereforethe cooldown rate will increase. The "A" distractor is plausible if the applicant thinks about the failure of the RCIC flow detectorsenseing a higher output (75% to 95%). This failure would cause the RCIC controller to thinkthat the flow too high and would then correct this by causing the system flow rate to lower toreturn the detector sensed flow to return to its setpoint. This lower flow would cause Cooldownrate to decrease since the turbine is working less. The second part is plausible since it is correct.The "B" distractor is plausible if the applicant thinks about the failure of the RCIC flow detectorsenseing a higher output (75% to 95%). This failure would cause the RCIC controller to thinkthat the flow too high and would then correct this by causing the system flow rate to lower toreturn the detector sensed flow to return to its setpoint. This lower flow would cause Cooldownrate to decrease since the turbine is working less. The second part is plausible if the applicant thinks that the 45.7 psig is enough to cause the isolation (setpoint 10 psig). The applicant doesnot understand that the 10 psig is sensed downstream of a diaphragm that ruptures at 150 psig.45.7 psig would only cause a trip only if the first diaphragm had ruptured already.The "D" distractor is plausible since the first part is correct. The second part is plausible if theapplicant thinks that the 45.7 psig is enough to cause the isolation (setpoint 10 psig). The applicant does not understand that the 10 psig is sensed downstream of a diaphragm that rupturesat 150 psig. 45.7 psig would only cause a trip only if the first diaphragm had ruptured already.A. Incorrect - See description above. B. Incorrect - See description above. C. Correct - See description above. D. Incorrect - See description above. Wednesday, January 21, 2015 2:26:48 PM 4 8 ILT-09 NRC EXAM

References:

NONEK/A:217000 Reactor Core Isolation Cooling System (RCIC)K3. Knowledge of the effect that a loss or malfunction of the REACTOR COREISOLATION COOLING SYSTEM (RCIC) will have on following:

(CFR: 41.7 / 45.4)K3.03 Decay heat removal . . . . . . . . . . . . . . . . . . . . . . . 3.5 3.5LESSON PLAN/OBJECTIVE:E51-RCIC-LP-03901, Reactor Core Isolation Cooling (RCIC), Ver. 6.1, EO 039.013.A.01References used to develop this question:34SO-E51-001-2, Reactor Core Isolation Cooling (RCIC) System, Ver. 25.1 Wednesday, January 21, 2015 2:26:48 PM 4 9 ILT-09 NRC EXAM 16.218000K1.04 001/03801B21/038.004.A.02/MOD/SYS-I/BOTH/218000K1.04/2/1/H/3/ARB/ELJUnit 2 has experienced a Loss of Offsite Power (LOSP). The following conditions existed at 15:00: oALL low pressure ECCS pumps have been manually startedoRPV Pressure ...................... 860 psig controlled by LLSoRWL --------.... -97 inches, lowering at 2 inches/minuteoDrywell Pressure ---..- 0.85 psig, rising at 0.2 psig/minuteoADS Inhibit Switches.......... "Normal" positionGiven these trends, The EARLIEST listed time that the ADS valves will have automatically OPENED is __________ . 15:02 15:04 15:07 15:13 A.B.C.D.Description:

Refer to logic drawing provided as a reference for developing this test item. 1.With a high Drywell pressure signal present (1.85 psig), the following must occur to initiateADS.

a.Low Reactor water level (Level 3) at +3.0"b.Low Reactor water level (Level 1) at -101"c.102.5 second timer timed out d.CS Pump discharge pressure of 152 psig or RHR Pump discharge pressure of 127 psige.Once the 102.5 second timer has timed out, if RHR or CS Pump discharge pressure isavailable, all 7 ADS Valves open2.Initiation of ADS without high Drywell pressure will occur if the following conditionsexist simultaneously: a.Low Reactor water level (Level 3) at +3.0"b.Low Reactor water level (Level 1) at -101"c. High Drywell Pressure Bypass Timer timed out - 11 minutesd.102.5 second timer timed out. Without the high Drywell pressure signal, the 102.5 secondtimer will not initiate until the High Drywell Pressure Bypass Timer times out and 2.a and2.b are present.Wednesday, January 21, 2015 2:26:49 PM50 ILT-09 NRC EXAMe.CS Pump discharge pressure of 152 psig or RHR Pump discharge pressure of 127 psig.f.Once the 102.5 second timer is timed out, if RHR or CS Pump discharge pressure isavailable, all 7 ADS Valves open.At 1502, RWL is <-101" and at 1505, DW pressure rises to 1.85 psig, which starts the 102.5 second timer. At 1507 the 102.5 second timer has timed out and the ADS valves haveautomatically opened. K/A JUSTIFICATION:This question satisfies the K/A statement by requiring the applicant to analyze DW pressure andwith DW pressure trend, this question is asking the effect that this will have on the ADS logic.

The cause-effect is that the ADS logic will initially begin timing the 11 minute timer when RWLlowers to < -101 inches however DW pressure rises to > 1.85 psig and starts the 102.5 secondtimer (11 minute timer irrelative at this point due to it has longer delay time). The "A" distractor is plausible if the applicant assumes the ADS permissives are met in 2minutes when RWL reaches -101 inches causing the ADS valves to auto open. The "B" distractor is plausible if the applicant ignores the DW pressure permissive and assumesthe ADS permissives are met in 4 minutes (based on RPV level rate & 102.5 second timer timingout) causing the ADS valves to auto open. The "D" distractor is plausible if the applicant assumes the ADS permissives are met in 13minutes (based on RWL reaching -101 inches in 2 minutes and DW pressure bypass timing outin 11 minutes) causing the ADS valves to auto open. A. Incorrect - See description above. B. Incorrect - See description above. C. Correct - See description above. D. Incorrect - See description above.

References:

NONEK/A:218000 Automatic Depressurization SystemK1. Knowledge of the physical connections and/or cause effect relationships betweenAUTOMATIC DEPRESSURIZATION SYSTEM and the following: (CFR: 41.2 to 41.9 / 45.7 to 45.8) K1.04 Drywell/containment pressure: Plant-Specific . . . . . . . . . . 3.9 4.2 Wednesday, January 21, 2015 2:26:49 PM 5 1 ILT-09 NRC EXAMLESSON PLAN/OBJECTIVE:B21-ADS-LP-03801 "Automatic Depressurization System (ADS)", Ver 4.0, EO 038.004.a.02 References used to develop this question:B21-ADS-03801, Automatic Depressurization System (ADS), Fig 2 & Fig 4 34SO-B21-001-2, ADS and LLS System, Ver 13.14Modified from HLT-7 NRC Exam Q#20 ORIGINAL QUESTION (HLT-7 NRC Exam Q#20) Unit 2 has experienced a Loss of Offsite Power (LOSP). The following conditions existed at 15:00: oReactor-------- All rods inoRPV Pressure...................... 860 psig controlled by LLSoRWL--------.... -93 inches, decreasing at 2 inches/minuteoDrywell Pressure---...- 0.6 psig, increasing at 0.05 psi/minute oADS Inhibit Switches.......... "Normal" positionGiven these trends, which ONE of the following predicts the EARLIEST time that the ADS valves will automatically open? A.15:04 B.

15:06 C.

15:15 D.

15:17 Wednesday, January 21, 2015 2:26:49 PM 5 2 ILT-09 NRC EXAM 1 7.218000K5.01 001/03801B21/038.001.A.04/BANK/SYS-B/BOTH/218000K5.01/2/1/H/3/ARB/ELJUnit 2 was operating at 100% RTP when a LOCA occurred. oADS logic automatically opened the ADS valves during the transientWhen both ADS Inhibit Switches are placed in the "INHIBIT" position, the ADS valves will __________ AND, the 102.5 second ADS timer __________ . close; will reset close; will NOT reset remain open; will reset remain open; will NOT reset A.B.C.D.Description:Two ADS Inhibit Switches (Normal/Inhibit) prevent ADS actuation when placed to INHIBIT, byopening contacts in the ADS circuit. A white light above each switch will light to alert the operator of this condition. If the white light above each switch is not illuminated, ADS may notbe inhibited. If these switches are placed to inhibit with the ADS valves open from anautomatic initiation signal, the valves will close. If all initiation signals are present and theInhibit switches are placed in Normal from Inhibit, all ADS valves will immediately open. Thisis because the 102.5 second timers and 11 minute timers are NOT reset when the Inhibit switchis placed in INHIBIT. See figure below. Wednesday, January 21, 2015 2:26:49 PM 5 3 ILT-09 NRC EXAMK/A JUSTIFICATION:This question satisfies the K/A statement by requiring the applicant to understand the operationof the ADS logic when the Inhibit switches are positioned and the impact this operation has onthe ADS valves (close) and 102.5 second timer (will NOT reset). The "A" distractor is plausible since the first part is correct. The second part is plausible if theapplicant remembers that the Inhibit switch will de-energize one side of the "A" logic and oneside of the "B" logic and thinks it is the side that the 102.5 second timer is located on. The "C" distractor is plausible if the applicant remembers that the "A" Inhibit switch willde-energize one side of the "A" logic and the "B" Inhibit switch will de-energize one side of the"B" logic and thinks it is the path that the 102.5 second timer is located on, in which case would reset the 102.5 second timer but leave the ADS valves open. The second part is plausible if theapplicant remembers that the "A" Inhibit switch will de-energize one side of the "A" logic andthe "B" Inhibit switch will de-energize one side of the "B" logic and thinks it is the side that the 102.5 second timer is located on. The "D" distractor is plausible if the applicant remembers that the "A" Inhibit switch willde-energize one side of the "A" logic and the "B" Inhibit switch will de-energize one side of the"B" logic and thinks it is the path that the 102.5 second timer is located on, in which case wouldreset the 102.5 second timer but leave the ADS valves open. The second part is plausible since it is correct. A. Incorrect - See description above. B. Correct - See description above. C. Incorrect - See description above. D. Incorrect - See description above. Wednesday, January 21, 2015 2:26:49 PM 5 4 ILT-09 NRC EXAM

References:

NONEK/A:218000 Automatic Depressurization SystemK5. Knowledge of the operational implications of the following concepts as they apply toAUTOMATIC DEPRESSURIZATION SYSTEM :

(CFR: 41.5 / 45.3)K5.01 ADS logic operation . . . . . . . . . . . . . . 3.8 3.8LESSON PLAN/OBJECTIVE:B21-ADS-LP-03801, Automatic Depressurization System (ADS), Ver. 4.1, EO 038.001.A.04 &EO 038.003.A.02 References used to develop this question:34SO-B21-001-2, Automatic Depressurization (ADS) And Low-Low Set (LLS) Systems, Ver. 13.14 Wednesday, January 21, 2015 2:26:49 PM 5 5 ILT-09 NRC EXAM 1 8.219000A2.13 001/20310PC/201.074.A.02/MOD/P-EOP/BOTH/219000A2.13/2/2/H/2/JSC/ELJUnit 2 is operating at 100% RTP when the following occurs;At 10:00, o'M' SRV inadvertantly opensoSuppression Pool temperature is 94°F and rising 1°F/minuteAt 10:01, oShift Supervisor directs a NPO to place RHR Loop A into Suppression Pool Cooling IAW34SO-E11-010-2, Residual Heat Removal SystemAt 10:10, o'M' SRV is closedoSuppression Pool temperature is 104°F and steadyWith the above conditions,At 10:02 and IAW 34SO-E11-010-2, 2E11-F047A, Hx Inlet Valve, __________ required to be CLOSED prior to starting the first RHR pump. At 10:10, and IAW 34AB-T23-003-2, Torus Temperature Above 95° F, ALL available RHR Loops __________ required to be placed into Suppression Pool Cooling. is; are is; are NOT is NOT; are is NOT; are NOT A.B.C.D.Description:Per 34AB-T23-003-2, Torus Temperature Above 95°F, 4.3 states "IF Suppression Pool bulkaverage temperature exceeds 95°F, PLACE RHR in Suppression Pool cooling per 34SO-E11-010-2, Residual Heat Removal".Per 34SO-E11-010-2, Inlet to the RHR Hx is initially isolated to prevent damage to the Hx fromhydraulic shock created by starting the RHR Pump. Step 7.2.5.1.5 states:UNLESS directed here by the Emergency Operating Procedures, OR UNLESS one RHR Pump is already in service, CLOSE 2E11-F047A, Hx Inlet Vlv. Wednesday, January 21, 2015 2:26:49 PM 5 6 ILT-09 NRC EXAMOnce Torus temperature exceeds 100°F, the Primary Containment Control flowchart will haveALL AVAILABLE SUPPRESSION POOL COOLING PLACED IN SERVICE. Placing allavailable suppression pool cooling in service does not require the HX Inlet valve to be closed. K/A JUSTIFICATION:This question satisfies the K/A statement by requiring the applicant to be able to predict theimpact high torus temperature will have on Suppression Pool Cooling. The higher SuppressionPool temperature effects the RHR Hx Inlet valve which now requires the Hx to be isolated priorto starting the first RHR pump based strictly on high Suppression Pool temperature. If Suppression Pool temperature was even higher (>100°F), the Hx would then not be required tobe isolated. The "B" distractor is plausible since the first part is correct. The second part is plausible if theapplicant thinks about the TS requirement to secure all testing that puts heat into the Suppression Pool before 105°F instead of the requirement to place all available Suppression Pool Cooling inservice at 100°F. The 105°F statement about the temperature is also located in the34AB-T23-003-2 which adds plausibility to the applicant remembering the wrong temperature. The "C" distractor is plausible since this is the action that the operator would take if required toplaced all Suppression Pool Cooling in service once >100°F. The second part is plausible since it is correct. The "D" distractor is plausible since this is the action that the operator would take if required toplaced all Suppression Pool Cooling in service once >100°F. The second part is plausible if theapplicant thinks about the TS requirement to secure all testing that puts heat into the SuppressionPool before 105°F instead of the requirement to place all available Suppression Pool Cooling in service at 100°F. The 105°F statement about the temperature is also located in the34AB-T23-003-2 which adds plausibility to the applicant remembering the wrong temperature. A. Correct - See description above. B. Incorrect - See description above. C. Incorrect - See description above. D. Incorrect - See description above.

References:

NONEK/A:219000 RHR/LPCI: Torus/Suppression Pool Cooling ModeA2. Ability to (a) predict the impacts of the following on the RHR/LPCI:TORUS/SUPPRESSION POOL COOLING MODE ; and (b) based on those predictions,Wednesday, January 21, 2015 2:26:49 PM 5 7 ILT-09 NRC EXAMuse procedures to correct, control, or mitigate the consequences of those abnormalconditions or operations:

(CFR: 41.5 / 45.6)A2.13 High suppression pool temperature . . . . . . . . . . . . . . . . 3.5 3.7LESSON PLAN/OBJECTIVE:EOP-PC-LP-20310, Primary Containment Control (PC), Ver 3.0, EO 201.074.A.02References used to develop this question:31EO-EOP-012-2, Primary Containment Control, Ver 634AB-T23-003-2, Torus Temperature above 95°F, Ver 2.434SO-E11-010-2, Residual Heat Removal System, Ver 40.4Modified from HLT Database Q#201001A1.03-001 which was used on NRC Exam 2009-301 Q#43Original QuestionUnit 1 is operating at 90% power.oA Safety Relief Valve (SRV) inadvertently opened causing Suppression Pool watertemperature to increaseoSuppression Pool water temperature reaches 97°F before operators are able to close theSRVIAW 34AB-T23-003-1, "Torus Temperature Above 95°F", which ONE of the below choicescorrectly completes the following Residual Heat Removal (RHR) Suppression Pool Coolingalignment statement?Place __________ loop(s) of RHR in Suppression Pool cooling, and the RHR heat exchanger __________ required to be isolated prior to starting the RHR pumpA. only one; is B. only one; is NOT C. all available; is D. all available; is NOT Wednesday, January 21, 2015 2:26:49 PM 5 8 ILT-09 NRC EXAM 1 9.223002A4.01 001/01301T23/013.045.A.05/NEW/SYS-I/BOTH/223002A4.01/2/1/H/3/ARB/ELJUnit 2 is operating at 100% RTP. At 10:00, an event occurred which resulted in the following conditions: oHPCI Equipment (Pipe Penetration) Room, 170°FoTorus Area Ambient Temperature, 170°FAt 10:15, oHPCI Equipment (Pipe Penetration) Room, 185°FoTorus Area Ambient Temperature, 185°FWith the above conditions,The EARLIEST listed time that 2E41-F002, HPCI Isolation valve, should have received an automatic isolation signal is __________ .2E41-F002 valve position __________ be monitored on 2H11-P601 Vertical Display.10:00; can 10:00; can NOT 10:15; can 10:15; can NOT A.B.C.D.Description:Any one of the following conditions will cause a Group 3 (HPCI) isolation:1)HPCI Turbine Exhaust Diaphragm Press High (10 psig)2)HPCI Steam Line Flow High U2 (202 in. H 2O dp or -100 in. H 2O dp)3)HPCI Steam Line Pressure Low (134 psig)4) HPCI Equipment (Pipe Penetration) Room Temp High (165°F)5)Suppression Chamber Area Air Temp High (165°F) (14 min TD)6)Suppression Chamber Area Diff Air Temp High (36°F) (14 min TD) 7)Emergency Area Cooler Temp High (165°F)Any one of the following conditions will cause a Group 4 (RCIC) isolation:1) Suppression Chamber Area Air Temp High (165°F) (29 min TD)2)Suppression Chamber Area Diff Air Temp High (36°F) (29 min TD) 3)RCIC Turbine Exhaust Diaphragm Press High (10 psig)Wednesday, January 21, 2015 2:26:49 PM 5 9 ILT-09 NRC EXAM4)RCIC Steam Line Flow High 143" H 2O or -100" H 2 O5)RCIC Steam Line Pressure Low (95 psig)6)RCIC Equipment Room Temp High (165°F)With the HPCI Pipe Penetration Room temperature above 165°F, the HPCI system will receivean auto isolation signal without a time delay. The 2H11-P601 Vertical Display has multiple system valve indications, including some but NOTall from the HPCI System. 2E41-F002, Inboard Isolation valve, is one of the valves listed but2E41-F041, Inboard Suction valve, is NOT indicated. Both are isolation valves and will close ona Group 3 signal. K/A JUSTIFICATION:This question satisfies the K/A statement by requiring the applicant to determine (monitor)2E41-F002 valve closure of the Primary Containment Isolation System (PCIS) in the Main Control Room. The operator can monitor valve position on Main Control Room panel2H11-P601. The "B" distractor is plausible since the first part is correct. The second part is plausible if theapplicant remembers that some of the HPCI Isolation valves are on the Vertical Display and thinks about the 2E41-F041 (which is NOT displayed) instead of the 2E41-F002 which isdisplayed. The "C" distractor is plausible if the applicant thinks the Torus area ambient temperatureisolation has a 14 minute time delay or does not realize that the HPCI Pipe Penetration(equipment) room temperature isolation does not have a time delay at all. The second part is plausible since it is correct. The "D" distractor is plausible if the applicant thinks the Torus area ambient temperatureisolation has a 14 minute time delay or does not realize that the HPCI Pipe Penetration(equipment) room temperature isolation does not have a time delay at all. The second part isplausible if the applicant remembers that some of the HPCI Isolation valves are on the Vertical Display and thinks about the 2E41-F041 (which is NOT displayed) instead of the 2E41-F002which is displayed. A. Correct - See description above. B. Incorrect - See description above. C. Incorrect - See description above. D. Incorrect - See description above. Wednesday, January 21, 2015 2:26:49 PM 6 0 ILT-09 NRC EXAM

References:

NONEK/A:223002 Primary Containment Isolation System/Nuclear Steam Supply Shut-OffA4. Ability to manually operate and/or monitor in the control room:(CFR: 41.7 / 45.5 to 45.8)A4.01 Valve closures . . . . . . . . . . . . . . . . . . . . . . . 3.6 3.5LESSON PLAN/OBJECTIVE:T23-PC-LP-01301, Primary Containment, Ver. 7.1, EO 013.045.A.05References used to develop this question:34AB-C71-001-2, Scram Procedure, Ver. 12.4 Wednesday, January 21, 2015 2:26:49 PM 6 1 ILT-09 NRC EXAM 2 0.223002K6.02 001/01401B21/014.007.A.01/NEW/SYS-B/BOTH/223002K6.02/2/1/H/3/JSC/ELJUnit 2 is operating at 100% RTP when the following occurs:

o RPS A DE-ENERGIZESTwo (2) minutes later, a DC Switchgear DE-ENERGIZES as indicated below on Panel, 2H11-P651; Based on the above electrical losses, The 125/250 VDC Battery Switchgear that de-energized is __________ . Ten (10) seconds later, __________ of the Main Steam Isolation Valves (MSIVs) will haveautomatically isolated. 125/250 VDC Batt Swgr 2B, 2R22-S017; four (4) 125/250 VDC Batt Swgr 2B, 2R22-S017; none 125/250 VDC Batt Swgr 2A, 2R22-S016; four (4) 125/250 VDC Batt Swgr 2A, 2R22-S016; none A.B.C.D.Description:Wednesday, January 21, 2015 2:26:49 PM 6 2 ILT-09 NRC EXAMValve operation is controlled by a 4-way spool valve which is positioned by two solenoids, oneAC powered and one DC powered. With both solenoids de-energized the spool valve supplies pneumatics to close the MSIV. With either of the solenoids energized the spool is positioned toopen the MSIV. Having both AC and DC Solenoid Valves provides redundancy to prevent anisolation on the loss of one power supply and also provides a means of opening the MSIVs in an emergency, if either AC or DC power is available. The inboard MSIVs solenoids are poweredfrom RPS A and R25-S001. The outboard MSIV solenoids are powered from RPS B andR25-S002. These solenoids can also be verified energized by checking the RED LEDs areilluminated on the vertical section of H11-P602 for the Inboard MSIVs or H11-P601 for theOutboard MSIVs. These LEDs were added to allow the Operator to monitor the status of each ofthe MSIVs solenoid valves to reduce the possibility of an inadvertent MSIV closure duringtesting. R25-S001 (third row) and R25-S003 (second and fourth row) are both powered from 125/250VDC Batt Swgr 2A, 2R22-S016. R25-S002 (top row) is powered from 125/250 VDC Batt Swgr 2B, 2R22-S017. K/A JUSTIFICATION:This question satisfies the K/A statement by requiring the applicant to know the effect on theMSIVs (closing and isolating the Main Steam Lines) upon on a loss of DC power (2R25-S001) to the pilot valve solenoid with the given plant conditions. The "A" distractor is plausible if the applicant thinks that 2R25-S002 is de-energized since2R22-S016 is lost. An applicant could think this since 2R22-S016 (even numbered) wouldsupply power to the 125/250 VDC Battery Switchgear 2B side based on normal logic scheme of A/C/1/3 and B/D/2/4 and etc.. The second part is plausible since it is correct. The "B" distractor is plausible if the applicant thinks that 2R25-S002 is de-energized since2R22-S016 is lost. An applicant could think this since 2R22-S016 (even numbered) wouldsupply power to the 125/250 VDC Battery Switchgear 2B side based on normal logic scheme ofA/C/1/3 and B/D/2/4 and etc.. The second part is plausible if the applicant thinks the power supply for the MSIV solenoids is similiar to that of Inboard PCIVs in that those are AC poweredsince they are inside the Drywell and the Outboard PCIVs are DC powered. The "D" distractor is plausible since the first part is correct. The second part is plausible if theapplicant thinks the power supply for the MSIV solenoids is similiar to that of Inboard PCIVs inthat those are AC powered since they are inside the Drywell and the Outboard PCIVs are DC powered. A. Incorrect - See description above. B. Incorrect - See description above. C. Correct - See description above. D. Incorrect - See description above. Wednesday, January 21, 2015 2:26:49 PM 6 3 ILT-09 NRC EXAM

References:

NONEK/A:223002 Primary Containment Isolation System/Nuclear Steam Supply Shut-OffK6. Knowledge of the effect that a loss or malfunction of the following will have on thePRIMARY CONTAINMENT ISOLATION SYSTEM/NUCLEAR STEAM SUPPLY SHUT-OFF : (CFR: 41.7 / 45.7)K6.02 D.C. electrical distribution . . . . . . . . . . . . . . . . . . . . . . . . 3.0 3.2LESSON PLAN/OBJECTIVE:B21-SLLS-LP-01401, Main Steam and Low Low Set, Ver 9.1, EO 014.007.A.01References used to develop this question:34AB-R22-001-2, Loss of DC Buses, Ver 4.3 Wednesday, January 21, 2015 2:26:49 PM 6 4 ILT-09 NRC EXAM 2 1.230000K2.02 001/00701E11/006.001.A.02/MOD/SYS-B/BOTH/230000K2.02/2/2/F/2/ARB/ELJUnit 2 experiences a Loss of Offsite power. o4160V 2G is the ONLY 4160V bus that is ENERGIZEDBased on the above conditions,RHR pump 2B __________ be used for Suppression Pool Spray. RHR pump 2D __________ be used for Suppression Pool Spray. can; can can; can NOT can NOT; can can NOT; can NOT A.B.C.D.Wednesday, January 21, 2015 2:26:49 PM 6 5 ILT-09 NRC EXAMDescription:AC Power supplies to RHR pumps:RHR Pump "A" - Powered by 4160 VAC Emergency Bus "2E", 2R22-S005 RHR Pump "C" & "D" - Powered by 4160 VAC Emergency Bus "2F", 2R22-S006 RHR Pump "B" - Powered by 4160 VAC Emergency Bus "2G", 2R22-S007 EDG 2C supplies power to 4160 VAC Emergency Bus "2G", 2R22-S007. RHRSW pump are powered by: 4160 VAC Bus "2E" supplies RHRSW "2A" 4160 VAC Bus "2F" (shared EDG suplies bus) supplies RHRSW "2C" 4160 VAC Bus "2G" supplies RHRSW "2B" and RHRSW "2D" K/A JUSTIFICATION:This question satisfies the K/A statement by requiring the applicant to determine which RHRpumps will be available for Torus Spray by knowing the Normal, Alternate & Emergency powersupplies to each RHR pump. The "A" distractor is plausible since the first part is correct. The second part is plausible if theapplicant remembers 4160V 2F powers two (2) RHR pumps (2C & 2D) and thinks it is RHR 2B& 2D, leaving 2D RHR pump available for Suppression Pool Spray. The "C" distractor is plausible if the applicant remembers that one (1) Division 2 RHR pump ispowered from 2F & one (1) Division 2 RHR pump is powered from 2G and thinks 2B RHRpump is powered from 2F 4160V bus. The second part is plausible if the applicant remembers 4160V 2F powers two (2) RHR pumps (2C & 2D) and thinks it is RHR 2B & 2D, leaving 2DRHR pump available for Suppression Pool Spray. The "D" distractor is plausible if the applicant remembers that one (1) Division 2 RHR pump ispowered from 2F & one (1) Division 2 RHR pump is powered from 2G and thinks 2B RHRpump is powered from 2F 4160V bus. The second part is plausible since it is correct. A. Incorrect - See description above. B. Correct - See description above. C. Incorrect - See description above. D. Incorrect - See description above. Wednesday, January 21, 2015 2:26:49 PM 6 6 ILT-09 NRC EXAM

References:

NONEK/A:230000 RHR/LPCI: Torus/Suppression Pool Spray ModeK2. Knowledge of electrical power supplies to the following: (CFR: 41.7)K2.02 Pumps . . . . . . . . . . . . . . . . . . . . . . . . . . . . . . . . . . . 2.8* 2.9*LESSON PLAN/OBJECTIVE:E11-RHR-LP-00701, Residual Heat Removal System, EO 006.001.A.02 References used to develop this question:34SO-E11-010-1, Residual Heat Removal System, Ver. 44.1 Used on NRC Exam 2011-301 Q#19Modified from HLT Database Q#230000K2.02 001Original QuestionUnit 1 experiences a Loss of Offsite power. o4160V 1G is the ONLY 4160V bus that is ENERGIZED.Which ONE of the following RHR pumps can be used for Torus Spray?A. RHR pump 1AB. RHR pump 1BC. RHR pump 1CD. RHR pump 1DWednesday, January 21, 2015 2:26:49 PM 6 7 ILT-09 NRC EXAM 2 2.239002K1.07 001/01301PC/300.006.A.22/NEW/P-EOP/BOTH/239002K1.07/2/1/F/3/JSC/ELJUnit 2 is operating at 100% RTP when at transient occurs resulting in Suppression Pool levellowering with the following times and levels: TIME LEVEL10:00 147 inches10:02 145 inchesBased on the above conditions and IAW TS 3.6.2.2, Suppression Pool Water Level, theEARLIEST listed time that a Required Action Statement (RAS) is entered is __________ . If Suppression Pool level lowers to 60 inches, the SRV T-Quenchers will be __________ .10:00;covered 10:00;uncovered10:02;covered 10:02;uncovered A.B.C.D.Description:Per 31EO-OPS-001-0, Per Unit 2 TS,3.6.2.2 Suppression Pool Water LevelLCO 3.6.2.2 Suppression pool water level shall be > 146 inches and < 150 inches.APPLICABILITY: MODES 1, 2, and 3.K/A JUSTIFICATION:Wednesday, January 21, 2015 2:26:49 PM 6 8 ILT-09 NRC EXAMK/A JUSTIFICATION:This question satisfies the K/A statement by requiring the applicant to know the physicallocation of the SRV t-quenchers in the Torus. This is important so that the unit does not open anSRV into the torus with level too low which would directly pressurize the containment.The "A" distractor is plausible if the applicant remembers the Torus water level low level alarmsetpt of 147.25 inches instead of the TS requirement of 146 inches. The second part is plausiblesince it is correct. The "B" distractor is plausible if the applicant remembers the Torus water level low level alarmsetpt of 147.25 inches instead of the TS requirement of 146 inches. The second part is plausibleif the applicant thinks about the Unit 1 Suppression Pool level of 63 inches for uncovering the SRV T-quenchers vice the Unit 2 Suppression Pool level of 57.5 inches.The "D" distractor is plausible since the first part is correct. The second part is plausible if theapplicant thinks about the Unit 1 Suppression Pool level of 63 inches for uncovering the SRVT-quenchers vice the Unit 2 Suppression Pool level of 57.5 inches. A. Incorrect - See description above. B. Incorrect - See description above. C. Correct - See description above. D. Incorrect - See description above.

References:

NONEK/A:239002 Relief/Safety ValvesK1. Knowledge of the physical connections and/or cause-effect relationships betweenRELIEF/SAFETY VALVES and the following: (CFR: 41.2 to 41.9 / 45.7 to 45.8)K1.07 Suppression pool . . . . . . . . . . . . . . . . . . . . . . . . . . . . 3.6 3.8LESSON PLAN/OBJECTIVE:T23-PC-LP-01301, Primary Containment, Ver 7.1, 300.006.A.22References used to develop this question:Unit 2 TS34AR-602-235-2, Torus Water Level High/Low, Ver 2.5Wednesday, January 21, 2015 2:26:49 PM 6 9 ILT-09 NRC EXAM 2 3.245000K5.07 001/02401R13/024.001.A.05/MOD/P-NORM/BOTH/245000K5.07/2/2/F/3/ARB/ELJUnit 2 is operating at 100% RTP when 2R13-C008A, Isophase Bus Cooling Unit fan, trips. With the above conditions,The 2R13-C008B, Isophase Bus Cooling Unit fan, __________ . If 2R13-C008B, Isophase Bus Cooling Unit fan, subsequently trips, the MAXIMUM Unit 2 Main Generator output limit is __________ . will already be running; 12800 amps will already be running; 14000 amps must be manually started; 12800 ampsmust be manually started; 14000 amps A.B.C.D.Wednesday, January 21, 2015 2:26:49 PM 7 0 ILT-09 NRC EXAMDescription:Unit 2 normal configuration for Isophase Bus Duct Cooling is one fan running with the other fan off. If the "A" fan trips, the "B" fan will not auto start and must be manually started. Thenormal Unit 1 configuration for Isophase Bus Duct Cooling is for both fans running. If a total loss of bus duct cooling occurs, the Main Generator output will have to be reduced towithin the self-cooled rating of the buses. This rating is 14,000 amps for Unit 2 and 12,800amps for Unit 1. K/A JUSTIFICATION:This question satisfies the K/A statement by requiring the applicant to know the auxiliary systemnormal operation and the operational implication (Turbine/Generator reduced output) due to the limitations on operating the Main Generator without Isophase Bus Cooling Unit fans in service.The "A" distractor is plausible if the applicant remembers that one Unit's Isophase Bus CoolingUnit Fans are BOTH normally in service and thinks it is Unit 2, therefore when the C008A fantrips, thinks C008B will already be in service. Also would be correct if asking the question on Unit 1. The second part is plausible if the applicant remembers the 12800 amp value and thinksit applies to Unit 2 instead of the 14000 amp limit. Also would be correct if asking the questionon Unit 1. The "B" distractor is plausible if the applicant remembers that one Units Isophase Bus CoolingUnit Fans are BOTH normally in service and thinks it is Unit 2, therefore when the C008A fan trips, thinks C008B will already be in service. Also would be correct if asking the question onUnit 1. The second part is plausible since it is correct. The "C" distractor is plausible since it is correct. The second part is plausible if the applicantremembers the 12800 amp value and thinks it applies to Unit 2 instead of the 14000 amp limit. Also would be correct if asking the question on Unit 1. A. Incorrect - See description above. B. Incorrect - See description above. C. Incorrect - See description above. D. Correct - See description above. Wednesday, January 21, 2015 2:26:49 PM 7 1 ILT-09 NRC EXAM

References:

NONEK/A:245000 Main Turbine Generator and Auxiliary SystemsK5. Knowledge of the operational implications of the following concepts as they apply toMAIN TURBINE GENERATOR AND AUXILIARY SYSTEMS : (CFR: 41.5 / 45.3)K5.07 Generator operations and limitations . . . . . . . . . . . . 2.6 2.9LESSON PLAN/OBJECTIVE:R13-LP-02401, Isophase Bus Duct Cooling, Ver. 4.0, EO 024.001.A.05References used to develop this question:34SO-N40-001-1, Main Generator Operation, Ver. 17.4 34SO-N40-001-2, Main Generator Operation, Ver. 19.2 Modified from HLT Database Q#LT-024001-001 Original QuestionUnit 1 is operating at 100% RTP.Which ONE of the choices below, completes the following statements? The MAXIMUM Unit 1 Main Generator output limit following a complete loss of IsophaseBus Duct Cooling is __________ amps.If the "A" Isophase Bus Duct Cooling fan trips, the "B" Isophase Bus Duct Cooling fan will __________ .A. 12,800 amps; be off B. 14,000 amps; be off C. 14,000 amps; be running D. 12,800 amps; be running Wednesday, January 21, 2015 2:26:49 PM 7 2 ILT-09 NRC EXAM 2 4.256000K6.01 001/03501P51/P70/EO 200.025.A.05/BANK/SYS-B/BOTH/256000K6.01/2/2/F/3/JSC/ELJUnit 2 is operating at 100% RTP when a TOTAL loss of Plant Air occurs. Based on the above conditions,The MAXIMUM listed Control Air pressure at which 2N21-F111, Feedwater Startup LevelControl Valve, will be LOCKED UP in its existing position is __________ .2N21-F117A and 2N21-F117B

, RFPT Minimum flow isolation valves, will __________ . 49 psig; fail open49 psig;remain closed 74 psig;fail open 74 psig;remain closed A.B.C.D.Wednesday, January 21, 2015 2:26:49 PM 7 3 ILT-09 NRC EXAMDescription:On a loss of air at 50 psig, the Feedwater Startup Level Control Valve, 2N21-F111, LOCKS UPin its existing position. Unit 2 Condensate, Condensate Booster, and Reactor Feed Pump minimum flow (Isolation andControl) valves fail OPEN as air pressure is reduced, possibly decreasing the flow available tothe Reactor. Both the isolation valve and control valve are hydraulically operated via air. This is not true for Unit 1 isolation valve. The Unit 1 isolation valve is a motor operated valve thereforeon a loss of air only the control valve will fail open not the isolation valve. K/A JUSTIFICATION:This question satisfies the K/A statement by requiring the applicant to know theCondensate/Feed system ( RFPT minimum flow protection and startup level control valve)response to a loss of Air. The "B" distractor is plausible since the first part is correct. The second part is plausible sinceUnit 1 isolation valve (motor operated) will not fail open on a loss of air. This is a Unitdifference between the two plants.The "C" distractor is plausible since this is the pressure (75 psig) at which Turbine BuildingInstrument Air Pressure, standby prefilter and afterfilter are automatically put into service. Thesecond part is plausible since it is correct.The "D" distractor is plausible since this is the pressure (75 psig) at which Turbine BuildingInstrument Air Pressure, standby prefilter and afterfilter are automatically put into service. The second part is plausible since Unit 1 isolation valve (motor operated) will not fail open on a lossof air. This is a Unit difference between the two plants.A. Correct - See description above. B. Incorrect - See description above. C. Incorrect - See description above. D. Incorrect - See description above. Wednesday, January 21, 2015 2:26:49 PM 7 4 ILT-09 NRC EXAM

References:

NONEK/A:256000 Reactor Condensate SystemK6. Knowledge of the effect that a loss or malfunction of the following will have on theREACTOR CONDENSATE SYSTEM : (CFR: 41.7 / 45.7)K6.01 Plant air systems . . . . . . . . . . . . . . . . . . . . 2.8 2.8LESSON PLAN/OBJECTIVE:P51-P52-P70-PLANT AIR-LP-03501, Plant Air Systems, Ver 3.0, EO 200.025.A.05 References used to develop this question:34AB-P51-001-2, Loss Of Instrument & Service Air System Or Water Instrusion Into TheService Air System, Ver 4.9 34SO-N21-007-2, Condensate and Feedwater System, Ver 52.1Wednesday, January 21, 2015 2:26:49 PM 7 5 ILT-09 NRC EXAM 2 5.259001G2.4.35 001/00201N21/002.006.A.03/NEW/SYS-B/BOTH/259001G2.4.35/2/2/F/3/ARB/ELJUnit 1 is operating at 100% RTP when an event occurs requiring the Main Control Room to beevacuated. The ONLY action taken prior to leaving the Main Control Room was to manually scram thereactor. Control has been established at the Remote Shutdown panels. The Shift Supervisor dispatches an operator to locally trip RFPT 1A. The operator will depress the local RFPT 1A trip pushbuttons at the __________ . After RFPT 1A is tripped locally, without any additional operator actions, RFPT 1A will be __________ . Turbine Building 1H21-P216 Panel; on the turning gear Turbine Building 1H21-P216 Panel; windmilling Turbine Building RFPT 1A area; on the turning gear Turbine Building RFPT 1A area; windmilling A.B.C.D.Wednesday, January 21, 2015 2:26:50 PM 7 6 ILT-09 NRC EXAMDescription:

31RS-OPS-001-1, Shutdown From Outside Control Room, states (4.13) IF two FeedwaterPumps are in operation, THEN dispatch an operator to manually trip one pump. A local RFPTtrip is accomplished using the Master Trip/Emergency Trip pushbuttons locally at the RFPT,which consists of two pushbuttons. Both RFPT trip pushbuttons must be simultaneously pressedto trip the RFPT. Once RFPT 1A is tripped, as long as the Condensate System remains inservice, RFPT 1A will lower in speed to approximately 100 rpm and be windmilling. If theCondensate System is not in service or the minimum flow valve does not open, then RFPT 1Awill lower in speed and go on Turning gear operation. K/A JUSTIFICATION:This question satisfies the K/A statement by requiring the applicant to have knowledge of localoperator actions to trip RFPT 1A during a shutdown from outside the Main Control Room (emergency) and the resultant effects to RFPT 1A (windmill or Turning Gear) operation. The "A" distractor is plausible since there are Feedwater controls at the 2H21-P216 to includethe FW isolation valves (N21-F006 A/B) and FW Heaters. If the applicant remembers that thereare FW controls at the H21-P216 then they also think that you can trip the RFP there also. The second part is plausible if the applicant thinks that since control of the plant has been establishedfrom the Remote Shutdown Panel, that the normal function of the RFPTs will not function anddoes not consider windmilling operation, therefore the RFPT 1A will coast down to zero (0) rpm and then automatically go on the Turning Gear. This is also plausible if the isolation valves wereclosed to the RFP then the RFP would not windmill but instead coast down to zero rpm and thenautomatically go on the Turning Gear.The "B" distractor is plausible since there are Feedwater controls at the 2H21-P216 to includethe FW isolation valves (N21-F006 A/B) and FW Heaters. If the applicant remembers that there are FW controls at the H21-P216 then they also think that you can trip the RFP there also. Thesecond part is plausible since it is correct. The "C" distractor is plausible since the first part is correct. The second part is plausible if theapplicant thinks that since control of the plant has been established from the Remote ShutdownPanel, that the normal function of the RFPTs will not function and does not consider windmilling operation, therefore the RFPT 1A will coast down to zero (0) rpm and thenautomatically go on the Turning Gear. This is also plausible if the isolation valves were closed tothe RFP then the RFP would not windmill but instead coast down to zero rpm and then automatically go on the Turning Gear.A. Incorrect - See description above. B. Incorrect - See description above. C. Incorrect - See description above. D. Correct - See description above. Wednesday, January 21, 2015 2:26:50 PM 7 7 ILT-09 NRC EXAM

References:

NONEK/A:259001 Reactor Feedwater SystemG2.4.35 Knowledge of local auxiliary operator tasks during an emergency and theresultant operational effects. (CFR: 41.10 / 43.5 / 45.13) . . . . . . . . . 3.8 4.0LESSON PLAN/OBJECTIVE:N21-CNDFW-LP-00201, Condensate And Feedwater System, Ver. 9.1, EO 002.006.A.03References used to develop this question:31RS-OPS-001-1, Shutdown From Outside Control Room, Ver. 5.24 34SO-N21-007-1, Condensate And Feedwater System, Ver. 49.2 Wednesday, January 21, 2015 2:26:50 PM 7 8 ILT-09 NRC EXAM 2 6.259002A3.04 001/04404B11/002.021.A.03/MOD/SYS-B/BOTH/259002A3.04/2/1/H/2/JSC/ELJUnit 2 is operating at 100% RTP with the following RWL indications: o2C32-R606A, GEMAC, indication: 37.0 incheso2C32-R606B, GEMAC, indication: 36.6 incheso2C32-R606C, GEMAC, indication: 36.9 inchesSubsequently, a leak occurs on the 2C32-R606A instrument REFERENCE leg which results in a 3 inch/minute change in RWL. Based on the above conditions and with NO operator actions,INITIALLY, the indication on RWL instrument 2C32-R606B will __________ .INITIALLY, Feedwater flow will __________ .LOWER;LOWERLOWER;INCREASEINCREASE;LOWERINCREASE;INCREASEA.B.C.D.Wednesday, January 21, 2015 2:26:50 PM 7 9 ILT-09 NRC EXAMDescription:This question tests the applicants understanding of the INITIAL response to a reference leg breakassociated with R606A (R606C uses this same reference leg), how it affects the RFPT speed and how the instruments that come off a different reference leg will respond.A leak in the reference leg causes 2C32-R606A & C to drift UP. Since one of these instrumentsis the Median signal (the level indication that is in the middle as determined by the SCMSmodule (C32-K648)), the RWLC sistem senses that RWL is high (above the setpoint of theMaster RFP controller). This causes an initial response of a RFP speed reduction, which causesactual RWL to decrease. Since the "R606B" is working normally, it responds to actual level conditions and its indicationbegins to decrease. K/A JUSTIFICATION:This question satisfies the K/A statement by asking the applicant's understanding of the INITIALresponse to a reference leg break associated with R606A (R606C uses this same reference leg),

how it affects the Feedwater flow and how the instruments that come off a different reference legwill respond.The "B" distractor is plausible since the first part is correct and the second if the applicantconfuses which instruments will be affected and which way they will fail. If so the applicant could think RWL is going down and think feedwater flow will be increasing. The "C" distractor is plausible if the applicant remembers two R606s will be affected and thinksR606B is the second instrument. If so, R606B would then be increasing along with R606A. Thesecond part is correct. The "D" distractor is plausible if the applicant remembers two R606s will be affected and thinksR606B is the second instrument. If so, R606B would then be increasing along with R606A. Thesecond is plausible if the applicant confuses which instruments will be affected and which way they will fail. If so the applicant could think RWL is going down and think Feedwater flow willbe increasing. A. Correct - See description above. B. Incorrect - See description above. C. Incorrect - See description above. D. Incorrect - See description above.

References:

NONEWednesday, January 21, 2015 2:26:50 PM 8 0 ILT-09 NRC EXAMK/A:259002 Reactor Water Level Control SystemA3. Ability to monitor automatic operations of the REACTOR WATER LEVELCONTROL SYSTEM including: (CFR: 41.7 / 45.7)A3.04 Changes in reactor feedwater flow . . . . . . . . . . . . 3.2 3.2LESSON PLAN/OBJECTIVE:B11-RXINS-LP-04404, Reactor Vessel Instrumentation, Ver 7.0, EO 200.002.A.014C32-RWLC-LP-00202, Reactor Water Level Control, Ver 6.1, EO 002.021.A.03 References used to develop this question:34SO-N21-007-2, Condensate and Feedwater System, Ver 52.1Modified from HLT Database Q#259002K3.07 001. This question used on HLT 6 exam #26. Original QuestionUnit 2 is operating at 100% RTP with the following RWL indications:o2C32-R606A, GEMAC, indication: +37.0"o2C32-R606B, GEMAC, indication: +36.6"o2C32-R606C, GEMAC, indication: +36.9"Subsequently, the REFERENCE leg for RWL intrument 2C32-R606A developes a significantleak.With NO operator actions, which ONE of the choices below describes the INITIAL response ofBOTH the RWL indicator 2C32-R606B AND the speed of the RPFTs to this reference leg leak?INITIALLY, the indication on RWL instrument 2C32-R606B will ___________ .INITIALLY, the RFPTs speed will ___________.

A. DECREASE DECREASEB. DECREASE INCREASEC. INCREASE DECREASED. INCREASE INCREASEWednesday, January 21, 2015 2:26:50 PM 8 1 ILT-09 NRC EXAM 2 7.261000A4.02 001/03001T46/030.001.A.01/MOD/SYS-I/BOTH/261000A4.02/2/1/H/2/ARB/ELJUnit 2 is operating at 100% RTP. At 10:00, Secondary Containment receives an isolation signal. At 10:01, the NPO secures SBGT 2A by placing the SBGT 2A fan control switch to the OFF p osition and then places the switch to the STBY position. (ONLY switch manipulated)At 10:02, the following Unit 2 SPDS Diagnostic Screen is observed:Based on the SPDS Diagnostic Screen at 10:02 , The Unit 2 SBGT system __________ operated as designed. If SPDS becomes unavailable, Unit 2 SBGT flow can be monitored on Panels 2H11-P657 and __________ . has; 2H11-P654 has;2H11-P700 has NOT;2H11-P654 has NOT;2H11-P700 A.B.C.D.Description:The parameter that initiates a Secondary Containment isolation signal will also provide a signalfor SBGT to initiate. Upon automatic initiation of Unit 2 SBGT, the following occurs perWednesday, January 21, 2015 2:26:50 PM 8 3 ILT-09 NRC EXAM 34SO-T46-001-2:7.2.1.1 Upon AUTOMATIC initiation, confirm the following actions: 7.2.1.1.1 2T46-D001A and 2T46-D001B, SBGT A and B Fan/Filters, START. 7.2.1.1.2 SBGT A and B HTR is ON. 7.2.1.1.3 2T46-F001A and 2T46-F001B, SBGT A and B Fltr Inlets From Rx Bldg, OPEN AND 2T46-F003A and 2T46-F003B, SBGT A AND B Fltr Inlets From Refuel Flr, OPEN. 7.2.1.1.4 2T46-F002A and 2T46-F002B, SBGT A AND B Fltr Disch dampers, OPEN.7.2.1.2 Standby Gas Treatment System Flow increases to 3.0-4.0 KCFM, as indicated on 2T41-R618 and 2U41-R600, SBGT A and B Flow To Main Stack.Shutting down the Unit 2 SBGT as follows per 34SO-T46-001-2:7.3.1 Shutdown 2T46-D001A, SBGT A Filter Train 7.3.1.1 Confirm annunciator 657-019, SBGT AUTO SIGNAL PRESENT, is RESET.

7.3.1.2 Confirm OR place 2T46-D001A, SBGT A Fan/Filter, control switch in the AUTO position. 7.3.1.3 Depress SBGT Fltr 2T46-D001A Fan/Htr Auto-Start Reset pushbutton. 7.3.1.4 Confirm SBGT A (Green) HTR OFF Light ILLUMINATED AND (Red) HTR On Light EXTINGUISHED. 7.3.1.5 Confirm Standby Gas Treatment Flow decreases to 0 KCFM as indicated on 2T41-R618 / 2U41-R600, SBGT A Flow to Main Stack. 7.3.1.6 Confirm 2T46-F002A, SBGT A Fltr Disch damper, CLOSES. 7.3.1.7 Confirm annunciator 2H11-P657-093, "SBGT FLTR A HI-HI TEMP TRIP OR FAN/HTR S/D" is NOT in the alarm condition. 7.3.1.8 Confirm closed OR close 2T46-F003A, SBGT A Fltr Inlet From Refuel Flr. 7.3.1.9 Confirm closed OR close 2T46-F001A, SBGT A Fltr Inlet From Rx Bldg. 7.3.1.10 Place the SBGT System A in STANDBY in accordance with the 'Standby - Ready for Automatic Start', subsection, of this procedure.Based on the SPDS graphic, the SBGT System is NOT operating properly since placing the only placing the SBGT 2A fan control switch to the OFF position will not cause F001A or F003A toautomatically close. Normal SBGT flow is 3000 to 4000 SCFM and is monitored on SPDS and Panel 1H11-P657.K/A JUSTIFICATION:This question satisfies the K/A statement by requiring the applicant to monitor the position of 2T46-F001A, SBGT A Filter Inlet (suction) valve, and know if it operated properly.The "A" distractor is plausible since this is a Unit difference. 1T46-F001A/B will automaticallyclose on Unit 1 when shutting down the respective Filter train. The 2T46-F001A/B and 2T46-F003A/B are the equivalent to the 1T46-F032A/B and 1T46-F040A/B and these valvesoperate the same between units. The second part is plausible since it is correct.The "B" distractor is plausible since this is a Unit difference. 1T46-F001A/B will automaticallyclose on Unit 1 when shutting down the respective Filter train. The 2T46-F001A/B and 2T46-F003A/B are the equivalent to the 1T46-F032A/B and 1T46-F040A/B and these valves operate the same between units. The second part is plausible if the applicant remembers that the2H11-P700 panel is where SBGT dP is indicated and thinks that this is where SBGT flow isWednesday, January 21, 2015 2:26:50 PM 8 4 ILT-09 NRC EXAMindicated. The "D" distractor is plausible since the first part is correct. The second part is plausible if theapplicant remembers that the 2H11-P700 panel is where SBGT dP is indicated and thinks that this is where SBGT flow is indicated. A. Incorrect - See description above. B. Incorrect - See description above. C. Correct - See description above. D. Incorrect - See description above.

References:

NONEK/A:261000 Standby Gas Treatment SystemA4. Ability to manually operate and/or monitor in the control room: (CFR: 41.7 / 45.5 to 45.8)A4.02 Suction valves . . . . . . . . . . . . . . . . . . . . 3.1 3.1LESSON PLAN/OBJECTIVE:T46-SBGT-LP-03001, Standby Gas Treatment System, Ver. 6.0, EO 030.001.A.01, References used to develop this question:34SO-T46-001-1, Standby Gas Treatment System, Ver. 21.0 34SO-T46-001-2, Standby Gas Treatment System, Ver. 14.14Wednesday, January 21, 2015 2:26:50 PM 8 5 ILT-09 NRC EXAM 2 8.262001A1.04 001/02702R22/027.010.A.02/MOD/P-NORM/BOTH/262001A1.04/2/1/H/3/JSC/ELJUnit 2 is at 60% RTP with an operator transferring 4160 VAC Bus 2A to its Alternate supply. The following conditions currently exist:

o Voltages are matchedoSync switch for the 4160 VAC 2A Alternate breaker is in the ON positionoSync light is at its dimmest (12 O'Clock position and steady)oStation SVC Interlock Cutout switch for ACB 135434-135454 is in the NORMAL (UP)positionSubsequently, the operator places the control switch for ACB 135454 (Alternate supply breaker)in the close position and IMMEDIATELY releases the switch.Based on the above conditions, Ten (10) seconds after the operator releases the control switch, the ampere indication will be as shown on __________ . Figure 1 Figure 2 Figure 3 Figure 4 Figure 1Figure 2Figure 3Figure 4A.B.C.D.Wednesday, January 21, 2015 2:26:50 PM 8 6 ILT-09 NRC EXAMDescription:The normal and alternate supply breakers are interlocked to prevent paralleling the 4160 VAC busses. Interlock cutout switches are provided to bypass this interlock between the normal andalternate supply breakers on 4160 VAC busses 2A, 2B, 2C, & 2D for a period of time whenswapping the power supplies is desired. This allows the busses to be paralleled only during thetime the house loads are being swapped from the startup source to the normal generator on lineoperation. Use of these switches is also allowed when shutting down to swap the house loadsover to the startup source. If both the alternate and supply breakers are closed at the same timewith the Interlock Service cutout switch in the NORMAL (UP) position, both breakers will tripopen. Immediately is used to ensure that the Interlock service cutout switch works properly. Ifthe alternate supply breaker is held in the close position for a few seconds, the alternate breakerwill remain closed.K/A JUSTIFICATION:This question satisfies the K/A statement by requiring the applicant to know the expectedresponse concerning load currents to the electrical plant based on breaker interlocks.The "A" distractor is plausible if the sync switch is left in the OFF position. When the operatorplaces the control switch for the alternate supply to close, it will not attempt to close the alternatesupply breaker.The "B" distractor is plausible if the applicant thinks that the station service buses work the sameas the emergency buses do. On the emergency buses, once you place the alternate supply breakerto close, the normal supply breaker will automatically open. The "C" distractor is plausible under heavy load with both the alternate and normal supplybreakers on the station service bus closed at the same time. A note in the 34SO-R22-001-2 states that "the current readings may increase on both the normal AND alternate supplies, WHEN boththe normal AND alternate supply breakers are closed"per engineering evaluations. This is onlyplausible when both breakers are still closed. A. Incorrect - See description above. B. Incorrect - See description above. C. Incorrect - See description above. D. Correct - See description above.

References:

NONEK/A:262001 A.C. Electrical DistributionWednesday, January 21, 2015 2:26:50 PM 8 7 ILT-09 NRC EXAMA1. Ability to predict and/or monitor changes in parameters associated with operating theA.C. ELECTRICAL DISTRIBUTION controls including:

(CFR: 41.5 / 45.5)A1.04 Load currents . . . . . . . . . . . . . . . . . . . . . . . . . . . 2.7 2.9LESSON PLAN/OBJECTIVE:R22-ELECT-LP-02702, 4160 VAC, Ver 6.1, EO 027.010.A.02References used to develop this question:34SO-R22-001-2, 4160 VAC System, Ver 21.0 Modified from HLT Database Q#LT-027010-004Original QuestionOn Unit 2, an operator has been ordered to transfer 4160 VAC Bus 2A from its Normal to itsStartup Supply. The following switch positions for the 4160 VAC Bus 2A Startup Supply feeder breaker exist: Switch Position Interlock Cutout Switch NORMAL (up) Sync Switch ON Breaker Control Switch mid-positionWhich ONE of the choices below completes the following statement? If the operator positions the 4160 VAC Bus 2A Alternate Supply Breaker Control Switch tothe CLOSE position and immediately releases the switch, the 4160 VAC Bus 2A:Normal Breaker will be ___________ , Startup Breaker will be ___________ .A. closed; closed B. closed; open C. open; closed D. open; Wednesday, January 21, 2015 2:26:50 PM 8 8 ILT-09 NRC EXAM 2 9.262002K4.01 001/02705R25/200.020.A.05/NEW/SYS-B/BOTH/262002K4.01/2/1/F/2/ARB/ELJUnit 2 is operating at 100% RTP when the following indications are received on Panel 2H11-P651; Based on the above indications and, Fifteen (15) seconds later, the Vital AC Bus is receiving power from its __________ Power Supply. The Alternate AC supply for the Vital AC Bus is __________ . Alternate AC; 600V Bus 2C Alternate AC; 600V Bus 2D Backup DC; 600V Bus 2C Backup DC; 600V Bus 2D A.B.C.D.Wednesday, January 21, 2015 2:26:50 PM 9 0 ILT-09 NRC EXAMDescription:The graphic provided indicates that the normal power supply from the battery charger via 600VAC Bus 2D is no longer available to provide power to the Vital AC bus. Vital AC willthen transfer to its first alernate power supply, Vital AC Batteries. If battery voltage drops below208 VDC, the alternate power supply from 600 VAC Essential Bus "C" will automatically pickup the Vital AC Bus. Normal supply is 600V 2D via the battery charger, then to the Batteries until battery voltagedrops below 208 VDC then transfers to Alternate (600V 2C). K/A JUSTIFICATION:This question satisfies the K/A statement by requiring the applicant to determine the powersupply to Vital AC which based on plant conditions has transferred power from Normal (preferred) to batteries (First Alternate). The "A" distractor is plausible if the applicant remembers that 600V 2D is a power supply toVital AC but does not remember the correct bus transfer sequence; incorrect-[Normal (600V 2D)to Alternate (600V 2C) to Batteries] versus the correct-[Normal (600V 2D) to Batteries to Alternate (600V 2C).] The second part is plausible since it is correct. The "B" distractor is plausible if the applicant remembers that 600V 2D is a power supply toVital AC but does not remember the correct bus transfer sequence; incorrect-[Normal (600V 2D)to Alternate (600V 2C) to Batteries] versus the correct-[Normal (600V 2D) to Batteries toAlternate (600V 2C).] The second part is plausible if the applicant remembers that 600V 2D is a power supply to Vital AC and would be correct if asking for the Vital AC Normal power source. The "D" distractor is plausible since the first part is correct. The second part is plausible if theapplicant remembers that 600V 2D is a power supply to Vital AC and would be correct if askingfor the Vital AC Normal power source. A. Incorrect - See description above. B. Incorrect - See description above. C. Correct - See description above. D. Incorrect - See description above. Wednesday, January 21, 2015 2:26:51 PM 9 1 ILT-09 NRC EXAM

References:

NONEK/A:262002 Uninterruptable Power Supply (A.C./D.C.)K4. Knowledge of UNINTERRUPTABLE POWER SUPPLY (A.C./D.C.) design feature(s)and/or interlocks which provide for the following: (CFR: 41.7)K4.01 Transfer from preferred power to alternate power supplies . . . . . . . . 3.1 3.4LESSON PLAN/OBJECTIVE:R25-ELECT-LP-02705, Vital AC Electrical System, Ver. 3.0, EO 200.020.A.05References used to develop this question:34SO-R25-002-2, 120/240 Volt Vital AC System, Ver. 5.2 34AR-651-133-2, 240V Vital AC Batt Volts Low, Ver. 1.0 34AR-651-134-2, Vital AC Sys Trouble, Ver. 1.2 Wednesday, January 21, 2015 2:26:51 PM 9 2 ILT-09 NRC EXAM 3 0.263000G2.1.19 001/05601SPDS/056.002.C.03/MOD/SYS-I/BOTH/263000G2.1.19/2/1/H/2/JSC/ELJUnit 2 was operating at 100% RTP when an event occurred resulting in an electrical powersupply de-energizing. While monitoring SPDS, the following indications exist on the PCIS Diagnostic Groups 1, 3, 4, & 5 Status screen; Based on the above SPDS indications, A loss of __________ has occurred. If HPCI receives a valid initiation signal, HPCI __________ start and inject into the RPV. 600V Rx. Bldg. MCC 2B, 2R24-S011A; will 600V Rx. Bldg. MCC 2B, 2R24-S011A; will NOT 250 VDC MCC 2B, 2R24-S022; will 250 VDC MCC 2B, 2R24-S022; will NOT A.B.C.D.Wednesday, January 21, 2015 2:26:51 PM 9 3 ILT-09 NRC EXAMDescription:Per 34SO-X75-002-2, Operation of SPDS Equipment, Attachment 4:

3.For valves:a.Green - closed.b.Red - open.c.Red and Green - in transit.d. Yellow - valve position data not available.The SPDS indication for the PCIVs will turn yellow when the valve position is not available (i.e.power loss). 2E41-F003 has lost power. Its power supply is 2R24-S022. The loss of 2R24-S022also removes power from the HPCI aux oil pump therefore the HPCI system will not start on aninitiation signal.K/A JUSTIFICATION:This question satisfies the K/A statement by requiring the applicant to know how the SPDScomputer monitors the status of the PCIVs and how the indications will change based on loss of power.The "A" distractor is plausible since this is the power supply for the inboard isolation valve forHPCI. The second part is plausible since if HPCI was already operating, it would continue tooperate (aux oil pump shuts off once sufficient pressure is obtained based on a shaft driven oil pump).The "B" distractor is plausible since this is the power supply for the inboard isolation valve forHPCI. The second part is plausible since it is correct. The "C" distractor is plausible since the first part is correct. The second part is plausible since ifHPCI was already operating, it would continue to operate (aux oil pump shuts off once sufficientpressure is obtained based on a shaft driven oil pump).A. Incorrect - See description above. B. Incorrect - See description above. C. Incorrect - See description above. D. Correct - See description above. Wednesday, January 21, 2015 2:26:51 PM 9 4 ILT-09 NRC EXAM

References:

NONEK/A:263000 D.C. Electrical DistributionG2.1.19 Ability to use plant computers to evaluate system or component status. (CFR: 41.10 / 45.12) . . . . . . . . . . . . . . . . . . 3.9 3.8LESSON PLAN/OBJECTIVE:X75-SPDS-LP-05601, Safety Parameter Display System, Ver 6.1, EO 056.002.C.03References used to develop this question:34SO-X75-002-2, Operation of SPDS Equipment, Ver 4.0 Modified from HLT Database Q#206000-010 Original QuestionUnit 2 HPCI is being operated in the CST to CST mode for testing.oA loss of Station Service 250VDC "2B" 2R24-S022 occursoOne (1) minute later, Drywell pressure increases to 2.1 psigThe valve position indicating lights on 2H11-P601 for HPCI Steam Supply Isolation Valve,2E41-F001, will be __________ .HPCI will __________ .A. lit; remain in CST to CST mode B. lit; inject to the core C. extinguished; remain in CST to CST mode D. extinguished; inject to the core Wednesday, January 21, 2015 2:26:51 PM 9 5 ILT-09 NRC EXAM 3 1.263000K5.01 001/02704R42/027.044.A.03/MOD/SYS-B/BOTH/263000K5.01/2/1/F/3/ARB/ELJUnit 2 is operating at 100% RTP with the "2A" and "2B" 125/250VDC Station Service Batteries on "Equalize" charge.Subsequently,oAll Control Building Chillers and fans trip and can NOT be restoredoControl Building temperatures start increasingWith the "2A" and "2B" 125/250VDC Station Service Batteries on "Equalize" charge, the125/250VDC Station Service Battery Chargers output voltage will be __________ the battery voltage. With Control Building temperatures increasing and NO operator actions, the 125/250VDCStation Service Battery __________ . equal to; Room Hydrogen concentrations will rise in each of the battery rooms equal to; Chargers will trip when their high temperature trip setpoint is reached greater than; Room Hydrogen concentrations will rise in each of the battery rooms greater than; Chargers will trip when their high temperature trip setpoint is reached A.B.C.D.Wednesday, January 21, 2015 2:26:51 PM 9 6 ILT-09 NRC EXAMDescription:Battery chargers can be supplying a float or equalizing charge for the batteries during their normal lineup. If a battery is on a float charge, the battery chargers output voltage is equal to thebattery voltage and the battery is "floating" on the system. The battery charger is supplyingenough output to equal the load on the bus with current input and output from the battery equal. If a battery is on an equalizing charge, the battery charger output voltage is elevated slightly overbattery voltage, with voltage input to the battery greater than the output. In this case, the batteryacts as a load on the battery charger. An equalizing charge brings the battery up to a fully charged condition. A ventilation system in each battery room prevents a buildup of combustible gases and helpsensure operation during emergency conditions. With a loss of CR ventilation and batterychargers in service, then the Emergency Exhaust Fans must be started to prevent hydrogenbuildup. The battery chargers have an alarm on high temperatures but have internal fans thatkeep them cool. Also, the battery chargers do NOT have a high temperature trip. K/A JUSTIFICATION:This question satisfies the K/A statement by requiring the applicant to determine that during thetime the batteries are being charged (equalized) and with the loss of normal ventilation, thestation service battery rooms will be experiencing higher than normal concentrations ofhydrogen. The "A" distractor is plausible if the applicant thinks about a float charge instead of an equalizecharge on the batteries. In this case the applicant will think that since the chargers are onequalize that the voltages must be equal. The second part is correct since it is correct. The "B" distractor is plausible if the applicant thinks about a float charge instead of an equalizecharge on the batteries. In this case the applicant will think that since the chargers are onequalize that the voltages must be equal. The second part is plausible if the applicant remembers the battery chargers have a high temperature alarm but forgets that they do not have a hightemperature trip and thinks the chargers will trip. The "D" distractor is plausible since the first part is correct. The second part is plausible if theapplicant remembers the battery chargers have a high temperature alarm but forgets that they donot have a high temperature trip and thinks the chargers will trip. A. Incorrect - See description above. B. Incorrect - See description above. C. Correct - See description above. D. Incorrect - See description above.

References:

Wednesday, January 21, 2015 2:26:51 PM 9 7 ILT-09 NRC EXAMNONEK/A:263000 D.C. Electrical DistributionK5. Knowledge of the operational implications of the following concepts as they apply toD.C. ELECTRICAL DISTRIBUTION : (CFR: 41.5 / 45.3)K5.01 Hydrogen generation during battery charging. . . . . . . . . . . . 2.6 2.9LESSON PLAN/OBJECTIVE:R42-ELECT-LP-02704, DC Electrical Distribution, Ver. 7.1, EO 027.044.A.03References used to develop this question:34SO-R42-001-2, 125 VDC & 125/250 VDC System, Ver. 7.15 34AR-654-040-2, Battery Room Exh Fan C015 Flow Low, Ver. 3.0 34AR-657-028-2, Battery Room Exh Fan C014 Flow Low, Ver. 2.0Modified from HLT 2012-301 NRC Exam Q#32 Original QuestionUnit 2 is operating at 100% power, with the "2A" and "2B" 125/250VDC Station ServiceBatteries on equalize charge.oAll Control Building Chillers and fans trip and can NOT be restoredoControl Building temperatures start increasingWhich ONE of the following completes both statements concerning the effect of charging the batteries with the above conditions present and any required actions to mitigate the consequencesof the event? With the above conditions, __________ . To mitigate the consequences of this event, the operator will enter __________ . A. Hydrogen concentration will rise in the battery rooms; 34AB-T41-001-2, "Loss Of ECCS, MCREC Or Area Ventilation Systems" and startEmergency Exhaust Fans 2Z41-C014 and 2Z41-C015 B. Hydrogen concentration will rise in the battery rooms; Wednesday, January 21, 2015 2:26:51 PM 9 8 ILT-09 NRC EXAM34SO-R42-001-2, "125 VDC AND 125/250 VDC SYSTEM", and secure all the batterychargersC. the Battery Chargers will trip when their high temperature trip setpoint is reached; 34AB-T41-001-2, "Loss Of ECCS, MCREC Or Area Ventilation Systems" and startEmergency Exhaust Fans 2Z41-C014 and 2Z41-C015 D. the Battery Chargers will trip when their high temperature trip setpoint is reached; 34SO-R42-001-2, "125 VDC AND 125/250 VDC SYSTEM", and secure all the batterychargers Wednesday, January 21, 2015 2:26:51 PM 9 9 ILT-09 NRC EXAM 3 2.264000A3.06 001/03301P41/033.003.A.04/MOD/SYS-B/BOTH/264000A3.06/2/1/F/3/JSC/ELJUnit 1 and Unit 2 are operating at 100% RTP when a TOTAL Loss Of Offsite Power (LOSP)occurs.The Diesel Gen 1B Keylock control switch is in the REMOTE UNIT 1 position. With the above conditions, The automatic start of the Standby Diesel Service Water Pump, 2P41-C002, can be monitoredfor operation at __________ . INITIALLY, the Standby Diesel Service Water Pump, 2P41-C002 will be powered via 4160V __________ . 2H11-P652 ONLY; 1F 2H11-P652 ONLY; 2F 2H11-P652 and 1H11-P652; 1F 2H11-P652 and 1H11-P652; 2F A.B.C.D.Description:The Standby Diesel Service Water pump can be monitored in four locations. These locations are1H11-P652, 2H11-P652, 1B EDG room, and locally at pump. 1H11-P652 and 2H11-P652 have a pressure gauge (vertical section) and a indicating light with control switch.The STANDBY DIESEL SERVICE WATER PUMP is located in the intake structure on thepump deck between the Unit 1 and Unit 2 RHR Service Water Strainers along the north wall.

The pump provides cooling water (normal source) to the "1B" Diesel Generator coolers and is acentrifugal pump rated at 700 gpm at 232' TDH. The pump is a powered from 600/208 MCC 1BESS Div B (1R24-S026). Either Unit can supply power to 1R24-S026 from 4160 VAC bus "1F" or "2F". The normal power supply to 1R24-S026 is from 4160 VAC 1F. 1R24-S026 will transferover to 4160 VAC 2F upon a loss of site power to the 4160 VAC 2F bus. Per Attachment 3 of 34AB-R43-001-2, on a complete Loss of Site Power, the Unit that willreceive the 1B EDG depends upon the switch position for the EDG in the 1B EDG room. TheSelect Switch is Normally aligned to Unit 1 per procedure.Wednesday, January 21, 2015 2:26:52 PM 100 ILT-09 NRC EXAMK/A JUSTIFICATION:This question satisfies the K/A statement by requiring the applicant to determine what thecooling supply to EDG 1B will be during automatic operation.The "A" distractor is plausible since it is partially correct. The standby diesel service water pumpcan be monitored at 1H11-P652 and 2H11-P652. The applicant may think that it can be onlymonitored on 2H11-P652 since the pump is labeled 2P41-C002. The second part is plausiblesince it is correct. The "B" distractor is plausible since it is partially correct. The standby diesel service water pumpcan be monitored at 1H11-P652 and 2H11-P652. The applicant may think that it can be onlymonitored on 2H11-P652 since the pump is labeled 2P41-C002. The second part is plausible ifUnit 2 were to receive a LOCA signal combined with the LOSP (since both have a LOSP). Alsowould be plausible if the Diesel Gen 1B Keylock control switch was positioned to the REMOTEUNIT 2 position.The "D" distractor is plausible since the first part is correct. The second part is plausible if Unit 2were to receive a LOCA signal combined with the LOSP (since both have a LOSP). Also wouldbe plausible if the Diesel Gen 1B Keylock control switch was positioned to the REMOTE UNIT 2 position.A. Incorrect - See description above. Wednesday, January 21, 2015 2:26:52 PM 101 ILT-09 NRC EXAMB. Incorrect - See description above. C. Correct - See description above. D. Incorrect - See description above.

References:

NONEK/A:264000 Emergency Generators (Diesel/Jet)A3. Ability to monitor automatic operations of the EMERGENCY GENERATORS(DIESEL/JET) including: (CFR: 41.7 / 45.7) A3.06 Cooling water system operation . . . . . . . . . 3.1 3.2LESSON PLAN/OBJECTIVE:P41-PSW-LP-03301, Plant Service Water System, Ver 10.3, EO 033.003.A.02 and033.003.A.04 References used to develop this question:34SO-R43-001-1, Diesel Generator Standby AC System, Ver 27.034AB-R43-001-2, Diesel Generator Recovery, Ver 3.4Modified from HLT Database Q#264000K1.04-001 which was used on 2012-301 NRC Exam Q#31 Original QuestionUnit 1 and Unit 2 are experiencing a TOTAL loss of Off-Site power. The Diesel Gen 1B Keylock control switch is in the REMOTE UNIT 1 position. Subsequently, Unit 2 receives a LOCA signal. Which ONE of the following identifies the Emergency Bus being powered from, and the coolingwater supply to, 1B Emergency Diesel Generator (EDG)? 1B EDG is powering 4160 V __________ Emergency Bus1B EDG is receiving cooling water from __________ . Wednesday, January 21, 2015 2:26:52 PM 102 ILT-09 NRC EXAMA. 1F; 2P41-C002, Standby Diesel Service Water Pump B. 1F; Unit 1 Division 1 Plant Service Water C. 2F; 2P41-C002, Standby Diesel Service Water Pump D. 2F; Unit 1 Division 1 Plant Service Water Wednesday, January 21, 2015 2:26:52 PM 103 ILT-09 NRC EXAM 3 3.271000K4.01 001/03101N62/031.001.A.13/MOD/SYS-B/BOTH/271000K4.01/2/2/F/3/ARB/ELJUnit 2 is starting up from an outage with the Off-Gas System being purged. The following Off-Gas System Loop Seal Isolation valves are in the OPEN position;o2N62-F085, Holdup Line Draino2N62-F030A, Cndsr/Sep A Drain o2N62-F030B, Cndsr/Sep B DrainThe Unit 2 Off-Gas System is purged with Service Air for a minimum of one (1) hour priorto placing the system in operation to __________ . While purging the Off-Gas System, if the Off-Gas System pressure increases to 8 psig, IAW 34SO-N61-001-2, Main Condenser Vacuum System And Closeout, the Off-Gas SystemLoop Seals Isolation valves are required to be _________ . dilute any hydrogen left in the system from previous use; manually isolated dilute any hydrogen left in the system from previous use; confirmed to have automatically isolated remove any radioactive Iodine deposited on the charcoal adsorbers; manually isolated remove any radioactive Iodine deposited on the charcoal adsorbers; confirmed to have automatically isolated A.B.C.D.Wednesday, January 21, 2015 2:26:52 PM 104 ILT-09 NRC EXAMDescription:The Off Gas System is purged for one hour with air prior to use to dilute any Hydrogen leftfrom previous use. The purge also provides flow through the recombiner while heating to 350°Fto ensure that it is free of moisture. IAW 34SO-N61-001-2, Main Condenser Vacuum System And Closeout, if pressure increases togreater than 6 psig on 2N62-R600, O/G To Prehtr, THEN close the following Loop SealIsolation Valves: 2N62-F085, Holdup Line Drain, 2N62-F030A, Cndsr/Sep A Drain,2N62-F030B, Cndsr/Sep B Drain. These valves are manually operated on Panel 2N62-P600.K/A JUSTIFICATION:This question satisfies the K/A statement by requiring the applicant to determine the reason(design) to purge with service air the hydrogen from the Off Gas System for the dilution ofhydrogen gas. Also the design feature to isolate the loop seals if Off Gas pressure exceeds 6 psigpreserving the loop seals. The "B" distractor is plausible since the first part is correct. The second part is plausible if theapplicant remembers that these valves will automatically close on certain Off Gas Systemparameters and thinks the loop seals will automatically isolate on >6 psig. The "C" distractor is plausible if the applicant remembers that radioactive Iodine is depositedonto the charcoal adsorbers and exchanged with nonradioactive Iodine and thinks purging withair will remove it. The radioactive iodine on the adsorbers will not be removed by purging with air and will normally be removed when the charcoal in the adsorber is replaced. The second partis plausible since it is correct. The "D" distractor is plausible if the applicant remembers that radioactive Iodine is depositedonto the charcoal adsorbers and exchanged with nonradioactive Iodine and thinks purging withair will remove it. The radioactive iodine on the adsorbers will not be removed by purging with air and will normally be removed when the charcoal in the adsorber is replaced. The second partis plausible if the applicant remembers that these valves will automatically close on certain OffGas System parameters and thinks the loop seals will automatically isolate on >6 psig. A. Correct - See description above. B. Incorrect - See description above. C. Incorrect - See description above. D. Incorrect - See description above. Wednesday, January 21, 2015 2:26:52 PM 105 ILT-09 NRC EXAM

References:

NONEK/A:271000 Offgas SystemK4. Knowledge of OFFGAS SYSTEM design feature(s) and/or interlocks which providefor the following: (CFR: 41.7)K4.01 Dilution of hydrogen gas concentration . . . . . . . . 2.9 3.3LESSON PLAN/OBJECTIVE:N62-OG-LP-03101, Off Gas System, Ver. 6.0, EO 031.001.A.13 and EO 031.001.A.14 References used to develop this question:34SO-N61-001-2, Main Condenser Vacuum System And Closeout, Ver. 21.6 34SO-N62-001-2, Off Gas System, Ver. 21.1Modified from HLT Database Q#LT-031001-011Original QuestionWhich ONE of the choices below describes the reason for air purging the Off-Gas system for onehour prior to placing the system in operation?A. To provide cooling air to the hydrogen recombiner.B. To dilute any hydrogen left in the system from previous use.C. To provide cooling air to the post treatment radiation monitors.D. To remove any radioactive Iodine deposited on the charcoal adsorbersWednesday, January 21, 2015 2:26:52 PM 106 ILT-09 NRC EXAM 3 4.272000A3.02 001/10007D11/200.030.A.05/BANK/SYS-B/BOTH/272000A3.02/2/2/F/2/JSC/ELJWhich ONE of the choices below is the set of conditions which will cause an automatic isolation of the Unit 2 Main Stack Isolation valve, 2N62-F057? Post Treatment Radiation Monitor channel "A" is __________ , ANDPost Treatment Radiation Monitor channel "B" is __________ .INOP; DOWNSCALE INOP; HIGH HIGH; DOWNSCALE HIGH; HIGH A.B.C.D.Wednesday, January 21, 2015 2:26:52 PM 107 ILT-09 NRC EXAMDescription:If a Hi-Hi-Hi, Inop or Downscale condition occurs on both Offgas Post Treatment Radiation detectors, then the following valves close: Offgas Stack Isolation Valve, 2N62-F057 Offgas Cooler Condenser/Moisture Separator Valves 2N62-F030A and B Offgas Holdup Line Drain, 2N62-F085 For a trip to occur and the Offgas system to isolate, both detectors must have any combination ofHi-Hi-Hi, Downscale, or Inop alarmsIf a Hi alarm is received on either detector, then the carbon bed bypass valve, 2N62-F043, closesand the carbon bed inlet valve, 2N62-F042, opens.K/A JUSTIFICATION:This question satisfies the K/A statement by requiring the applicant to know the automaticclosure of 2N62-F057, Offgas Stack Isolation Valve based upon Post Treatment Radiationmonitors.The "B" distractor is plausible since the first part is correct. The second part is plausible since aHI alarm will cause 2N62-F043 to close and 2N62-F042 to open.The "C" distractor is plausible since a HI alarm will cause 2N62-F043 to close and 2N62-F042to open. The second part is plausible since it is correct.The "D" distractor is plausible since a HI alarm will cause 2N62-F043 to close and 2N62-F042to open. The second part is plausible since a HI alarm will cause 2N62-F043 to close and 2N62-F042 to open.A. Correct - See description above. B. Incorrect - See description above. C. Incorrect - See description above. D. Incorrect - See description above. Wednesday, January 21, 2015 2:26:52 PM 108 ILT-09 NRC EXAM

References:

NONEK/A:272000 Radiation Monitoring SystemA3. Ability to monitor automatic operations of the RADIATION MONITORINGSYSTEM including: (CFR: 41.7 / 45.7)A3.02 Offgas system isolation indications . . . . . . . . . . 3.6 3.7LESSON PLAN/OBJECTIVE:D11-PRM-LP-10007, Process Radiation Monitors, Ver 5.0, EO 200.030.A.05References used to develop this question:34SO-N62-001-2, Off Gas System, Ver 21.1Bank question LT-200030-005 from HLT DatabaseWednesday, January 21, 2015 2:26:52 PM 109 ILT-09 NRC EXAM 3 5.288000K1.04 001/01303T41/037.004.A.01/BANK/SYS-B/BOTH/288000K1.04/2/2/F/3/ARB/ELJOn Unit 1 , Cooling water to the Safeguard Equipment Cooling (SEC) coolers is supplied by the __________ System. Cooling water to the Main Control Room Air Conditioning Unit Condensers is supplied bythe __________ System.Reactor Building Chilled Water; Plant Service Water Reactor Building Chilled Water; Control Building Chilled Water Plant Service Water; Plant Service Water Plant Service Water; Control Building Chilled Water A.B.C.D.Wednesday, January 21, 2015 2:26:52 PM 110 ILT-09 NRC EXAMDescription:Cooling Water Systems The Plant Service Water System provides cooling for the SEC Coolers on BOTH units providescooling water to the Main Control Room Air Conditioning Units (MCREC). K/A JUSTIFICATION:This question satisfies the K/A statement by requiring the applicant to know the cooling water(PSW) interface (physical connection) of the SEC coolers and the Main Control Room AirConditioning Units (MCREC). Both sets of coolers are part of the Plant Ventilation System. The "A" distractor is plausible since Reactor Building Chill Water supplies cooling water to theventilation coolers inside the Reactor Building where the SEC coolers are also located in theReactor Building. The second part is plausible since it is correct.The "B" distractor is plausible since Reactor Building Chill Water supplies cooling water to theventilation coolers inside the Reactor Building where the SEC coolers are also located in theReactor Building. The second part is plausible if the applicant remembers Control Building Chilled Water cools ventilation in the Control Building such as Vital AC rooms on both Units.Since the Main Control Room is located in the Control Building, the applicant may think thenControl Building Chill Water instead of PSW. The "D" distractor is plausible since the first part is correct. The second part is plausible if theapplicant remembers Control Building Chilled Water cools ventilation in the Control Building such as Vital AC rooms on both Units. Since the Main Control Room is located in the ControlBuilding, the applicant may think then Control Building Chill Water instead of PSW. A. Incorrect - See description above. B. Incorrect - See description above. C. Correct - See description above. D. Incorrect - See description above. Wednesday, January 21, 2015 2:26:52 PM 111 ILT-09 NRC EXAM

References:

NONEK/A:288000 Plant Ventilation SystemsK1. Knowledge of the physical connections and/or cause-effect relationships betweenPLANT VENTILATION SYSTEMS and the following:

(CFR: 41.2 to 41.9 / 45.7 to 45.8)K1.04 Applicable component cooling water system: Plant-Specific . . . . . 2.6 2.6LESSON PLAN/OBJECTIVE:T41-SC HVAC-LP-01303, Secondary Containment HVAC Systems, Ver. 5.1, EO 037.004.A.01References used to develop this question:Wednesday, January 21, 2015 2:26:52 PM 112 ILT-09 NRC EXAM 3 6.290001A4.02 001/00301G31/003.002.A.10/NEW/P-AB/BOTH/290001A4.02/2/2/F/2/JSC/ELJUnit 1 is operating at 100% RTP with RWCU Pump 1A in service. NPOs in the Main Control Room, monitoring the RWCU System, observe the following:RWCU HX RWCU Pump 1A Time Room Temp Room Temp 10:55 125°F 125°F 11:00 145°F 145°F 11:05 168°F 168°F Based on the above conditions, When the RWCU Room temperatures reached their isolation setpoint, the RWCU Systemshould have automatically isolated __________ .The EARLIEST listed time that the NPO is procedurally required to manually isolate theRWCU System is __________ .immediately; 11:00 immediately; 11:05 after a time delay; 11:00 after a time delay; 11:05 A.B.C.D.Wednesday, January 21, 2015 2:26:52 PM 113 ILT-09 NRC EXAMDescription:

1G31-F001, RWCU Inboard Isolation, AND 1G31-F004, RWCU Outboard Isolation, CLOSE on the following signals: Low Reactor water level, -35 inches. High differential flow, 56 gpm for 42.5 sec. High RWCU area ventilation differential temperature. RWCU Pump Room 60°F RWCU Hx Room 45°F RWCU Phase Separator Room 40°F High RWCU area ambient temperature. RWCU Pump Room 140°F (Annunciated at 130°F) RWCU Hx Room 140°F (Annunciated at 130°F) RWCU Phase Separator Room 140°F (Annunciated at 130°F)K/A JUSTIFICATION:This question satisfies the K/A statement by requiring the applicant to know the Group 5 isolation signals associated with room ventilation temperatures.The "B" distractor is plausible since the first part is correct. The second part is plausible if theapplicant thinks about the HPCI or RCIC room high temperature isolation (165°F) instead of the RWCU room temperature isolation (140°F). The "C" distractor is plausible if the applicant remembers the time delay associated with theHigh Differential Flow isolation (56 gpm for 42.5 seconds) instead of the High Ambient temperature isolation. The applicant also may think about the HPCI/RCIC high temperatureisolation signals (29 min/14 min). The second part is plausible since it is correct.The "D" distractor is plausible if the applicant confuses the time delay associated with the HighDifferential Flow isolation (56 gpm for 42.5 seconds) with that of the High Ambient temperatureisolation. The applicant also may think about the HPCI/RCIC high temperature isolation signals (29 min/14 min). The second part is plausible if the applicant thinks about the HPCI or RCICroom high temperature isolation (165°F) instead of the RWCU room temperature isolation(140°F). A. Correct - See description above. B. Incorrect - See description above. C. Incorrect - See description above. D. Incorrect - See description above. Wednesday, January 21, 2015 2:26:52 PM 114 ILT-09 NRC EXAM

References:

NONEK/A:290001 Secondary ContainmentA4. Ability to manually operate and/or monitor in the control room: (CFR: 41.7 / 45.5 to 45.8)A4.02 Reactor building area temperatures: Plant-Specific . . . . . . 3.3 3.4REPLACED THE BELOW K/A AFTER PHONE CONVERSATION WITH CHIEFEXAMINER PHIL CAPEHART ON 9/17/2014.A4. Ability to manually operate and/or monitor in the control room: (CFR: 41.7 / 45.5 to 45.8)A4.04 Auxiliary building area temperature: Plant-Specific . . . . . 2.6* 2.7LESSON PLAN/OBJECTIVE:G31-RWCU-LP-00301, Reactor Water Cleanup, Ver 5.2, EO 003.002.A.10References used to develop this question:34SO-G31-003-1, Reactor Water Cleanup System, Ver 42.534SO-E41-001-1, High Pressure Coolant Injection (HPCI) System, Ver 28.134SO-E51-001-1, Reactor Core Isolation Cooling, (RCIC) System, Ver 28.0Wednesday, January 21, 2015 2:26:52 PM 115 ILT-09 NRC EXAM 3 7.295001AA2.06 001/00401B31/200.037.A.02/MOD/SYS-I/BOTH/295001AA2.06/1/1/H/3/ARB/ELJ A Unit 2 startup is in progress IAW 34GO-OPS-001-2, Plant Startup. The following sequence of events occur: oBoth Recirc Pump speeds were raised from minimum speed to 30%oAfter the Recirc Pump speeds were raised, ASD 2A trippedWith the above current conditions, To manually calculate core flow rate, Loop "A" Jet Pump flow, 2B21-R611A, must be __________ Loop B Jet Pump Flow, 2B21-R611B. Core Flow Recorder, 2B21-R613, __________ be indicating accurate core flow. subtracted from; will NOT subtracted from; will added to;will NOT added to;will A.B.C.D.Wednesday, January 21, 2015 2:26:52 PM 116 ILT-09 NRC EXAMDescription:During single loop operation (sensed by the "ASD Not Running" status provided by the RecircNXG computer), when the speed of the running pump decreases below approximately 35%speed, positive flow through the idle pump loop due to natural circulation overcomes thenegative flow due to reverse flow. The total core flow summing circuitry will continue tosubtract this positive idle loop flow from the running loop flow and give a misleading LOW core flow indication. Total Core Flow can be calculated by adding the JET PUMP LOOP "A" ANDthe JET PUMP LOOP "B" flows. (Convert to % core flow by dividing total core flow by 77Mlbm/hr).K/A JUSTIFICATION:This question satisfies the K/A statement by requiring the applicant to determine if the CoreFlow Recorder 2B21-R613, (Nuclear Boiler Instrument) is accurate based on Loop A & B Jet Pump Flows and the speed of the operating Recirc pump. After the applicant makes thisdetermination, the correct method of determing accurate core flow using Loop Jet pump flows isestablished. The "A" distractor is plausible if the applicant remembers that Loop A & B flows are alwaysautomatically subtracted when one Recirc pump is in service to obtain an accurate core flow andwould be correct if the running Recirc pump speed was higher than 35%. The second part isplausible since it is correct.The "B" distractor is plausible if the applicant remembers that Loop A & B flows are alwaysautomatically subtracted when one Recirc pump is in service to obtain an accurate core flow and would be correct if the running Recirc pump speed was higher than 35%. The second part isplausible if the applicant does not consider < 35% running Recirc speed and the naturalcirculation effect on core flow indication. In this case the applicant will think the recorder is accurate. The "D" distractor is plausible since the first part is correct. The second part is plausible if theapplicant does not consider < 35% running Recirc speed and the natural circulation effect oncore flow indication. In this case the applicant will think the recorder is accurate.A. Incorrect - See description above. B. Incorrect - See description above. C. Correct - See description above. D. Incorrect - See description above.

References:

NONEK/A:Wednesday, January 21, 2015 2:26:52 PM 117 ILT-09 NRC EXAM295001 Partial or Complete Loss of Forced Core Flow CirculationAA2. Ability to determine and/or interpret the following as they apply to PARTIAL ORCOMPLETE LOSS OF FORCED CORE FLOW CIRCULATION : (CFR: 41.10 / 43.5 / 45.13)AA2.06 Nuclear boiler instrumentation . . . . . . . . . . . . . . . . . . . . . . . . 3.2 3.3LESSON PLAN/OBJECTIVE:B31-RRS-LP-00401, Reactor Recirculation System, Ver. 10.5, EO 200.037.A.02 References used to develop this question:34SO-B31-001-2, Reactor Recirc System, Ver. 44.6, P&L 5.1.5 34AR-602-127-2, Recirc Loop A Out Of Service, Ver. 3.0 34SV-SUV-023-2, Jet Pump & Recirc Flow Mismatch Operability, Ver. 7.15 Modified from HLT Database Q#295001AK1.01-002ORIGINAL QUESTIONA Unit 2 startup is in progress in accordance with 34GO-OPS-001-2, Plant Startup.The following sequence of events occur:oBoth Recirc Pump speeds were raised from minimum speed to 30%oAfter the recirc pump speeds were raised, the 2A ASD tripped.Five minutes after the ASD trip, the following control panel indications exist:oAnnunciator "RECIRC LOOP A OUT OF SERVICE" (602-127) in alarmoCore Flow Recorder 2B21-R613 7.2 Mlb/hroLoop A Jet Pump Flow 2B21-R611A 5.6 Mlb/hroLoop B Jet Pump Flow 2B21-R611B 12.8 Mlb/hrGiven these current conditions, which ONE of the choices below is correct?A.

Core Flow recorder indication is NOT correct."A" and "B" jet pump flows should be summed to obtain an accurate core flow rate.B. Core Flow recorder indication is NOT correct."A" jet pump flow must be added to the recorder flow to obtain an actual core flow rate.C. Core Flow recorder indication is correct."A" jet pump flow is being subtracted.D. Core Flow recorder indication is correct.Wednesday, January 21, 2015 2:26:52 PM 118 ILT-09 NRC EXAM"A" jet pump flow is NOT being subtracted.Wednesday, January 21, 2015 2:26:52 PM 119 ILT-09 NRC EXAM 3 8.295003AK2.04 001/03301P41/033.002.A.03/BANK/SYS-I/BOTH/295003AK2.04/1/1/H/3/JSC/ELJUnit 1 was operating at 100% RTP when a Loss of Offsite Power occurred. The following conditions exist: o"1E" 4160 VAC Bus ........................ DE-ENERGIZES and remains de-energizedo"1F" 4160 VAC Bus ........................ ENERGIZED o"1G" 4160 VAC Bus ........................ DE-ENERGIZES and remains de-energizedWith NO operator actions and based on the above conditions,The PSW System WILL supply cooling water to the Reactor Building __________ . The TOTAL number of PSW pumps supplying cooling water to these buildings is __________ . and Diesel Building ONLY; one (1)and Diesel Building ONLY: two (2)Turbine Building and Diesel Building; one (1)Turbine Building and Diesel Building; two (2) A.B.C.D.Description:The Turbine Building Isolation Valves (two valves in series in each division header) isolate onany of the following: A signal from either Core Spray System LOCA logic Div 1 or Div 2. (-101" RWL or 1.85psig D/W pressure)

LOSP - The valves will close after power is restored to the valves. During the power loss,the valves remain in the open position because they are MOVs and have no power toreposition.

Condenser Room Flooding - Level switches are located in the Cond Bay floor with a smalldam around each switch. The switches will pick up if water level increases to 3" above thefloor level. There is a keylock override switch (one per division) in the Control Room (P652 panel) to override the closure of the TB isolation valves due to a LOCA or LOSP signal. Power Supplies: "A" pump - 4160 VAC bus "E" (R22-S005)

"B" pump - 4160 VAC bus "G" (R22-S007)

"C" & "D" pumps - 4160 VAC bus "F" (R22-S006)Wednesday, January 21, 2015 2:26:52 PM 120 ILT-09 NRC EXAMWith 1E & 1G de-energized, none of the TB isolation valves have power to close, therefore willremain in their normal open position.

With 1F energized the 1C & 1D PSW pump will be running supplying water to the Reactor, Turbine and Diesel Generator Buildings.

K/A JUSTIFICATION:This question satisfies the K/A statement by requiring the applicant to know the response to thepartial loss of the emergency buses (loss of E/G) and the affected loads (PSW pumps andTurbine Building Isolation valves P41-F310A-D).The "A" distractor is plausible since the Turbine Building Isolation Valves do close on a LOSPhowever power has to be restored before they close automatically. If the valves close, flow will be to the Diesel Building and Reactor Building ONLY. The second part is plausible if theapplicant thinks about the power supply for the RHRSW pumps instead of the PSW pumps. THeapplicant may also think about 1E or 1G being energized then only one PSW pump would be running.The "B" distractor is plausible since the Turbine Building Isolation Valves do close on a LOSPhowever power has to be restored before they close automatically. If the valves close, flow willbe to the Diesel Building and Reactor Building ONLY. The second part is plausible since it iscorrect.The "C" distractor is plausible since the first part is correct. The second part is plausible if theapplicant thinks about the power supply for the RHRSW pumps instead of the PSW pumps. THe applicant may also think about 1E or 1G being energized then only one PSW pump would berunning.A. Incorrect - See description above. B. Incorrect - See description above. C. Incorrect - See description above. D. Correct - See description above. Wednesday, January 21, 2015 2:26:52 PM 121 ILT-09 NRC EXAM

References:

NONEK/A:295003 Partial or Complete Loss of A.C. PowerAK2. Knowledge of the interrelations between PARTIAL OR COMPLETE LOSS OF A.C.POWER and the following: (CFR: 41.7 / 45.8)AK2.04 A.C. electrical loads . . . . . . . . . . . . . . . 3.4 3.5LESSON PLAN/OBJECTIVE:P41-PSW-LP-03301, Plant Service Water, Ver 10.3, 033.002.A.03 and 033.015.A.02References used to develop this question:34AB-R22-002-1, Loss of 4160V Emergency Bus, Ver 1.10Bank question used from HLT Database LT-028025 008Wednesday, January 21, 2015 2:26:52 PM 122 ILT-09 NRC EXAM 3 9.295004AK2.01 001/02704R42/027.044.A.04/MOD/P-NORM/BOTH/295004AK2.01/1/1/H/2/ARB/ELJUnit 2 is operating at 70% RTP when the following sequence of events occur: o11:00 - 125/250V DC SWITCHGEAR 2B, 2R22-S017, de-energizes due to a faulto11:10 - 2R22-S017 fault is determined to be one of the previous in serviceDivision II 125 VDC Battery Chargers o11:24 - ALL Division II 125 VDC Battery Charger AC Input and DC Output breakers areplaced in the OFF position o11:30 - Maintenance replaces the Division II Battery Fuses and 2R22-S017 is re-energizedfrom the Division II Batteries IAW 34SO-R42-001-2, 125 VDC & 125/250 VDC System and with the above conditions, Local manipulation of a Throwover Switch ________ REQUIRED to return the newcombination of Division II 125 VDC Battery Chargers to service. When placing a Battery Charger in service, in order to PREVENT Battery Charger damage,the proper sequence of AC Input and DC Output breaker operation is to FIRST position the________ breaker to the ON position. is; AC Input is; DC Output is NOT; AC Input is NOT; DC Output A.B.C.D.Description:There are two throwover switches per division used to determine which chargers are in serviceand which is in standby. The throwover switches are manually operated, four pole switches,which allows the battery chargers to be rotated to equalize wear. The throwover switches arelocated in the associated 125/250 VDC Switchgear rooms. Below is a table depicting thethrowover switch alignment.Wednesday, January 21, 2015 2:26:52 PM 123 ILT-09 NRC EXAMWith 2 chargers normally in service the throwover switches will be aligned for charger output. Ifone of the in-service chargers is the faulted load the throwover switches will be re-aligned for thenew combination of chargers. The battery chargers on the 125/250 VDC Station Service Power Supply system requires the ACinput breaker closed first, then the DC output breaker closed when starting up. The order of breaker operation is to prevent damage to the main control card in the battery charger. The otherbattery chargers in the various DC systems are started by closing the DC output breaker and thenclosing the AC input breaker. If the AC input breaker was closed first, the charger sees a zero volts condition on the output and goes to maximum voltage. This could result in an excessivecurrent surge upon closure of the DC output breaker and could result in damage to the batterycharger. K/A JUSTIFICATION:This question satisfies the K/A statement by requiring the applicant to know how the throwoverswitches are arranged after a loss of 2R22-S017 DC switchgear and the proper relationship with restoring the individual breakers (AC Input and DC Output) for the new combination of batterychargers. The "B" distractor is plausible since the first part is correct. The second part is plausible if theapplicant remembers there are other battery chargers that are started by closing the DC output breaker and then closing the AC input breaker and thinks this is how the 125/250 BatteryChargers operate. The "C" distractor is plausible if the applicant does not understand the operation of the DCThrowover Switches and thinks the original alignment does not need to be changed for the 2Chargers being placed into service. The applicant may also think about how the Fire Protection Diesel pump battery chargers operate. The Fire Protection Diesels are started by one of two 24VDC batteries and chargers. The normal alignment is to have both chargers in service at alltimes. There is not a throwover in this system. The second part is plausible since it is correct. The "D" distractor is plausible if the applicant does not understand the operation of the DCThrowover Switches and thinks the original alignment does not need to be changed for the 2 Chargers being placed into service. The applicant may also think about how the Fire ProtectionDiesel pump battery chargers operate. The Fire Protection Diesels are started by one of two 24VDC batteries and chargers. The normal alignment is to have both chargers in service at all times. There is not a throwover in this system. The second part is plausible if the applicantremembers there are other battery chargers that are started by closing the DC output breaker andthen closing the AC input breaker and thinks this is how the 125/250 Battery Chargers operate. A. Correct - See description above. Wednesday, January 21, 2015 2:26:52 PM 124 ILT-09 NRC EXAMB. Incorrect - See description above. C. Incorrect - See description above. D. Incorrect - See description above. Wednesday, January 21, 2015 2:26:52 PM 125 ILT-09 NRC EXAM

References:

NONEK/A:295004 Partial or Complete Loss of D.C. PowerAK2. Knowledge of the interrelations between PARTIAL OR COMPLETE LOSS OF D.C.POWER and the following: (CFR: 41.7 / 45.8)AK2.01 Battery charger . . . . . . . . . . . . . . . . . . 3.1 3.1LESSON PLAN/OBJECTIVE:R42-ELECT-LP-02704, DC Electrical Distribution, Ver. 7.1, EO 027.044.A.04 References used to develop this question:34SO-R42-001-2, 125 VDC & 125/250 VDC System, Ver. 7.15 34SO-X43-001-1, Fire Pumps Operating Procedure, Ver 6.1Modified from HLT Database Q#LT-027030-002 Original QuestionWhich ONE of the choices below correctly describes the proper sequence of componentoperations that would PREVENT Battery Charger damage when placing a Unit 2, 125 VDC battery charger in service? A. Place the AC input breaker in the ON position, reposition throwover switches to therequired combination of battery chargers, then place the DC output breaker in the ONposition.B. Reposition throwover switches to the required combination of battery chargers, place theDC output breaker in the ON position, then place the AC input breaker in the ON position.C. Reposition throwover switches to the required combination of battery chargers, place theAC input breaker in the ON position, then place the DC output breaker in the ON position.D. Place the DC output breaker in the ON position, reposition throwover switches to therequired combination of battery chargers, then place the AC input breaker in the ONposition.Wednesday, January 21, 2015 2:26:52 PM 126 ILT-09 NRC EXAM 4 0.295005AK3.02 001/00401RRS/004.001.A.11/MOD/P-NORM/BOTH/295005AK3.02/1/1/F/3/JSC/ELJUnit 1 is operating late in core life with the EOC RPT breakers in service. If the Main Turbine trips, the MINIMUM reactor power level which will result in an EOC RPT breaker trip is __________ . The trip of the EOC RPT breakers is designed to prevent exceeding the __________ limit. 24.0%;APLHGR 24.0%;MCPR 27.6%;APLHGR 27.6%;MCPR A.B.C.D.Wednesday, January 21, 2015 2:26:52 PM 127 ILT-09 NRC EXAMDescription:The EOC-RPT instrumentation initiates a recirculation pump trip (RPT) to reduce the peak RPV pressure and power resulting from turbine trip or generator load rejection transients to provideadditional margin to core thermal MCPR Safety Limits (SLs). A pressure transient such as aturbine trip without bypass valves could add positive reactivity sufficient to approach MCPRlimits. Late in core life control rods are further out resulting in longer scram times. In addition,the core is more reactive due to decrease in the effective delayed neutron fraction, the voidcoefficient is less negative and control rod worth is less. To limit the reactivity effect due topump coast-down time during a load reject pressure transient, two RPT (Recirc Pump Trip)breakers have been installed in series between the ASD and the recirc pump motor. The EOC-RPT breakers are required to open when reactor power is >27.6% during a load rejector turbine trip to prevent approaching the MCPR limit. K/A JUSTIFICATION:This question satisfies the K/A statement by requiring the applicant to know the reason for EOCRPT breaker trips as it relates to a turbine trip and safety limits. The "A" distractor is plausible if the applicant thinks about the thermal power limit concerningTS 3.2.2 Minimum Critical Power Ratio (MCPR) with the turbine trip and EOC RPT breaker trip instead of the reactor power limit for the Safety limit concerning low flow/low pressureconditions. The second part is plausible since this is the reasoning behind the Safety limit for lowflow/low pressure conditions.The "B" distractor is plausible if the applicant thinks about the thermal power limit concerningTS 3.2.2 Minimum Critical Power Ratio (MCPR) with the turbine trip and EOC RPT breaker trip instead of the reactor power limit for the Safety limit concerning low flow/low pressureconditions. The second part is plausible since it is correct. The "C" distractor is plausible since the first part is correct. The second part is plausible sincethis is the reasoning behind the Safety limit for low flow/low pressure conditions.A. Incorrect - See description above. B. Incorrect - See description above. C. Incorrect - See description above. D. Correct - See description above. Wednesday, January 21, 2015 2:26:52 PM 128 ILT-09 NRC EXAM

References:

NONEK/A:295005 Main Turbine Generator TripAK3. Knowledge of the reasons for the following responses as they apply to MAINTURBINE GENERATOR TRIP: (CFR: 41.5 / 45.6)AK3.02 Recirculation pump downshift/trip: Plant-Specific . . . . . . . . . 3.4 3.5LESSON PLAN/OBJECTIVE:B31-RRS-LP-00401, Reactor Recirculation System, Ver 10.5, EO 004.001.A.11 and EO004.002.A.06References used to develop this question:Modified from HLT 3 audit exam #88 ORIGINAL QUESTIONWith the Unit 1 reactor operating near rated conditions at the end of an operatingcycle, the Recirculation System RPT Breakers are discovered to be inoperable.SELECT the statement that describes the most probable potential effect of the inopRPT Breakers if a Turbine Trip were to occur, andThe EOC-RPT instrumentation is required to be operable when thermal power isgreater than or equal to _________ % power.A. Pressure could exceed the reactor coolant system integrity safety limit.

27.6%B.The MCPR thermal limit may be exceeded.

27.6%C. The APLHGR thermal limit may be exceeded.

30%D. Cladding temperature could exceed 1,500 °F followin g the scram.

30%Wednesday, January 21, 2015 2:26:52 PM 129 ILT-09 NRC EXAM 4 1.295006G2.1.31 001/00101C11/001.013.A.07/MOD/SYS-I/BOTH/295006G2.1.31/1/1/H/2/JSC/ELJUnit 2 is operating at 100% RTP with the 2C11-F002A, Flow Control Valve, in service.Subsequently, a transient occurs resulting in RWL lowering to -10 inches (lowest RWL reached)before being restored to normal with Feedwater. One (1) minute later and based on the above indications/conditions,2C11-F002A __________ responding correctly. 2C11-F002A indicating LIGHTS are located __________ .is; 2H11-P603 and locally at the 2C82-P001, Remote Shutdown Panel is; on 2H11-P603 Panel ONLY is NOT; 2H11-P603 and locally at the 2C82-P001, Remote Shutdown Panel is NOT; on 2H11-P603 Panel ONLY A.B.C.D.Wednesday, January 21, 2015 2:26:53 PM 130 ILT-09 NRC EXAMDescription:Phil, this was question 3 of 10 of the previously submitted questions. Changes wereincorporated based on your ES-401-9 comments. The cause of the Scram can effect how the system responds. On a NON-LOCA SCRAM, theCRD pumps will remain running and the system will respond as follows: The flow control, valveF002A/B, senses maximum flow and closes. Due to valve construction, 5 gpm is maintained tothe CRD system to prevent thermal shock when the scram is reset and flow is returned to thenormal flow path. When the Scram is reset, the accumulators will recharge, the Flow Control Valve will open, and the system returns to normal conditions. The System response for a Scram with a LOCA signal is similar except the CRD pumps will tripand the flow controller will sense no flow, causing the Flow Control Valve (F002) to open fully.2C11-F002A/B indications are located on 2H11-P603. K/A JUSTIFICATION:This question satisfies the K/A statement by requiring the applicant to know the effect of aNON-LOCA scram has on C11-F002A, Flow Control valve and know where the indicationsexist in the MCR for the flow control valve.The "A" distractor is plausible since the first part is correct. The second part is plausible sinceother control rod drive components are located on 2C82-P001, Remote Shutdown Panel (2BCRD pump). The "C" distractor is plausible if the applicant thinks that the valve should have opened since theflow control valve will sense low flow condition and fail open on a LOCA scram since the CRD pumps will be tripped. The second part is plausible since other control rod drive components arelocated on 2C82-P001, Remote Shutdown Panel (2B CRD pump). The "D" distractor is plausible if the applicant thinks that the valve should have opened since theflow control valve will sense low flow condition and fail open on a LOCA scram since the CRDpumps will be tripped. The second part is plausible since it is correct. A. Incorrect - See description above. B. Correct - See description above. C. Incorrect - See description above. D. Incorrect - See description above. Wednesday, January 21, 2015 2:26:53 PM 131 ILT-09 NRC EXAM

References:

NONEK/A:295006 SCRAMG2.1.31 Ability to locate control room switches, controls, and indications, and to determinethat they correctly reflect the desired plant lineup. (CFR: 41.10 / 45.12) . . . . . 4.6 4.3LESSON PLAN/OBJECTIVE:C11-CRD-LP-00101, Control Rod Drive System, Ver 9.0, EO 001.013.A.07References used to develop this question:34SO-C11-005-2, Control Rod Drive Hydraulic System, Ver 32.0Modified from HLT Database Q#LT-001013-004 Original QuestionUnit 1 was operating at 100% RTP when a scram occurred on low Reactor water level (-20"lowest level reached). Which ONE of the choices below describes the response of the CRD system Flow ControlValve, 1C11-F002A? The FCV __________ , because (to) __________ .A. fully closes it senses high flow to the accumulators B. slightly closes return flow rate to its preset value C. slightly opens to return flow rate to its preset value D. fully opens it senses no CRD system flow Wednesday, January 21, 2015 2:26:53 PM 132 ILT-09 NRC EXAM 4 2.295008G2.1.20 001/01401B21/014.003.A.02/NEW/P-AB/BOTH/295008G2.1.20/1/2/H/2/JSC/ELJUnit 2 was operating at 100% RTP when a transient results in the following:oRWL increases to 102 inchesoRPV pressure is 490 psigA NPO places the control switch for Safety Relief Valve, 2B21-F013M, in the OPEN position. Based on the above conditions and when 2B21-F013M control switch is placed to OPEN, INITIALLY, 2B21-F013M will be passing __________ .2B21-F013M "Amber" indicating light will ILLUMINATE __________ .steam; immediately steam; after several seconds water; immediately water; after several seconds A.B.C.D.Description:Each relief valve tailpipe contains two pressure switches. Both of the tailpipe pressure switchesprovide an input to the Low-low set logic and are set at 85 psig. One pressure switch providesadditional inputs to SPDS for SRV position and illuminates an amber light above the relief valvecontrol switch on H11-P602. Actuation of either the temperature or pressure switch indicatesthat the relief valve may be passing steam to the suppression chamber due to leakage or actuation. The pressure switch may not actuate, or may have a delayed actuation, if the SRV ispassing water or 2 phase flow. Water begins entering the MSLs when RWL reaches 111" (U1 & U2). The fluid state at the SRV inlet will depend upon the water level in the MSL below the SRV. If the water level is lowthe SRV, when opened, will pass steam. If the water level is intermediate the SRV, whenopened, will pass steam with a rapid transition to two-phase flow. If the MSL is full, the valve, when opened, will pass liquid which may be transitioned to two-phase flow and then to steam. The water temperature under the above cases could range from approximately 200°F to 490°F[350°F to 60°F sub-cooling]. A longer total SRV opening time is possible when the MSL and SRVs are full of water. A total maximum valve opening time of 2 to 4 seconds could beanticipated under these conditions. A valve main disc stroke time of approximately 0.500seconds can be expected.In the January 26, 2000 Unit 1 scram, initial SRV manual actuations produced tailpipe temperatures peaking at 220°F or 380°-420°F. The lower peaks may be aresult of greater subcooling of the water passing through the SRVs.In the presence of a water or two phase mixture, when an SRV is opened:Wednesday, January 21, 2015 2:26:53 PM 133 ILT-09 NRC EXAM-Peak downstream tailpipe pressures may not reach the 85 psig required to actuate tailpipepressure switches for some time.

Water/two-phase mixtures create smaller shock forces thansaturated steam.-There may be a significant delay in receiving the amber light.

In the most recent scram, a delay of 3.5 minutes was seen in one case.-Tailpipe temperatures can range from about 220°F to 420°F.

The temperature trace indicatestemperatures of 400°F can be reached when the SRVDL pressure is maintained near 225 to 250psia.

-RPV pressure may not change appreciably.-Since the medium at the SRV is water or two phase flow, discharge leg temperatures for givenpressures cannot be predicted using an isenthalpic process on a Mollier diagram.K/A JUSTIFICATION:This question satisfies the K/A statement by requiring the applicant to be able to interpret a stepin the 34AB-C32-001-2 procedure concerning the Condensate system as it applies to the situation with RWL > +60 inches. The student must decide what actions to take if any arenecessary.The "B" distractor is plausible since the first part is correct. The second part is plausible if theapplicant thinks the MSLs are full of water. OE at Hatch proved that the amber light may not come on with water discharging on January 26, 2000. The applicant may also think that RPVpressure is too low to actually create the 85 psig to actuate the amber light. The "C" distractor is plausible if the applicant thinks that the bottom of the MSLs is at 100inches since 34AB-C32-001-2 has the operator secure all injection sources except CRD at +100inches and if the injection sources cannot be secured the operator will have to close the MSIVs if

+100 inches is exceeded. The second part is plausible since it is correct.The "D" distractor is plausible if the applicant thinks that the bottom of the MSLs is at 100inches since 34AB-C32-001-2 has the operator secure all injection sources except CRD at +100inches and if the injection sources cannot be secured the operator will have to close the MSIVs if+100 inches is exceeded. The second part is plausible if the applicant thinks the MSLs are full of water. OE at Hatch proved that the amber light may not come on with water discharging onJanuary 26, 2000. The applicant may also think that RPV pressure is too low to actually createthe 85 psig to actuate the amber light. A. Correct - See description above. B. Incorrect - See description above. C. Incorrect - See description above. D. Incorrect - See description above. Wednesday, January 21, 2015 2:26:53 PM 134 ILT-09 NRC EXAM

References:

NONEK/A:295008 High Reactor Water LevelG2.1.20 Ability to interpret and execute procedure steps. (CFR: 41.10 / 43.5 / 45.12) . . . . . . . . . . . . . 4.6 4.6LESSON PLAN/OBJECTIVE:B21-SLLS-LP-01401, Main Steam and Low Low Set, Ver 9.1, EO 014.003.A.02References used to develop this question:34AB-C32-001-2, Reactor Water Level Above +60 inches, Ver 1.1Wednesday, January 21, 2015 2:26:53 PM 135 ILT-09 NRC EXAM 4 3.295014AA2.04 001/40001C95/400.060.E.01/MOD/P-NORM/BOTH/295014AA2.04/1/2/H/3/ARB/ELJUnit 2 is operating at 65% RTP when an inadvertent initiation of HPCI occurs. The HPCI initiation signal will NOT reset. HPCI injects at rated flow for approximately 30 seconds before the NPO secures HPCI. The STA reports the following Thermal Limit values:MFLCPR 1.013 MFLPD 0.974 MAPRAT 0.936 PCRAT 0.979 With the above conditions,IAW 34SO-E41-001-2, High Pressure Coolant Injection (HPCI) System, to shutdown HPCI,the NPO DEPRESSED and HELD the Trip Pushbutton and __________ . A Thermal Limit VIOLATION __________ occur. IMMEDIATELY placed 2E41-F001, HPCI Turb Steam Supply valve, to the CLOSEposition; did IMMEDIATELY placed 2E41-F001, HPCI Turb Steam Supply valve, to the CLOSEposition; did NOT WAITED until HPCI speed reached ZERO rpm and then placed 2E41-C002-3, Aux OilPump, in the PULL-TO-LOCK OFF position; did WAITED until HPCI speed reached ZERO rpm and then placed 2E41-C002-3, Aux OilPump, in the PULL-TO-LOCK OFF position; did NOT A.B.C.D.Description:IAW 34SO-E41-001-2, High Pressure Coolant Injection (HPCI) System, contains the followingsteps:7.3.1.1 - IF the HPCI System must be shutdown with an initiation signal present, THEN usethe Preventing HPCI Injection to the RPV subsection of this procedure. 7.3.1.2 - Confirm the HPCI system is no longer required to be in operation for reactor vessellevel OR pressure control. Wednesday, January 21, 2015 2:26:53 PM 136 ILT-09 NRC EXAM7.3.1.3 - Confirm OR place in START control switch for 2E41-C002-2, Barom CndsrVacuum Pump. 7.3.1.4 - Confirm a HPCI Initiation Signal is NOT present, by depressing the HPCI InitiationSignal Reset pushbutton on panel 2H11-P601 AND confirming the white lightEXTINGUISHES. 7.3.1.4.1 - IF HPCI Initiation signal "White Light" will NOT reset, THEN shutdownHPCI per the 'Preventing HPCI Injection to the RPV', subsection 7.4.4. 7.3.1.5 - Using 2E41-R612, HPCI Flow Control, reduce turbine speed to about 2000 rpmAND depress AND hold the Turbine Trip pushbutton. 7.3.1.6 - Confirm 2E41-C002-3, Aux Oil Pump, AUTO starts, PRIOR to the turbine speeddecreasing below 1500 rpm. 7.3.1.7 - Close 2E41-F001, Turb Steam Supply Valve. 7.3.1.8 - WHEN 2E41-F001, Turb Steam Supply Valve, is fully CLOSED, release theTurbine Trip pushbutton. 7.4.4.1 - IF HPCI is NOT operating, place 2E41-C002-3, HPCI Aux Oil Pump, inPULL-TO-LOCK, panel 2H11-P601. 7.4.4.2 - IF HPCI is operating, perform the following: 7.4.4.2.1 - Depress AND hold the HPCI Turbine Trip pushbutton. 7.4.4.2.2 - WHEN the HPCI turbine has stopped, place 2E41-C002-3, HPCI Aux OilPump, in PULL-TO-LOCK. 7.4.4.2.3 - WHEN HPCI TURBINE BRG OIL PRESS LOW alarm is received, release theHPCI Turbine Trip push-button. Since the Initiation Signal will NOT reset, section 7.4.4 will be performed which places theHPCI Aux Oil pump in the PULL-TO-LOCK position. The plant process computer is continually monitoring the inputs from the local power rangemonitors (LPRMs) and other sources to determine the status of the plant's departure from established thermal limits. Computer programs such as the P-1 program and the 3D Monicoreprogram report the status of these limits. The reports show the established limits and actualcalculated values. Human Factors Engineering has been employed to enable the operator todetermine the safety of the plant with a minimum of effort or analysis. All analyses of the valuesfor the thermal limits have been programmed to provide a reported value of less than 1.0 fora safe condition. This allows the operator to quickly scan the report and determine the thermallimit status. The maximum fraction of limiting power distribution (MFLPD) for each region of the core iscalculated. Information provided includes the MFLPD limit, as well as the ten or twelve mostlimiting core locations for this limit. A listing of less than 1.0 for all regions of the core ensuresthat the linear heat generation rate (LHGR) limit has not been exceeded.The maximum average planar ratio (MAPRAT) for each region of the core is calculated. Information provided includes the MAPRAT limit, as well as the ten or twelve most limitingcore locations for this limit. A listing of less than 1.0 for all regions of the core ensures that theaverage planar linear heat generation rate (APLHGR) limit has not been exceeded. The maximum fraction of limiting critical power ratio (MFLCPR) for each region of the core iscalculated. Information provided includes the MFLCPR limit as well as the ten or twelve mostlimiting core locations for this limit. A listing of less than 1.0 for all regions of the core ensuresthat the critical power ratio (CPR) limit has not been exceeded. With the MFLCPR value greater than 1.0, Thermal limits have been violated. Wednesday, January 21, 2015 2:26:54 PM 137 ILT-09 NRC EXAMK/A JUSTIFICATION:This question satisfies the K/A statement by requiring the applicant to review the fuel thermallimits (MFLCPR, MFLPD, MAPRAT & PCRAT) and then determine if any of the thermallimits were violated during the positive reactivity being added by the cold water injection fromHPCI. The "A" distractor is plausible if the applicant remembers the section for the normal shut downof the HPCI system. In the normal shutdown section, the initiation signal will reset therefore theF001 is shut after pressing the Turbine trip pushbutton. The second part is plausible since it iscorrect. The "B" distractor is plausible if the applicant remembers the section for the normal shut downof the HPCI system. In the normal shutdown section, the initiation signal will reset therefore theF001 is shut after pressing the Turbine trip pushbutton. The second part is plausible if theapplicant remembers that one of the thermal limits is safe when the value is above 1.0 and thinksMFLCPR is the one that is >1.0 and the remaining thermal limits are indicating <1.0. The "D" distractor is plausible since the first part is correct. The second part is plausible if theapplicant remembers that one of the thermal limits is safe when the value is above 1.0 and thinks MFLCPR is the one that is >1.0 and the remaining thermal limits are indicating <1.0. A. Incorrect - See description above. B. Incorrect - See description above. C. Correct - See description above. D. Incorrect - See description above.

References:

NONEK/A:295014 Inadvertent Reactivity AdditionAA2. Ability to determine and/or interpret the following as they apply to INADVERTENTREACTIVITY ADDITION : (CFR: 41.10 / 43.5 / 45.13)AA2.04 fViolation of fuel thermal limits . . . . . . . . . . . . . . . . . . . . . . 4.1 4.4*LESSON PLAN/OBJECTIVE:C95-PC-LP-40001, Process Computer, Ver. 3.0, EO 400.060.E.01 Wednesday, January 21, 2015 2:26:54 PM 138 ILT-09 NRC EXAME41-HPCI-LP-00501, High Pressure Coolant Injection (HPCI), Ver. 6.0, EO 005.004.a.02References used to develop this question:34SO-E41-001-2, High Pressure Coolant Injection (HPCI) System, Ver. 28.3 34SV-SUV-020-0, Core Parameter Surveillance, Ver. 17.18Modified from HLT Database Q#206000A4.10-003 Original QuestionThe Unit 2 HPCI system is in service with the following parameters: oRPV water level ................ +38" (lowest level reached -15")oReactor Pressure ................ 920 psig oDrywell pressure ............... 1.0 psig (highest pressure reached 1.2 psig)IAW 34SO-E41-001-2, "High Pressure Coolant Injection (HPCI) System", to shutdown HPCI with the above conditions, the Trip Pushbutton is DEPRESSED and __________. A. 2E41-F001, HPCI Turb Steam Supply valve, is placed to the CLOSE position ONLYafter reaching ZERO rpm B. 2E41-F001, HPCI Turb Steam Supply valve, is placed to the CLOSE positionIMMEDIATELYC. 2E41-C002-3, Aux Oil Pump, is placed to the PULL-TO-LOCK OFF position ONLYafter reaching ZERO rpmD. 2E41-C002-3, Aux Oil Pump, is placed to the PULL-TO-LOCK OFF positionIMMEDIATELY Wednesday, January 21, 2015 2:26:54 PM 139 ILT-09 NRC EXAM 4 4.295015AK1.02 001/20328RCA/201.070.A.06/NEW/P-AB/BOTH/295015AK1.02/1/2/H/3/JSC/ELJUnit 1 was operating at 100% RTP when an ATWS occurred. oReactor power is 10% RTPoSBLC is injectingoRWL is being maintained between -110 inches and -140 inchesSubsequently, the following conditions exist: o45 Control Rods are at various positions (cannot be inserted)oSBLC Tank level is 12%Based on the current conditions, IAW 31EO-EOP-017, CP-3, RWL will be REQUIRED to be maintained between __________ .If a RPV pressure reduction occurs, Reactor power __________ to the point of Criticality. -110 inches and -140 inches; will increase -110 inches and -140 inches; will NOT return +3 inches and +50 inches; will increase +3 inches and +50 inches; will NOT return A.B.C.D.Description:Injection of the Cold Shutdown Boron Weight (CSBW) into the RPV also provides adequateassurance that the reactor is and will remain shutdown. The CSBW is the least weight of soluble boron which, if injected into the RPV and mixed uniformly, will maintain the reactor shutdownunder all conditions. This weight is utilized to assure the reactor will remain shutdownirrespective of control rod position or RPV temperature. CSBW has been injected when SBLC tank level reaches 14% or as determined by alternate injection methods. The Hot ShutdownBoron Weight (HSBW) is defined to be the least weight of soluble boron which, if injected intothe RPV and uniformly mixed, will maintain the reactor shutdown under hot standby conditions.

HSBW has been injected when SBLC tank level reaches 35% or as determined by alternateinjection methods. Wednesday, January 21, 2015 2:26:54 PM 140 ILT-09 NRC EXAMK/A JUSTIFICATION:This question satisfies the K/A statement by requiring the applicant to know the operationalimplications of cooling down will have on plant if CSBW is not injected into the RPV.The "A" distractor is plausible if the applicant thinks that 8% is the percentage required forCSBW to be injected. 8% is the lower instrument level tap for SBLC and this is where anoverride on the RCA flowchart has the crew secure SBLC to prevent damage to the pump.

Therefore is the applicant thinks CSBW has not been injected, RWL can not be increased. Thesecond part is plausible since reactivity is being added to the core due to the positive temperaturecoeffient of reactivity, however the core will remain shutdown under all conditions since CSBW has been injected. The core just will not have as much margin. The "B" distractor is plausible if the applicant thinks that 8% is the percentage required forCSBW to be injected. 8% is the lower instrument level tap for SBLC and this is where anoverride on the RCA flowchart has the crew secure SBLC to prevent damage to the pump.Therefore is the applicant thinks CSBW has not been injected, RWL can not be increased. The second part is plausible since it is correct.The "C" distractor is plausible since the first part is correct. The second part is plausible sincereactivity is being added to the core due to the positive temperature coeffient of reactivity,however the core will remain shutdown under all conditions since CSBW has been injected. Thecore just will not have as much margin. A. Incorrect - See description above. B. Incorrect - See description above. C. Incorrect - See description above. D. Correct - See description above. Wednesday, January 21, 2015 2:26:54 PM 141 ILT-09 NRC EXAM

References:

NONEK/A:295015 Incomplete SCRAMAK1. Knowledge of the operational implications of the following concepts as they apply toINCOMPLETE SCRAM : (CFR: 41.8 to 41.10)AK1.02 Cooldown effects on reactor power . . . . . . . . . . . . . . . . . . . . . 3.9 4.1LESSON PLAN/OBJECTIVE:EOP-RCA-LP-20328, RPV Control, ATWS (RCA), Ver 3.0, EO 201.070.A.06References used to develop this question:31EO-EOP-011-1, RCA RPV Control (ATWS), Ver 10.031EO-EOP-017-1, ATWS Level Control, Ver 11.0Wednesday, January 21, 2015 2:26:54 PM 142 ILT-09 NRC EXAM 4 5.295016G2.4.11 001/05201C82/010.018.A.02/BANK/P-AB/BOTH/295016G2.4.11/1/1/F/3/ARB/ELJAn evacuation of the Main Control Room has occurred. oThe Unit 1 Reactor was NOT scrammed prior to leaving the Control RoomoSPDS is NOT availableIAW 31RS-OPS-001-1, Shutdown From Outside Control Room, Guidance is given to LOCALLY scram the reactor by tripping the Scram Discharge Volume(SDV) __________ . This procedure provides guidance to confirm the Reactor is shutdown by visually verifyingthat each __________ .Thermal Level Switches; SDV Vent and Drain valve is CLOSED Thermal Level Switches; HCU Scram Inlet and Outlet valve is OPEN Float Level Switches; SDV Vent and Drain valve is CLOSED Float Level Switches; HCU Scram Inlet and Outlet valve is OPEN A.B.C.D.Description:There are no switches on the RSDP which will shutdown the reactor. There are two methods tolocally scram the reactor covered in the remote shutdown procedures. The Reactor Protection System (RPS) can be momentarily de-energized. This is done by opening breakers CB3A andCB3B in the RPS distribution panel, C71-P001. These breakers de-energize the Average PowerRange Monitors (APRMs) resulting in an APRM Inop scram signal. This panel is located on the 130' elevation of the Control Building (RPS MG set room). The automatic scram signal whichoccurs when the breakers are opened should result in the control rods fully inserting, thusshutting the reactor down. After both breakers have been opened reclose the breakers.

The Hi-Hi scram discharge volume level switch can be manually tripped to input a SCRAM. There are four level switches that are used to detect the level in the scram discharge volumewhere an auto scram is initiated (C11-NO13-A-D). These switches are on the Scram Discharge Volume piping located the 130' elevation of the Reactor Building, two of them on the northwestside, the other two on the southwest side. The switches are tripped to indicate a high level in thescram discharge volume. The switches are tripped by unscrewing the screw on top of the cover, removing the cover housing to expose the switch, and tripping the magnetrol switches. Whentwo switches on either the north side or south side are actuated (tripped) a scram occurs causingcontrol rods to be inserted. 31RS-OPS-001-1 calls (step 4.4.2) for tripping of the level switches 1C11-N013A-D which arethe float level switches (the thermals are 1C11-N060A-D). The procedure also has the operator check (step 4.6) the scram inlet/outlet valves are open (the SDV vent and drains are expected toWednesday, January 21, 2015 2:26:54 PM 143 ILT-09 NRC EXAMclose but are not used as indication of rod insertion). K/A JUSTIFICATION:This question satisfies the K/A statement by requiring the applicant to know 31RS-OPS-001-1steps for inputting a RPS scram signal (step 4.4.2) and where to monitor (step 4.6) for asuccessful scram. The "A" distractor is plausible since either the float switches or thermal switches will cause ascram due to SDV volume however the float switches are the only ones manipulated to cause amanual SDV scram per 31RS-OPS-001-1. The second part is plausible because when a scram isinput these valves will travel closed to bottle up the SDV but are not an indication that the individual control rod scram valves have opened. The "B" distractor is plausiblesince either the float switches or thermal switches will cause ascram due to SDV volume however the float switches are the only ones manipulated to cause amanual SDV scram per 31RS-OPS-001-1. The second part is plausible since it is correct. The "C" distractor is plausible since the first part is correct. The second part is plausible becausewhen a scram is input these valves will travel closed to bottle up the SDV but are not anindication that the individual control rod scram valves have opened. A. Incorrect - See description above. B. Incorrect - See description above. C. Incorrect - See description above. D. Correct - See description above. Wednesday, January 21, 2015 2:26:54 PM 144 ILT-09 NRC EXAM

References:

NONEK/A:295016 Control Room AbandonmentG2.4.11 Knowledge of abnormal condition procedures. (CFR: 41.10 / 43.5 / 45.13) . . . . . . . 4.0 4.2LESSON PLAN/OBJECTIVE:C82-RSDP-LP-05201, Remote Shutdown Panel (RSDP), Ver. 4.0, EO 010.018.A.02References used to develop this question:31RS-OPS-001-1, Shutdown From Outside Control Room, Ver. 5.24 34AB-C71-001-1, Scram Procedure, Ver. 12.7 57SV-C11-001-1, Scram Discharge Level FT&C, Ver. 6.0 Bank question used on HLT 2009-301 NRC ExamWednesday, January 21, 2015 2:26:54 PM 145 ILT-09 NRC EXAM 4 6.295018AA2.02 001/03301P41/033.015.A.01/MOD/SYS-B/BOTH/295018AA2.02/1/1/H/3/JSC/ELJUnit 2 is starting up at 2% RTP, with the CRD and RWCU Systems maintaining RWL.oRWCU dump flow is 50 gpmSubsequently:oRWCU dump flow is raised to 75 gpmo2P41-F316A and 2P41-F316D, Turbine Bldg. PSW Isolation valves, closeoCONDENSER ROOM FLOODING, 650-164, alarm is ILLUMINATEDRBCCW suction temperature is 102°F and increasing and the Reactor is manually scrammed.Based on the above conditions,The RBCCW system response is due to __________ . 2P41-F316A and 2P41-F316D __________ be overridden and re-opened from the MCR. a loss of cooling medium to the RBCCW Hx; cana loss of cooling medium to the RBCCW Hx; can NOT an excessive RWCU dump flow ONLY; can an excessive RWCU dump flow ONLY; can NOT A.B.C.D.Wednesday, January 21, 2015 2:26:54 PM 146 ILT-09 NRC EXAMDescription:Turbine Building Isolation Valves 2P41-F316A-D (1P41-F310A-D)The Turbine Building Isolation Valves (two valves in series in each division header) isolate onany of the following: a.A signal from either Core Spray System LOCA logic Div 1 or Div 2.

(-101" RWL or 1.85 psig D/W pressure)b.LOSP - The valves will close after power is restored to the valves. During the power loss,the valves remain in the open position because they are MOVs and have no power toreposition.c.Condenser Room Flooding - Level switches are located in the Cond Bay floor with a smalldam around each switch. Switches will pick up if water level rises to 3" above the floor level. d.There is a keylock override switch (one per division) in the Control Room (P652 panel) tooverride the closure of the TB isolation valves due to a LOCA or LOSP signal. With 2P41-F316A-D valves closing, all cooling water is isolated from the RBCCW Hx. Increasing RWCU Dump flow will cause RBCCW temp to increase, just not to the extent thatisolating all flow to the RBCCW Hx.K/A JUSTIFICATION:This question satisfies the K/A statement by requiring the applicant to know the effect of the lossof PSW and RWCU dump flow has on the RBCCW system cooling water temperature. The "A" distractor is plausible since the first part is correct. The second part is plausible sincethe TB isolation valves can be overriden for a LOCA or LOSP however condenser bay flooding cannot be overriden.The "C" distractor is plausible since the increased dump flow will cause NRHX temperature toincrease therefore increasing RBCCW cooling water temperature however this effect is minimalas compared to the loss of PSW to the RBCCW heat exchanger. The second part is plausiblesince the TB isolation valves can be overriden for a LOCA or LOSP however condenser bay flooding cannot be overriden.The "D" distractor is plausible since the increased dump flow will cause NRHX temperature toincrease therefore increasing RBCCW cooling water temperature however this effect is minimalas compared to the loss of PSW to the RBCCW heat exchanger. The second part is plausiblesince it is correct.A. Incorrect - See description above. B. Correct - See description above. C. Incorrect - See description above. D. Incorrect - See description above.

References:

Wednesday, January 21, 2015 2:26:54 PM 147 ILT-09 NRC EXAMNONEK/A:295018 Partial or Complete Loss of Component Cooling WaterAA2. Ability to determine and/or interpret the following as they apply to PARTIAL ORCOMPLETE LOSS OF COMPONENT COOLING WATER :

(CFR: 41.10 / 43.5 / 45.13)AA2.02 Cooling water temperature . . . . . . . . . . . . . . . . . . . . 3.1 3.2LESSON PLAN/OBJECTIVE:P41-PSW-LP-03301, Plant Service Water, Ver 10.3, EO 033.015.A.01References used to develop this question:34AB-P42-001-2, Loss of Reactor Building Closed Cooling Water, Ver 2.634AB-P41-001-2, Loss of Plant Service Water, Ver 12.0Modified from HLT-6 (2011-301) NRC Exam Q#87Original QuestionUnit 2 is starting up at 2% RTP, with the CRD and RWCU Systems maintaining RWL. oRWCU dump flow is 50 gpm.Subsequently: oRWCU dump flow is raised to 75 gpmo2P41-F316A and 2P41-F316D, Turbine Bldg. PSW Isolation valves, inadvertently close.RBCCW suction temperature is 102°F and increasing and the reactor is manually scrammed. Which ONE of the following identifies the cause of the RBCCW System response AND thereporting requirements IAW REG-0025, One, Four, and Eight Hour Reporting Requirements of10 CFR 50.72? A. Loss of cooling medium to the RBCCW Hx; 4 Hour report is requiredB. Loss of cooling medium to the RBCCW Hx; 1 Hour report is requiredC. Excessive RWCU dump flow ONLY; Wednesday, January 21, 2015 2:26:54 PM 148 ILT-09 NRC EXAM 4 Hour report is required D. Excessive RWCU dump flow ONLY; 1 Hour report is requiredWednesday, January 21, 2015 2:26:54 PM 149 ILT-09 NRC EXAM 4 7.295019AA1.02 001/03501P51/P70/200.025.A.03/MOD/SYS-I/BOTH/295019AA1.02/1/1/H/2/ARB/ELJUnit 2 is operating at 100% RTP when a complete (100%) rupture on the Non-Essential Instrument Air Header, down stream of 2P52-F015, occurs. With NO operator action, the 2P52-F015, Turb. Bldg Inst Air To RW Bldg Isol valve, will __________ .The setpoint for 2P52-F565, Rx Bldg Inst N 2 To Non-Int Air El. 185 Isol Vlv, toautomatically open is __________ . close and remain closed; 70 psig close and remain closed; 80 psig continously cycle open and closed; 70 psig continously cycle open and closed; 80 psig A.B.C.D.Wednesday, January 21, 2015 2:26:54 PM 150 ILT-09 NRC EXAMDescription:Automatic Initiations and Isolations:The standby Service Air Compressor (A or B) starts at 100 psig and "C" starts at 107 psig.The Nitrogen Backup Valve, F565, opens automatically at 80 psig decreasing to supplyNitrogen pressure to the Non-Interruptible Essential Air Header. The Air Compressor Discharge Valves F010A/B/C (F200A/B/C) close if compressor discharge pressure decreases to 80 psig. At 70 psig, the Service Air Isolation valve F017 closes, isolating the Service Air System.At 50 psig, Non-Essential Instrument Air Header Isolation Valve closes, isolating the Non-Essential Instrument Air Header. In this question, where the break is, the F015 would cycle close then open as pressure cycles up and down due to isolation of the leak and due to the location of the pressure sensing line(upstream of valve). K/A JUSTIFICATION:This question satisfies the K/A statement by requiring the applicant to determine (monitor) thestatus of Instrument air system valve (F015) with a loss of instrument air. The "A" distractor is plausible if the applicant thinks that the sensing line for F015 isdownstream of the valve. If downstream, the valve would close and remain closed. The second part is plausible if the applicant thinks about the pressure setpoint (70 psig) for the F017 andthinks this is the setpoint for the F565. The "B" distractor is plausible if the applicant thinks that the sensing line for F015 isdownstream of the valve. If downstream, the valve would close and remain closed. The secondpart is plausible since it is correct. The "C" distractor is plausible since the first part is correct. The second part is plausible if theapplicant thinks about the pressure setpoint (70 psig) for the F017 and thinks this is the setpoint for the F565. A. Incorrect - See description above. B. Incorrect - See description above. C. Incorrect - See description above. D. Correct - See description above.

References:

NONEK/A:Wednesday, January 21, 2015 2:26:54 PM 151 ILT-09 NRC EXAM295019 Partial or Complete Loss of Instrument AirAA1. Ability to operate and/or monitor the following as they apply to PARTIAL ORCOMPLETE LOSS OF INSTRUMENT AIR : (CFR: 41.7 / 45.6)AA1.02 Instrument air system valves: Plant-Specific . . . . . . . . . 3.3 3.1 LESSON PLAN/OBJECTIVE:P51-P52-P70-PLANT AIR-LP-03501, Plant Air Systems, Ver. 3.0, EO 200.025.a.03References used to develop this question:34SO-P51-002-2, Instrument And Service Air Systems, Ver. 21.5 Modified from question in HLT Database Q# LT-200025-066 ORIGINAL QUESTIONUnit 2 is at 100% rated power when a complete (100%) rupture on the Non-Essential InstrumentAir Header, down stream of 2P52-F015, occurs. With NO operator action, which ONE of the choices below describes how the plant will respondto this pipe break? The Turb. Bldg Inst Air To RW Bldg Isol valve, 2P52-F015, will __________; and,Wednesday, January 21, 2015 2:26:54 PM 152 ILT-09 NRC EXAMThe Service Air Header Isolation valve, 2P51-F017, will __________.A.continously cycle open and closed; close and remain closed;B. close and remain closed; close and remain closed;C. continously cycle open and closed; continously cycle open and closedD. close and remain closed; continously cycle open and closedWednesday, January 21, 2015 2:26:54 PM 153 ILT-09 NRC EXAM 4 8.295020AA1.02 001/01301T23/013.009.A.01/BANK/P-NORM/BOTH/295020AA1.02/1/2/H/3/JSC/ELJUnit 2 is operating at 100% RTP.oNitrogen (N

2) is being added to the Unit 2 Drywell (DW) from the Unit 1 N2 Storage Tank34SO-T48-002-2, "Containment Atmosphere Control and Dilution System" section 7.3.1,"Alternate Primary Containment Nitrogen Makeup From CAD loop A, Unit 1 or Unit 2 N 2Storage Tank" is being used to add the N 2.o2T48-F113, "Nitrogen to DW isolation valve" is OPENo2T48-F114, "Nitrogen to DW isolation valve" is OPENoDW venting using Standby Gas Treatment is in progressoAn inadvertant loss of the"2A" Reactor Protection System (RPS) bus occurredoIt will take 8 hours9.259259e-5 days <br />0.00222 hours <br />1.322751e-5 weeks <br />3.044e-6 months <br /> to restore the "2A" RPS busIf the operator stationed at the 2H11-P657 panel does NOT take any action, which ONE ofthe following describes the operational implications for the DW?

N 2 addition to the DW will continue and a loss of DW cooling will eventually occur due tohigh DW pressure N 2 addition to the DW will continue and DW cooling will remain in operation indefinitely N 2 addition to the DW will automatically isolate and DW cooling will remain in operationindefinitely

N 2 addition to the DW will automatically isolate and a simultaneous loss of DW cooling willoccur A.B.C.D.Wednesday, January 21, 2015 2:26:54 PM 154 ILT-09 NRC EXAMDescription:The procedure (34SO-T48-002-2) explains that 2T48-F113, 2T48-F114 will not close on aGroup 2 isolation and that an operator must be stationed to close the valves in the event that anisolation signal/condition actually occurs. As a result of the loss of RPS, a Group 2 signal will be generated; however, nitrogen willcontinue to be added to the Drywell. The Group 2 signal will isolate the DW vent line up. As aresult, Drywell pressure will increase and eventually result in a High Drywell pressure LOCAsignal (assuming no operator actions are taken, which the question stem states). Typically nitrogen addition to the Drywell automatically isolates on a Group 2 isolation signal; however, when performing Alternate Nitrogen makeup (as in this question), it does notautomatically isolate.K/A JUSTIFICATION:This question satisfies the K/A statement by requiring the applicant to know how a Group 2signal will affect the Drywell atmosphere when making up Nitrogen to the Drywell via alternatemethods.The "B" distractor is plausible since Drywell cooling will be lost eventually when a HighDrywell pressure LOCA signal occurs. The second part is plausible if the candidate assumes that Drywell venting remains in service.The "C" distractor is plausible since Nitrogen addition will not auto isolate and Drywell coolingwill eventually be lost when a Drywell pressure LOCA signal occurs. Plausible if the candidateassumes nitrogen addition auto isolates due to the Gr 2 signal generated by the loss of RPS.The "D" distractor is plausible since nitrogen addition does not auto isolate. 2nd part is notcorrect, Drywell cooling is not lost until the Drywell pressure signal is received (in this case).

Plausible if the candidate assumes that Drywell cooling is lost as a result of a Group 2 signalwhich is generated due to the RPS loss. Drywell cooling loss would actually occur if a Group 2signal due to Drywell pressure (1.85 psig) occurs A. Correct - See description above. B. Incorrect - See description above. C. Incorrect - See description above. D. Incorrect - See description above. Wednesday, January 21, 2015 2:26:54 PM 155 ILT-09 NRC EXAM

References:

NONEK/A:295020 Inadvertent Containment IsolationAA1. Ability to operate and/or monitor the following as they apply to INADVERTENTCONTAINMENT ISOLATION : (CFR: 41.7 / 45.6) AA1.02 Drywell ventilation/cooling system . . . . . . . . . . . . . . . . . . . . 3.2 3.2LESSON PLAN/OBJECTIVE:T23-PC-LP-01301, Primary Containment, Ver 7.1, EO 013.009.A.01References used to develop this question:34SO-T48-002-2, Containment Atmospheric Control and Dilution Systems, Ver 25.234AB-C71-002-2, Loss of RPS, Ver 5.4Bank question from 2009 HLT 4 NRC exam question 48 (HLT questions NRC2009301 048)Wednesday, January 21, 2015 2:26:54 PM 156 ILT-09 NRC EXAM 4 9.295021AK3.01 001/00701E11/007.007.A.02/BANK/P-AB/BOTH/295021AK3.01/1/1/F/3/ARB/ELJWith the plant shutdown in Mode 4 and RHR loop "B" operating in Shutdown Cooling, thefollowing conditions exist: oRPV pressure is 0 psigoRecirc suction temperature is 170°FoReactor water level indicates 58 inches on 2C32-R606A-C, GEMACsoRHR Loop "B" flow is 7700 gpmRHR Loop "B" develops a leak and a SDC isolation results. Following the isolation thefollowing parameters exist: oRPV pressure is 0 psigoRecirc suction temperature is 170°FoRWL indicates 1 inch on 2C32-R606A-CIAW 34AB-E11-001-2, Loss of Shutdown Cooling, RWL will be raised to a MINIMUM of 53 inches as indicated on __________ .The reason RWL is raised is to _________ . 2B21-R605, Floodup Range; increase coolant inventory ONLY 2B21-R605, Floodup Range; increase coolant inventory AND ensure a flow path for natural circulation 2C32-R606A, Narrow Range; increase coolant inventory ONLY 2C32-R606A, Narrow Range; increase coolant inventory AND ensure a flow path for natural circulation A.B.C.D.Wednesday, January 21, 2015 2:26:54 PM 157 ILT-09 NRC EXAMDescription:RWL must be greater than +53 inches if RHR flow is less than 7700 gpm (>53 inches allows for natural circulation). If RHR flow is greater than 7700 gpm, RWL must be maintained above +33inches. When shutdown cooling is placed in service, RHR flow is adjusted to 6200-8200 gpm(7700-8200 gpm). This ensures adequate cooling and flow through the core (preventsstratification). IAW 34AB-E11-001-2, Loss of Shutdown Cooling, step 4.6 states "IF a minimum RHR waterflow of 7700 gpm through the reactor vessel CANNOT be maintained, increase reactor vesselwater level greater than 53 inches corrected AND as high as possible within the indicatedrange to ensure a flow path for natural circulation AND to increase coolant inventory." Also the prior NOTE states: "During cold conditions (£212°F), 2C32-R606A, 2C32-R606B,AND 2C32-R606C, Reactor Level Instruments, read approximately 15 inches higher than actualreactor water level. To utilize 2B21-R605 for water level determination, refer to34SV-SUV-019-2." K/A JUSTIFICATION:This question satisfies the K/A statement by requiring the applicant to know that with a loss ofSDC, RWL is raised to >+53 inches to increase coolant inventory and to promote naturalcirculation flow in the core. The "A" distractor is plausible since the first part is correct. The second part is plausible if theapplicant understands that by increasing RWL to >53 inches, coolant inventory will thus beincreased, thinking this is the reason to raise RWL and does not consider a flow path for naturalcirculation. The "C" distractor is plausible if the applicant does not remember the Narrow Range RWLinstruments have a (-) 15" correction factor while in Mode 4 and thinks the normal (0-60") RWL instruments are still valid. The second part is plausible if the applicant understands that byincreasing RWL to >53 inches, coolant inventory will thus be increased, thinking this is thereason to raise RWL and does not consider a flow path for natural circulation. The "D" distractor is plausible if the applicant does not remember the Narrow Range RWLinstruments have a (-) 15" correction factor while in Mode 4 and thinks the normal (0-60") RWL instruments are still valid. The second part is plausible since it is correct. A. Incorrect - See description above. B. Correct - See description above. C. Incorrect - See description above. D. Incorrect - See description above. Wednesday, January 21, 2015 2:26:54 PM 158 ILT-09 NRC EXAM

References:

NONEK/A:295021 Loss of Shutdown CoolingAK3. Knowledge of the reasons for the following responses as they apply to LOSS OFSHUTDOWN COOLING : (CFR: 41.5 / 45.6) AK3.01 Raising reactor water level . . . . . . . . . . . . . . . . . . . . . . . . . . . 3.3 3.4LESSON PLAN/OBJECTIVE:E11-RHR-LP-00701, Residual Heat Removal System, Ver. 9.1, EO 007.007.A.02 References used to develop this question:34AB-E11-001-2, Loss Of Shutdown Cooling, Ver. 6.13 Wednesday, January 21, 2015 2:26:54 PM 159 ILT-09 NRC EXAM 5 0.295023AK1.03 001/05401RMCS/001.010.A.12/MOD/P-NORM/BOTH/295023AK1.03/1/1/F/3/JSC/ELJUnit 1 has just entered Mode 5.oALL Control Rods FULLY insertedoRPS Shorting Links INSTALLEDSubsequently, while performing HCU tagouts, it is discovered that four (4) Control Rods in

o ne quadrant have re-positioned to the FULL OUT position. SRM A indicates the following: o300 cps and rising oSlight positive periodThe remaining SRMs continue to indicate 30 cps. Based on these indications and with NO operator actions:Annunicator, ROD DRIFT, 603-247, will FIRST be ILLUMINATED as soon as a ControlRod moves __________ . If SRM A continues to increase, SRM A __________ automatically initiate a FULL Scram tostop the Reactor Power increase. away from position 00; will away from position 00; will NOT past position 02; will past position 02; will NOT A.B.C.D.Description:When a control rod is at any other reed switch other than an even reed switch position, a rod driftalarm occurs unless the rod drift is bypassed for that control rod. Drift alarm senses power to theinsert, withdraw and settle buses. The alarm is bypassed when any of the three are energized forthat rod. A rod drifting while other normal rod movement is being conducted will still initiate a"ROD DRIFT" alarm.The UPSCALE TRIP (High High scram) is used only during refueling operations (setpoint 3 x 10 5 cps). With the shorting links removed, a single trip signal from any nuclear instrumentchannel (8 IRMs or 4 SRMs) or a single 2/4 voter module will cause a Full Reactor scram. For asingle 2/4 logic module to trip, it must see a trip from at least 2 OPERABLE & UNBYPASSED APRMs. The shorting links are required to be removed by the Technical Requirements ManualWednesday, January 21, 2015 2:26:54 PM 160 ILT-09 NRC EXAMin Mode 5 with any Control Rod withdrawn from a core cell containing one or more fuelassemblies and SDM not demonstrated per 42CC-ERP-010-0, Shutdown Margin Demonstration,for current core configuration.K/A JUSTIFICATION:This question satisfies the K/A statement by requiring the applicant to know the indications of acritical quadrant and the know the operational implications as associated with increasing reactorpower with the shorting links installed based on the input from the SRMs.The "A" distractor is plausible since the first part is correct. The second part is plausible if theapplicant thinks about the operational implications associated with the Shorting links removed.

A full reactor scram will occur if any SRM/IRM/Voter Module were to have a scram signal (1 of16 logic). With the shorting links installed, the SRMs can not cause a scram and IRM/PRMsoperate normally.The "C" distractor is plausible if the applicant thinks that Rod Drift alarm is only actuated whenan odd number reed switch is made up with the insert, withdraw or the settle bus not energized.

The applicant may think that position 02 is correct since the RWM uses position 02 indetermining if the reactor is shutdown based on rod positions. The second part is plausible if theapplicant thinks about the operational implications associated with the Shorting links removed.

A full reactor scram will occur if any SRM/IRM/Voter Module were to have a scram signal (1 of16 logic). With the shorting links installed, the SRMs can not cause a scram and IRM/PRMsoperate normally.The "D" distractor is plausible if the applicant thinks that Rod Drift alarm is only actuated whenan odd number reed switch is made up with the insert, withdraw or the settle bus not energized.

The applicant may think that position 02 is correct since the RWM uses position 02 indetermining if the reactor is shutdown based on rod positions. The second part is plausible sinceit is correct.A. Incorrect - See description above. B. Correct - See description above. C. Incorrect - See description above. D. Incorrect - See description above. Wednesday, January 21, 2015 2:26:54 PM 161 ILT-09 NRC EXAM

References:

NONEK/A:295023 Refueling AccidentsAK1. Knowledge of the operational implications of the following concepts as they apply toREFUELING ACCIDENTS : (CFR: 41.8 to 41.10) AK1.03 Inadvertent criticality . . . . . . . . . . . . . . . . . . . . . . . . 3.7 4.0LESSON PLAN/OBJECTIVE:C11-RMCS-LP-05401, Reactor Manual Control System, Ver 6.0, EO 001.010.A.12References used to develop this question:34AR-603-247-1, ROD DRIFT, Ver 5.2Modified from HLT Database which was used on HLT-5 NRC Exam Q#15 Original QuestionUnit 2 is in a refueling outage.Plant conditions:oReactor Protection System (RPS) Shorting Links have been REMOVEDoDue to a detector malfunction, the SRM "A" count rate begins to riseWhich ONE of the choices below completes the following statement?When SRM "A" count rate increases to 4 x 10 5 cps , THEN a __________ will exist. A. control rod block (ONLY) B. control rod block AND a trip in RPS Channel "A" (ONLY)

C. control rod block AND a trip in BOTH RPS Channels D. "SRM Upscale OR Inoperative" (603-204) alarm (ONLY) Wednesday, January 21, 2015 2:26:54 PM 162 ILT-09 NRC EXAM 5 1.295024EA2.08 001/01301T23/013.046.A.05/MOD/SYS-B/BOTH/295024EA2.08/1/1/H/2/ARB/ELJUnit 2 was operating at 55% RTP, due to a fuel leaker, when a steam line break insidecontainment occurs.Plant conditions are below:oDrywell pressure is 10.5 psig and slowly increasingoDrywell radiation is 95.0 R/hr and steadyoTorus pressure is 10.0 psig and slowly increasingo Neither Drywell or Torus sprays have been initiatedWith the above conditions, At this time, ________ Sprays are REQUIRED to be placed in service. The AMBER light above 2T48-F318, Torus Vent Valve, will be __________ . BOTH Drywell and Torus; extinguished BOTH Drywell and Torus; illuminated ONLY Torus; extinguished ONLY Torus; illuminated A.B.C.D.Description:The determination that Torus pressure cannot be maintained below 11 psig (Unit1 10 psig),which is the Suppression Chamber Spray Initiation Pressure (SCSIP), can be made beforereaching the actual limit based on trend or future prediction based on other plant conditions. Torus sprays are initiated before exceeding the Suppression Chamber Spray Initiation Pressure of11 psig (Unit 1 10 psig). Although operation of Torus sprays may not, by itself, preclude chugging, torus sprays are initiated before reaching the Suppression Chamber Spray InitiationPressure to assure that operation of this system is attempted for reducing primary containmentpressure before operation of Drywell Sprays is directed. Also, it is unknown if the high pressure was caused by noncondensables being forced into the torus or if there exists some bypass pathwhich has created the elevated pressure. With Unit 2 Torus pressure 10 psig only Torus sprays will be initiated. At 138 R/hr in the drywell, all the primary containment 18" purge and vent valves close. Thevalves are; Drywell purge, T48-F307 and F308; Torus purge, T48-F309 and F324; Drywell vent,Wednesday, January 21, 2015 2:26:54 PM 163 ILT-09 NRC EXAMT48-F319 and F320; and Torus vent T48-F318 and F326.

If the valves are closed on a HighRadiation signal, then an amber light above the valve indicator on H11-P602 will illuminate totell the operator that the valves closed, or would have closed, on high radiation in the drywell.NOTE: These valves are part of PCIS Group 2.Group 2 Isolation;a.Either of the following conditions will cause a Group 2 isolation:1)Reactor Water Level Low (Level 3) (+3.0")2)Drywell Pressure High (1.85 psig)IAW NMP-EP-110-GL02, HNP-EALs-ICs, Threshold Values and Basis, 84 R/Hr is a loss of the RCS Barrier on the DW radiation monitors. KA JUSTIFICATION:This question satisfies the K/A statement by requiring the applicant to interpret highDrywell/Torus pressures along with Drywell radiation levels and determine if a PCIS isolation signal will be indicated by the high Drywell radiation level (Amber light above 2T48-F318).The "A" distractor is plausible if the applicant remembers the Unit 1 value of 10.0 psig in theTorus and thinks both the Drywell & Torus are required to have sprays initiated and would becorrect if asking on Unit 1. The second part is plausible since it is correct. The "B" distractor is plausible if the applicant remembers the Unit 1 value of 10.0 psig in theTorus and thinks both the Drywell & Torus are required to have sprays initiated and would be correct if asking on Unit 1. The second part is plausible if the applicant remembers 84 R/hr(radiation value for RCS Barrier) and thinks that this is the value associated with that the amberlight and thinks it will be illuminated (recent change from 138 R/hr). The "D" distractor is plausible since the first part is correct. The second part is plausible if theapplicant remembers 84 R/hr (radiation value for RCS Barrier) and thinks that this is the value associated with that the amber light and thinks it will be illuminated (recent change from 138R/hr). A. Incorrect - See description above. B. Incorrect - See description above. C. Correct - See description above. D. Incorrect - See description above.

References:

NONEK/A:Wednesday, January 21, 2015 2:26:54 PM 164 ILT-09 NRC EXAM295024 High Drywell PressureEA2. Ability to determine and/or interpret the following as they apply to HIGHDRYWELL PRESSURE: (CFR: 41.10 / 43.5 / 45.13) EA2.08 Drywell radiation levels . . . . . . . . . . . . . . . . . . . . . 3.6 4.0LESSON PLAN/OBJECTIVE:T23-PC-LP-01301, Primary Containment, Ver. 7.1, EO 200.004.A.03 EOP-PC-LP-20310, Primary Containment Control (PC), Ver. 3.0, EO 201.076.A.16 References used to develop this question:31EO-EOP-012-2, PC Primary Containment Control, Ver. 6.0 34AR-602-436-2, Containment Radiation High/Inop, Ver. 2.4 NMP-EP-110-GL02, HNP EALs - ICs, Threshold Values and Basis, Ver 3.0Modified from HLT Database Q#295024EA2.06-001, which also was used on 2012-301 HLT-7 NRC Exam Q#50Original QuestionA steam line break inside containment has occurred on Unit 2.oDrywell pressure is 10.5 psig and slowly increasing oTorus pressure is 10 psig and slowly increasingoNeither Drywell or Torus sprays have been initiatedWhich ONE of the following describes the effect of the steam line break on Torus watertemperature and requires Primary Containment sprays with these conditions? The Torus water temperature will heat up __________ .With the above conditions and at this time, ________ Sprays are REQUIRED to be placed inservice. A. uniformly throughout the Torus due to the design of the downcomers;BOTH Drywell and TorusB. uniformly throughout the Torus due to the design of the downcomers;ONLY Torus C. directly under the area of the DW leak due to the energy being distributed directly to theTorus water in that area;Wednesday, January 21, 2015 2:26:54 PM 165 ILT-09 NRC EXAMBOTH Drywell and TorusD. directly under the area of the DW leak due to the energy being distributed directly to theTorus water in that area;ONLY Torus Wednesday, January 21, 2015 2:26:54 PM 166 ILT-09 NRC EXAM 5 2.295025EK2.01 001/01001C71/300.008.A.02/MOD/P-AB/BOTH/295025EK2.01/1/1/H/3/JSC/ELJUnit 2 is operating at 73% RTP when the following MSIVs close:o2B21-F022A, Inboard Main Steam Isolation Valveo2B21-F028B, Outboard Main Steam Isolation ValveoRPV pressure peaks at 1078 psigBased on the above conditions,Closure of the above combination of MSIVs, __________ result in a RPS half (1/2) Scramsignal from MSIV position input to the RPS logic. The High Reactor Pressure RPS Scram setpoint __________ been exceeded. will; has will; has NOT will NOT; has will NOT; has NOT A.B.C.D.Description:Bases: High pressure in the Reactor could cause a rupture to the nuclear system process barrier,resulting in the release of fission products. The Reactor high-pressure scram works inconjunction with the pressure relief system in preventing RPV pressure from exceeding themaximum allowable pressure. The Reactor high-pressure scram also protects the core fromexceeding thermal-hydraulic limits during certain pressure transients, which occur when theReactor is operating at less than rated power and flow. The scram setpoint is chosen far enoughabove the normal operating pressure to avoid spurious scrams, yet set low enough to provide awide margin to the maximum allowable pressure. The scram signal is set to 1074 psig.1)The MSIV scram logic is as follows:a)Closure of any MSIV: NO ACTION b)Closure of either MSIV in MSLs A & D (B & C): NO ACTIONc)Closure of either MSIV in MSLs A & B (or C & D): HALF SCRAM CHANNEL "A"d)Closure of either MSIV in MSLs A & C (or B & D): HALF SCRAM CHANNEL "B" e)Closure of any MSIV in three MSLs: FULL SCRAMK/A JUSTIFICATION:This question satisfies the K/A statement by requiring the applicant to diagnose a reactor scramWednesday, January 21, 2015 2:26:54 PM 167 ILT-09 NRC EXAMand determine its cause. The applicant has to know how the MSIV logic works and how theclosure of the MSIVs affects RPV pressure which causes the transient.The "B" distractor is plausible since the first part is correct. The second part is plausible if theapplicant thinks the RPS scram logic signal for RPV pressure is 1080 psig which the the TS setpoint for the RPV pressure. TS requires the setpoint to be < 1080 psig. The setpoint is 1074psig which is a conservative pressure to TS.The "C" distractor is plausible since not all combinations of MSIV closures will result in theRPS actuation (half scram). Combinations that will cause a half scram are A/B, D/C, B/D, andA/C. Combinations that will not cause a half scram are B/C and A/D. The second part isplausible since it is correct. The "D" distractor is plausible since not all combinations of MSIV closures will result in theRPS actuation (half scram). Combinations that will cause a half scram are A/B, D/C, B/D, andA/C. Combinations that will not cause a half scram are B/C and A/D. The second part isplausible if the applicant thinks the RPS scram logic signal for RPV pressure is 1080 psig which the the TS setpoint for the RPV pressure. TS requires the setpoint to be < 1080 psig. The setpointis 1074 psig which is a conservative pressure to TS.A. Correct - See description above. B. Incorrect - See description above. C. Incorrect - See description above. D. Incorrect - See description above. Wednesday, January 21, 2015 2:26:55 PM 168 ILT-09 NRC EXAM

References:

NONEK/A:295025 High Reactor PressureEK2. Knowledge of the interrelations between HIGH REACTOR PRESSURE and thefollowing: (CFR: 41.7 / 45.8)EK2.01 RPS . . . . . . . . . . . . . . . 4.1* 4.1LESSON PLAN/OBJECTIVE:C71-RPS-LP-01001, Reactor Protection System, Ver 8.2, EO 300.008.A.02References used to develop this question:34AR-603-104-2, MSIVs Not Fully Open Trip, Ver 3.0 Modified from HLT Database Q#LT-300008-006 Original QuestionUnit 2 is at 60% power when Inboard MSIV 2B21-F022B and Outboard MSIV 2B21-F028Ainadvertently fail closed.Which ONE of the choices below completes the following statement?INITIALLY, reactor power will __________; and, a RPS half scram signal __________ be generated.A. increase; will B. increase; will NOT C. remain the same; will D. remain the same; will NOT Wednesday, January 21, 2015 2:26:55 PM 169 ILT-09 NRC EXAM 5 3.295026EK1.02 001/20310PC/201.074.A.09/MOD/P-EOP/BOTH/295026EK1.02/1/1/F/3/ARB/ELJA leak inside the Drywell (DW) has occurred on Unit 2 resulting in the following conditions: oDW pressure is 1.2 psig and slowly increasingoDW temperature is 135°F and slowly increasingoTorus pressure is 0.8 psig and slowly increasingoTorus temperature is 101°F and slowly increasingWith the above conditions, Steam condensation from the event will cause Torus water temperature to heat up

___________ . IAW 31EO-EOP-012-2, PC Primary Containment Control, the LOWEST listed Torustemperature requiring entry into RC Point A of 31EO-EOP-010-2, RC RPV Control(NON-ATWS), is __________ . uniformly throughout the Torus due to the design of the downcomers; 111°F uniformly throughout the Torus due to the design of the downcomers; 121°F directly under the area of the DW leak due to the energy being distributed directly to theTorus water in that area; 111°F directly under the area of the DW leak due to the energy being distributed directly to theTorus water in that area; 121°F A.B.C.D.Description:The steam will enter the torus via a ring header and downcomers. The ring header helps toensure steam distribution is approximately equal throughout the torus. The plausibility for localarea heating of the torus is SRVs leaking. Torus temperature control is necessary to preserve the capability of the plant's emergency heatsink to depressurize the reactor. The loss of the plant's pressure suppression capability due tohigh torus water temperature may permit high containment pressures to be reached since thedecay heat energy in the form of pressure cannot be quenched. As torus water temperature increases, torus cooling is placed in service. If bulk torus temperature increases to 110°F, amanual reactor scram (Mode switch to S/D) is required per Tech Specs. IAW 31EO-EOP-012-2, PC Primary Containment Control, entering either RC or RCA flowchartat point "A" assures that, if possible, the reactor is scrammed and shutdown by control rodinsertion before the requirement for boron injection is reached. Entry into the RC [A] must be explicitly stated because conditions requiring entry into the PC flowchart do not necessarilyWednesday, January 21, 2015 2:26:55 PM 170 ILT-09 NRC EXAMrequire entry into the RC [A] flowchart.

Therefore, a scram may not have been initiated yet.

IFSuppression Pool bulk average temperature exceeds 110ºF, RC Point A is entered.

110°F is thelimit for exceeding the Boron Injection Initiation Temperature.

K/A JUSTIFICATION:This question satisfies the K/A statement by requiring the applicant to understand the concept ofsteam condensing into the Torus either by SRV discharge or LOCA and as a result of the Torusheating up from this steam condensation, the temperature at which a reactor scram is required. The "B" distractor is plausible since the first part is correct. The second part is plausible if theapplicant remembers that 120°F is the TS temperature at which the NPO enters34GO-OPS-013-2, Normal Plant Shutdown, AND reduces RPV pressure to < 200 PSIG within12 hours. The "C" distractor is plausible if the applicant does not understand the purpose/design of the ringheader/ downcomers and does not remember that the downcomer will evenly distribute the heat.

The applicant could think the downcomer/ring header works like the SRVs since a SRV openingwill heat up a local area of the Torus based on which SRV opens. The second part is plausiblesince it is correct. The "D" distractor is plausible if the applicant does not understand the purpose/design of the ringheader/ downcomers and does not remember that the downcomer will evenly distribute the heat.

The applicant could think the downcomer/ring header works like the SRVs since a SRV openingwill heat up a local area of the Torus based on which SRV opens. The second part is plausible ifthe applicant remembers that 120°F is the temperature at which the NPO enters 34GO-OPS-013-2, Normal Plant Shutdown, AND reduces RPV pressure to < 200 PSIG within12 hours. A. Correct - See description above. B. Incorrect - See description above. C. Incorrect - See description above. D. Incorrect - See description above.

References:

NONEK/A:295026 Suppression Pool High Water TemperatureEK1. Knowledge of the operational implications of the following concepts as they apply toSUPPRESSION POOL HIGH WATER TEMPERATURE : (CFR: 41.8 to 41.10)Wednesday, January 21, 2015 2:26:55 PM 171 ILT-09 NRC EXAMLESSON PLAN/OBJECTIVE:EOP-PC-LP-20310, Primary Containment Control (PC), Ver. 3.0, EO 201.074.A.14 & EO 201.074.A.08 References used to develop this question:31EO-EOP-012-2, PC Primary Containment Control, Ver. 6.0 Modified from ILT-7 NRC Exam 2012-301 Q#50 Original QuestionA steam line break inside containment has occurred on Unit 2. oDrywell pressure is 10.5 psig and slowly increasingoTorus pressure is 10 psig and slowly increasingoNeither Drywell or Torus sprays have been initiatedWhich ONE of the following describes the effect of the steam line break on Torus water temperature and requires Primary Containment sprays with these conditions? The Torus water temperature will heat up __________ With the above conditions and at this time, ________ Sprays are REQUIRED to be placed inservice. A. uniformly throughout the Torus due to the design of the downcomers; BOTH Drywell and Torus B. uniformly throughout the Torus due to the design of the downcomers; ONLY Torus C. directly under the area of the DW leak due to the energy being distributed directly to theTorus water in that area; BOTH Drywell and Torus D. directly under the area of the DW leak due to the energy being distributed directly to theTorus water in that area; ONLY Torus Wednesday, January 21, 2015 2:26:55 PM 172 ILT-09 NRC EXAM 5 4.295028G2.1.25 001/04401B11/200.002.A.02/MOD/P-AB/BOTH/295028G2.1.25/1/1/H/2/JSC/ELJUnit 2 has experienced a LOCA. At 10:00, the following conditions exist:oRTD 2T47-N001A 300°FoRTD 2T47-N001K 275°FoRPV pressure 1000 psig and steadyo2B21-R623B, Wide Range (compensated) -90 inches Based on the above conditions, and IAW 34AB-B21-002-2, RPV Water Level Corrections, Corrected RWL is __________ . If reactor pressure decreases to 300 psig within the next fifteen (15) minutes, 2B21-R623B, __________ can be used for accurate RWL indication.Reference Provided -99 inches; Fuel Zone ONLY -99 inches; Wide Range AND Fuel Zone -102 inches; Fuel Zone ONLY -102 inches; Wide Range AND Fuel Zone A.B.C.D.Wednesday, January 21, 2015 2:26:55 PM 173 ILT-09 NRC EXAMDescription:Wide Range Yarway (Compensated or Uncompensated) Instrumentation is required for levelmonitoring AND Bulk Average Drywell temperature is greater than 150°F. In order to use thewide range instruments, EOP cautions 1 and 2 must be considered. The wide range instrumentscan not be used if they are showing erratic behavior or they do not meet the Minimum IndicatedLevel when using the Maximum Run Temperature (highest RTD in group). To determine theRWL, the applicant must determine the max temp for the instrument and subtract the correctionfactor if temp >165°F using attachment 4 of 34AB-B21-002-2.Caution from 34AB-B21-002-2:

2B21-LI-R604A (2B21-LI-R604B) and 2B21-LR-R623A (2B21-LR-R623B) (Wide Range Signals) cannot be used to determine RPV water level during rapid RPVdepressurization below 500 psig.K/A JUSTIFICATION:This question satisfies the K/A statement by requiring the applicant to know how to determineRWL based on high DW temperature IAW 34AB-B21-002-2, RPV Water Level Corrections.The applicant must understand how to interpret the RWL wide range compensation graph

.The "A" distractor is plausible if the applicant uses the wrong RTD group to determinecorrection factor from graph. The applicant's mistake would be using -9 inches correction instead of the required -12 inches correction. The second part is plausible since it is correct.The "B" distractor is plausible if the applicant uses the wrong RTD group to determinecorrection factor from graph. The applicant's mistake would be using -9 inches correctioninstead of the required -12 inches correction. The second part is plausible if the applicant doesnot understand that a rapid depressurization (700 psig in 15 minutes) below 500 psig would cause erratic behavior of the wide range instrument.The "D" distractor is plausible since the first part is correct. The second part is plausible if theapplicant does not understand that a rapid depressurization (700 psig in 15 minutes) below 500psig would cause erratic behavior of the wide range instrument.A. Incorrect - See description above. B. Incorrect - See description above. C. Correct - See description above. D. Incorrect - See description above.

References:

34AB-B21-002-2, RPV Water Level Corrections, Att. 1 & Att. 4. REMOVE CAUTION FROM BOTTOM OF ATT.1, PAGE 3.Wednesday, January 21, 2015 2:26:55 PM 174 ILT-09 NRC EXAMK/A:295028 High Drywell TemperatureG2.1.25 Ability to interpret reference materials, such as graphs, curves, tables, etc. (CFR: 41.10 / 43.5 / 45.12) . . . . . . . . . . . . . . . . . 3.9 4.2LESSON PLAN/OBJECTIVE:B11-RXINS-LP-04404, Reactor Vessel Instrumentation, Ver 7.0, EO 200.002.A.02 References used to develop this question:34AB-B21-002-2, RPV Water Level Corrections, Ver 6.15Modified from HLT Database Q#295028G4.03-001 Original QuestionUnit 2 has experienced a LOCA.Plant conditions:oRTD 2T47-N001A ......................................................... 305°FoRTD 2T47-N001K ......................................................... 252°FoReactor pressure ............................................................. 700 psi and steadyo2B21-R623A, Wide Range (compensated) .................... (-)60 inchesBased on the above plant conditions.Which one of the choices below correctly complete the following statement?IAW 34AB-B21-002-2, "RPV Water Level Corrections," and using 2B21-R623A, "Wide Range RWL" indication, actual Reactor Water Level is determined to be _______.REFERENCE PROVIDED (34AB-B21-002-2, RPV Water Level Corrections, Attachment 1( page 1 and 2 of 3) RTD Group assignments and first half of Caution 1, and Attachment4 (page 1 of 1) Temperature correction graph,) A. (-)53 inches B. (-)60 inches C. (-)67 inches D. (-)71 inches Wednesday, January 21, 2015 2:26:55 PM 175 ILT-09 NRC EXAM 5 5.295030EA1.05 001/20310PC/201.075.A.11/MOD/P-EOP/BOTH/295030EA1.05/1/1/H/3/ARB/ELJUnit 1 was scrammed from 80% RTP. oRPV pressure is being controlled by LLSoRWL is -50 inches and steadyoHPCI is the ONLY high pressure injection sourceoTorus level is 116 inches and decreasing at 2 inches/minuteIAW 31EO-EOP-012-1, Primary Containment Control, One (1) minute later, the HPCI System is __________ . Ten (10) minutes later, the Ring Header DOWNCOMER openings are __________ . REQUIRED to have been manually shutdown; COVERED REQUIRED to have been manually shutdown; UNCOVERED ALLOWED to continue injecting into the RPV; COVERED ALLOWED to continue injecting into the RPV; UNCOVERED A.B.C.D.Description:The torus level needs to be maintained above the discharge of the HPCI steam turbine exhaustline to ensure adequate steam condensing. This precludes possible primary containment failuredue to over pressurization caused by HPCI steam exhaust discharging directly into the torus airspace. Operation of the HPCI System with its exhaust discharge line (located at 115 inches withUnit 2 at 110 inches) not submerged will directly pressurize the torus air space. HPCI operation is therefore secured, and prevented from restarting, to preclude the occurrence of this condition. NO instruction regarding RCIC operation is included in this step (or in an equivalent step) fortwo reasons: The exhaust flow rate of RCIC is approximately equal to that of decay heat, and is thus consistent with the basis used for determining the Primary Containment Pressure Limit.

Elevated torus pressure will cause the RCIC turbine to trip much sooner than the HPCIturbine. Therefore the Unit 1 HPCI System will NOT be injecting when Torus water level drops below 115 inches. The RPV is not permitted to remain at pressure if suppression of steam discharged from the RPVinto the drywell cannot be assured. When the downcomer vent openings are not adequatelysubmerged (Unit 1 is 102 inches and Unit 2 is 98 inches), any steam discharged from the RPV into the drywell may not condense in the torus before torus pressure reaches unacceptable levels. Wednesday, January 21, 2015 2:26:55 PM 177 ILT-09 NRC EXAMEmergency RPV depressurization will be required at or before the point at which this low waterlevel condition occurs.

Torus water level at which the SRV T-Quenchers are uncovered is 63.0 inches.K/A JUSTIFICATION:This question satisfies the K/A statement by requiring the applicant to know the operation ofHPCI with Torus water level decreasing and then securing HPCI when Torus level can NOT bemaintained >115 inches. The "A" distractor is plausible since the first part is correct. The second part is plausible if theapplicant thinks about the torus level at which the SRV T-Quenchers become uncovered (63.0inches) and thinks this is the value for the Downcomers becoming uncovered (102 inches). The "C" distractor is plausible if the applicant thinks about 110 inches on Unit 1 instead of thaton Unit 2 which has a value of 115 inches and thinks one minute later HPCI will still beinjecting. Unit Difference. The second part is plausible if the applicant thinks about the torus level at which the SRV T-Quenchers become uncovered (63.0 inches) and thinks this is the valuefor the Downcomers becoming uncovered (102 inches). The "D" distractor is plausibleif the applicant thinks about 110 inches on Unit 1 instead of thaton Unit 2 which has a value of 115 inches and thinks one minute later HPCI will still beinjecting. Unit Difference. The second part is plausible since it is correct. A. Incorrect - See description above. B. Correct - See description above. C. Incorrect - See description above. D. Incorrect - See description above.

References:

NONEK/A:295030 Low Suppression Pool Water Level EA1. Ability to operate and/or monitor the following as they apply to LOWSUPPRESSION POOL WATER LEVEL: (CFR: 41.7 / 45.6) EA1.05 HPCI . . . . . . . . . . . . . . . . 3.5 3.5 REPLACED THE BELOW K/A AFTER PHONE CONVERSATION WITH CHIEFWednesday, January 21, 2015 2:26:55 PM 178 ILT-09 NRC EXAMEA1. Ability to operate and/or monitor the following as they apply to LOWSUPPRESSION POOL WATER LEVEL:

(CFR: 41.7 / 45.6)EA1.03 HPCS: Plant-Specific . . . . . . . . . . . . . . . . 3.4 3.4LESSON PLAN/OBJECTIVE:EOP-PC-LP-20310, Primary Containment Control (PC), Ver. 3.0, EO 201.075.A.11 & EO 201.075.A.13 References used to develop this question:31EO-EOP-012-1, PC Primary Containment Control, Ver. 6.0 31EO-EOP-012-2, PC Primary Containment Control, Ver. 6.0 31EO-OPS-001-0, EOP General Information, Ver 1.8Modified from HLT Database Q#295030EA1.02-001 Original QuestionUnit 1 was scrammed from 80% power.

  • RPV pressure is being controlled by LLS
  • RWL is -153 inches and steady
  • RCIC is the ONLY high pressure injection source
  • Torus level is 114 inches and decreasing at 2 inches a minuteIAW 31EO-EOP-012-1, Primary Containment Control, which ONE of the choices belowcompletes these statements? The RCIC System __________ required to be shutdown/tripped. The HIGHEST Torus water level at which an Emergency Depressurization is required, isBEFORE Torus level reaches __________ . A. is NOT; 102 inches B. is NOT; 98 inches C. is; 102 inches D. is; Wednesday, January 21, 2015 2:26:55 PM 179 ILT-09 NRC EXAM98 inches Wednesday, January 21, 2015 2:26:55 PM 180 ILT-09 NRC EXAM 5 6.295031EK2.09 001/00401B31/004.003.A.02/MOD/SYS-I/BOTH/295031EK2.09/1/1/H/3/JSC/ELJUnit 2 is at 60% RTP with the following conditions:o2A RFP is in AUTOMATICo2B RFP has just been transferred to the M/A StationSubsequently, RFP 2A experiences a malfunction that results in RFP 2A speed lowering to 2100 rpm and stabilizes. oRWL is recovered to normal using the RFP 2B oLOWEST RWL reached was -8 inches With the above conditions,Of the listed RWL values, the HIGHEST RWL at which a Recirc Pump runback signal will FIRST be generated is __________ . The FINAL Recirc Pump speed will be __________ . 29 inches; 22% 29 inches; 33% 31 inches; 22% 31 inches; 33% A.B.C.D.Description:Wednesday, January 21, 2015 2:26:55 PM 181 ILT-09 NRC EXAMA #4 runback occurs due to RWL lowering to < 30 inches and the runback will cause recirc tolower to 33% speed since RWL lowers to < 20 inches. The #1 runback will occur since RWL lowered to < 3 inches which caused the plant to scram.This satisfied the requirements for total F/W flow to be < 20%. The #1 runback will occur alsodue to RWL < 20 inches and total steam flow < 60% of previous 6 min average. The #2 runback will not occur since neither RFP has a trip signal from TMR.K/A JUSTIFICATION:This question satisfies the K/A statement by requiring the applicant to know Recirc Runbackslogic. Three of the four runback signals use low RWL as an input. The "B" distractor is plausible since the first part is correct. The second part is plausible sincethe conditions are met for a #4 runback to occur and the Recirc pumps would lower to 33%however a #1 runback is more limiting. The "C" distractor is plausible if the applicant thinks that a #2 runback conditions have been met.All conditions are met for a #2 runback with the exception that neither RFP has a trip signal from the TMR. The second part is plausible since it is correct.The "D" distractor is plausible if the applicant thinks that a #2 runback conditions have beenmet. All conditions are met for a #2 runback with the exception that neither RFP has a trip signalfrom the TMR. The second part is plausible since the conditions are met for a #4 runback tooccur and the Recirc pumps would lower to 33% however a #1 runback is more limiting. A. Correct - See description above. B. Incorrect - See description above. C. Incorrect - See description above. D. Incorrect - See description above. Wednesday, January 21, 2015 2:26:55 PM 182 ILT-09 NRC EXAM

References:

NONEK/A:295031 Reactor Low Water LevelEK2. Knowledge of the interrelations between REACTOR LOW WATER LEVEL and thefollowing: (CFR: 41.7 / 45.8) EK2.09 Recirculation system: Plant-Specific . . . . . . . . . . . . . . . . . 3.3 3.4LESSON PLAN/OBJECTIVE:B31-RRS-LP-00401, Reactor Recirculation System, Ver 10.5, EO 004.003.A.02 References used to develop this question:34SO-B31-001-2, Reactor Recirculation System, Ver 44.6 Modified from HLT Database Q#202002K3.03-001 which was used on HLT-7 2012-301 NRC Exam Q#3.Original QuestionUnit 2 is at 100% RTP when a malfunction causes a Recirc runback signal to Speed Limiter #1on BOTH Recirc Pumps. While the Recirc Pumps are reducing speed, INDICATED reactor water level on2C32-R606A, B & C, "Narrow Range" instruments will __________. The FINAL Recirc Pump speeds will be __________.A. decrease; 22% B. decrease; 33% C. increase; 22% D. increase; 33% Wednesday, January 21, 2015 2:26:55 PM 183 ILT-09 NRC EXAM 5 7.295032EK3.03 001/03901E51/039.012.A.02/MOD/P-EOP/BOTH/295032EK3.03/1/2/F/3/ARB/ELJUnit 2 is operating at 100% RTP when a pipe ruptures in the Torus Area. The RCIC System has automatically isolated due to high temperatures in the Torus Area. IAW 34SO-E51-001-2, Reactor Core Isolation Cooling (RCIC) System, the time delay forthe RCIC isolation is __________ . The reason for the RCIC System isolation at this setpoint is __________ . 29 minutes; because the Max Safe Operating temperature limit has been exceeded 29 minutes; to limit radioactive release to the public to within FSAR DBA assumptions 14 minutes; because the Max Safe Operating temperature limit has been exceeded 14 minutes; to limit radioactive release to the public to within FSAR DBA assumptions A.B.C.D.Description:IAW 34SO-E51-001-2, 5.2.2 The RCIC System will automatically isolate upon receipt of any of the following signals: 5.2.2.4 Steam Leak Detection System High Temperatures: Instrument Setpoint > 165°F or > 36°F diff temp for more than 29 minutes. Technical Specifications Limit < 169°F OR < 42°F diff temp.IAW TS Bases 3.3.6.1, The primary containment isolation instrumentation automaticallyinitiates closure of appropriate primary containment isolation valves (PCIVs). The function ofthe PCIVs, in combination with other accident mitigation systems, is to limit fission product release during and following postulated Design Basis Accidents (DBAs). Primary containmentisolation within the time limits specified for those isolation valves designed to closeautomatically ensures that the release of radioactive material to the environment will beconsistent with the assumptions used in the analyses for a DBA. The Max Safe Operating Temperature is defined as the highest temperature at which safeshutdown equipment will not fail NOR will personnel access required for safe shutdown beprecluded. The Max Safe Operating Temperatures are based on Georgia Power analysis forequipment qualification in high temperature environments. K/A JUSTIFICATION:This question satisfies the K/A statement by requiring the applicant to know why a systemWednesday, January 21, 2015 2:26:55 PM 184 ILT-09 NRC EXAM(RCIC) is isolated with area temperatures above the Maximum Normal Operating value. The "A" distractor is plausible since the first part is correct. The second part is plausible if theapplicant thinks about the Maximum Safe Operating temperatures and they are based uponGeorgia Power EQ analysis and thinks that this is why the system is isolated. The "C" distractor is plausible if the applicant remembers the HPCI system Steam LeakDetection System High Temperature isolation occurs after a 14 minute time delay. The second part is plausible if the applicant thinks about the Maximum Safe Operating temperatures andthey are based upon Georgia Power EQ analysis and thinks that this is why the system isisolated.The "D" distractor is plausible if the applicant remembers the HPCI system Steam LeakDetection System High Temperature isolation occurs after a 14 minute time delay. The secondpart is plausible since it is correct.A. Incorrect - See description above. B. Correct - See description above. C. Incorrect - See description above. D. Incorrect - See description above. Wednesday, January 21, 2015 2:26:55 PM 185 ILT-09 NRC EXAM

References:

NONEK/A:295032 High Secondary Containment Area TemperatureEK3. Knowledge of the reasons for the following responses as they apply to HIGHSECONDARY CONTAINMENT AREA TEMPERATURE : (CFR: 41.5 / 45.6)EK3.03 Isolating affected systems . . . . . . . . . . . . . . . . . 3.8 3.9*LESSON PLAN/OBJECTIVE:E51-RCIC-LP-03901, Reactor Core Isolation Cooling (RCIC), Ver 6.1, EO 039.012.A.02 References used to develop this question:34SO-E41-001-2, High Pressure Coolant Injection (HPCI) System, Ver 28.3 34SO-E51-001-2, Reactor Core Isolation Cooling (RCIC) System, Ver 25.1Modified from question used on DAEC 2009 NRC Exam Q#65. Original Question65.What is the reason for automatic closure of the RWCU Primary Containment IsolationValves if a RWCU area high temperature were to occur.What is the reason for this requirement?A. To ensure that the release of radioactive material to the environment will be consistentwith the assumptions used in the final safety analyses.B. To minimize moisture buildup and overheating in the Standby Gas Treatment Systemcharcoal beds.C. To prevent exceeding the Environmental Qualification temperature limits on theelectrical buses in the Turbine Building required for safe shutdown.D. To ensure operator access to secondary containment for event mitigation actions.Wednesday, January 21, 2015 2:26:55 PM 186 ILT-09 NRC EXAM 5 8.295033EK2.02 001/03701Z41/037.008.A.02/MOD/SYS-B/BOTH/295033EK2.02/1/2/F/3/JSC/ELJMaintenance is in the process of removing the Unit 2 Steam Dryer from the RPV when theSteam Dryer momentarily becomes partially uncovered. oContamination does not become Airborne as a result of the momentarily uncovering of theSteam DryerALL Unit 2 Refueling Floor Area Radiation Monitors (ARMs) increase to 17 mr/hr beforelowering to their pre-event value. With the above conditions and NO operator actions, The Unit 2 SBGT System fans will __________ . The Main Control Room Environmental Control (MCREC) System will __________ . have automatically started; have aligned to Pressurization Mode have automatically started; remain in the Standby Mode remain in Standby; have aligned to Pressurization Mode remain in Standby; remain in the Standby Mode A.B.C.D.Description:Phil, this was question 4 of 10 of the previously submitted questions. Changes wereincorporated based on your ES-401-9 comments. Any of the following signals for Unit 1 or Unit 2 will initiate all four SBGT Trains:Unit 1 or 2 Reactor Zone exhaust high radiation: - Unit 1: 18 mrem/hr on 1D11-K609 A-D - Unit 2: 18 mrem/hr on 2D11-K609 A-DUnit 1 or 2 Refueling Zone exhaust high radiation: - Unit 1 18 mrem/hr on 1D11-K611-A-D - Unit 2 18 mrem/hr on 2D11-K611 A-D, OR 6.9 mrem/hr on 2D11-K634 A-D , OR 5.7 mrem/hr on 2D11-K635 A-D.High Drywell pressure (1.85 psig).

Low Reactor water level (-35 inches)These radiation monitors are grouped such that it requires an "A" and a "B" Reactor Buildinghigh radiation monitor reaching a high radiation setpoint to start the "A" SBGT. "C" and "D"monitors would have to actuate to start the "B" SBGT. The Unit 2 Refueling Floor highradiation logic uses 12 detectors consisting of the K611's, K634's, and K635's. To actuate "A"Wednesday, January 21, 2015 2:26:55 PM 187 ILT-09 NRC EXAMSBGT a K611A and K634B reaching a high radiation setpoint and would auto start SBGT "A". SBGT will not start in this situation given the Steam Dryer momentarily becomes partiallyexposed and will remain wet therefore not allowing ventilation detectors to be directly affected.With the steam dryer being wet, it will remain as a point source vice allowing airborne particlesto be moved through the ventilation system. The Control Room Ventilation System will automatically switch to the Pressurization Mode, inorder to protect Control Room personnel, on any of the following signals: LOCA signal from Unit 1 or Unit 2 RF Area High Radiation (ARM) from Unit 1 or Unit 2 (15 mr/hr)

Main Steam Line High Flow from Unit 1 or Unit 2 Main Control Room Air Intake High RadiationK/A JUSTIFICATION:This question satisfies the K/A statement by requiring the applicant to evaluate which systems are actuated automatically from ARMs and/or PRMs (interrelations). The MCREC system willautomatically shift to the pressurization mode due to the elevated area radiation levels on the Refuel Floor but SBGT will automatically initiate on process radiation monitors. The "A" distractor is plausible if the applicant thinks SBGT will start on Area Radiation Monitor(ARM) signal on the refuel floor vice the ventilation radiation detectors. If the process radiationmonitors indicated 16-17 mr/hr they will have exceeded 2D11-K634 and K635 setpoints for Unit 2 SBGT start signals. The applicant may think that these instruments are in the same vicinitytherefore thinking that SBGT will start. The second part is plausible since it is correct.The "B" distractor is plausible if the applicant thinks SBGT will start on Area Radiation Monitor(ARM) signal on the refuel floor vice the ventilation radiation detectors. If the process radiationmonitors indicated 16-17 mr/hr they will have exceeded 2D11-K634 and K635 setpoints for Unit 2 SBGT start signals. The applicant may think that these instruments are in the same vicinitytherefore thinking that SBGT will start. The second part is plausible if the applicant thinks theonly radiation signal that will cause MCREC to shift to the pressurization mode is the Main Control Room air intake high radiation (0.9 mrem/hr). Based on this reasoning, since theradiation condition is limited to the Refuel Floor, MCREC would remain is the Standby Mode. Also plausible if the applicant thinks the signal to start MCREC comes from the same signal that causes secondary containment to isolate. The "D" distractor is plausible since the first part is correct. The second part is plausible if theapplicant thinks the only radiation signal that will cause MCREC to shift to the pressurizationmode is the Main Control Room air intake high radiation (0.9 mrem/hr). Based on thisreasoning, since the radiation condition is limited to the Refuel Floor, MCREC would remain is the Standby Mode. Also plausible if the applicant thinks the signal to start MCREC comes fromthe same signal that causes secondary containment to isolate. A. Incorrect - See description above. B. Incorrect - See description above. Wednesday, January 21, 2015 2:26:55 PM 188 ILT-09 NRC EXAMC. Correct - See description above. D. Incorrect - See description above.

References:

NONEK/A:295033 High Secondary Containment Area Radiation Levels EK2. Knowledge of the interrelations between HIGH SECONDARY CONTAINMENTAREA RADIATION LEVELS and the following: (CFR: 41.7 / 45.8) EK2.02 Process radiation monitoring system . . . . . . . . . . . . . . . . . . . 3.8 4.1 LESSON PLAN/OBJECTIVE:Z41-MCREC-LP-03701, Main Control Room Environmental Control System, Ver 6.0 , EO 037.008.A.02 References used to develop this question:34SO-Z41-001-1, Control Room Ventilation System, Ver 22.234SO-T46-001-2, Standby Gas Treatment System, Ver 14.14Modified from HLT Database Q#295023AA1.04-001 which was used on HLT-7 2012-301 NRC Exam Q#49 Original QuestionA refueling accident occurs on Unit 2. The following Area Radiation Monitors (ARM) red Trip Lights illuminate at Control Room ARM Panel 2D21-P600: o2D21-K601A, Reactor head laydown area o2D21-K601M, Spent Fuel Pool & New StorageWhich ONE of the following predicts how the Main Control Room Environmental Control (MCREC) system is affected and operation of the ARMs? With the above conditions, the MCREC system will __________ . Wednesday, January 21, 2015 2:26:55 PM 189 ILT-09 NRC EXAMTrip setpoints, the ARM red TRIP lights __________ reset. A. remain in the Normal Mode; must be manually B. remain in the Normal Mode; will automatically C. align to the Pressurization Mode; must be manually D. align to the Pressurization Mode; will automatically Wednesday, January 21, 2015 2:26:55 PM 190 ILT-09 NRC EXAM 5 9.295036EA1.04 001/20325SCRR/201.093.A.01/MOD/P-EOP/BOTH/295036EA1.04/1/2/H/3/ARB/ELJUnit 2 is operating at 100% RTP when a leak occurs in Secondary Containment (SC) requiringentry into 31EO-EOP-014-2, SC Secondary Containment Control/ RR Radioactivity Release. Subsequently, a loss of 2R25-S064, Instrument Bus 2A, occurs. The Shift Supervisor directs the NPO to monitor SC water and radiation levels. SC radiation levels _________ be monitored by using area radiation monitoring (ARM)instrumentation located in the Main Control Room. Personnel _________ to determine that Max Normal SC Water level has been exceeded. can NOT; can use the Main Control Room SC sump alarms by themselves can NOT; must be dispatched LOCALLY can; can use the Main Control Room SC sump alarms by themselves can; must be dispatched LOCALLY A.B.C.D.Wednesday, January 21, 2015 2:26:55 PM 191 ILT-09 NRC EXAMDescription:The Secondary Containment Radiation monitors are powered from 2R25-S064, Instrument Bus 2A. Since this bus is de-energized, radiation levels can NOT be monitored from the ControlRoom ARM panel (Breaker #9). A recent change to the Annunciator Response Procedures associated with SecondaryContainments water level, requires local observation of water level for determination of MaxNormal levels. K/A JUSTIFICATION:This question satisfies the K/A statement by requiring the applicant to determine if radiationlevels can be monitored from the Main Control Room and if high area sump level annunciationcan be used in the Main Control Room for determining if Max Normal SC Water levels have been exceeded.The "A" distractor is plausible since the first part is correct. The second is plausible if theapplicant remembers that the Maximum Normal water levels could be determined using sumplevel annunciators on the 2H11-P657 however SC water levels have been changed to 4 inches above floor level. The annunciators will still be alarm however level may not reach Max Normallevels. The "C" distractor is plausible if the applicant thinks the power supply is Instrument Bus 2B viceInstrument Bus 2A for the ARM panel in the Control Room. The second is plausible if theapplicant remembers that the Maximum Normal water levels could be determined using sump level annunciators on the 2H11-P657 however SC water levels have been changed to 4 inchesabove floor level. The annunciators will still be alarm however level may not reach Max Normallevels. The "D" distractor is plausible if the applicant thinks the power supply is Instrument Bus 2Bvice Instrument Bus 2A for the ARM panel in the Control Room. The second part is plausible since it is correct.A. Incorrect - See description above. B. Correct - See description above. C. Incorrect - See description above. D. Incorrect - See description above.

References:

NONEK/A:Wednesday, January 21, 2015 2:26:55 PM 192 ILT-09 NRC EXAM295036 Secondary Containment High Sump/Area Water LevelEA1. Ability to operate and/or monitor the following as they apply to SECONDARYCONTAINMENT HIGH SUMP/AREA WATER LEVEL : (CFR: 41.7 / 45.6) EA1.04 Radiation monitoring: Plant-Specific . . . . . . . . . . . . . . . . . 3.1 3.4LESSON PLAN/OBJECTIVE:EOP-SCRR-LP-20325, Secondary Containment / Radioactivity Release Control, EO 201.093.A.01 Reference(s) used to develop this question:A-20181, Plant Hatch Load List, 2R25 S064, Ver. 39.0 A-20182, Plant Hatch Load List, 2R25 S065, Ver. 28.0 31EO-EOP-014-2, SC Secondary Containment Control/ RR Radioactivity Release, Ver. 11.0 Modified from HLT Database Q#295036EA1.04-001 which was used on Hatch ILT-6 2011-301 NRC Exam Q#59. Original QuestionUnit 2 is operating at 100% RTP when a leak occurs in Secondary Containment (SC) requiringentry into 31EO-EOP-014-2, SC Secondary Containment Control/ RR Radioactivity Release. Subsequently, a loss of Instrument Bus 2A occurs.

The Shift Supervisor directs the NPO to monitor SC water and radiation levels. Which ONE of the choices below completes the following statements? SC radiation levels _________ be monitored by using area radiation monitoring (ARM)instrumentation located in the Main Control Room. Personnel _________ to determine that Max Normal SC Water level has been exceeded. A. can NOT; can use the Main Control Room SC sump alarms by themselves B. can NOT; must be dispatched LOCALLY C. can; can use the Main Control Room SC sump alarms by themselves Wednesday, January 21, 2015 2:26:55 PM 193 D. ILT-09 NRC EXAMcan; must be dispatched LOCALLY Wednesday, January 21, 2015 2:26:55 PM 194 ILT-09 NRC EXAM 6 0.295037EK3.03 001/20327CP3/201.089.A.02/MOD/P-EOP/BOTH/295037EK3.03/1/1/H/2/JSC/ELJUnit 2 was operating at 100% RTP when an ATWS occurred. RC-1 actions are completed.Reactor power stabilizes at 8% RTP. Based on the above conditions and IAW 34AB-C71-001-2, Scram Procedure, the Recirc pumps will be __________ .The reason RWL will be intentionally lowered is to __________ .tripped; DECREASE core inlet subcooling by uncovering feedwater spargerstripped; INCREASE void fraction inside the core shroudoperating at minimum speed; DECREASE core inlet subcooling by uncovering feedwater spargersoperating at minimum speed; INCREASE void fraction inside the core shroud A.B.C.D.Description:Wednesday, January 21, 2015 2:26:55 PM 195 ILT-09 NRC EXAMTo prevent or mitigate the consequences of any large irregular neutron flux oscillations induced by neutronic/thermal-hydraulic instabilities, RWL is lowered sufficiently below the elevation ofthe feedwater sparger nozzles. This places the feedwater spargers in the steam space providingeffective heating of the relatively cold feedwater and eliminating the potential for high core inletsubcooling. K/A JUSTIFICATION:This question satisfies the K/A statement by requiring the applicant to know the basis forlowering RWL during an ATWS. The "B" distractor is plausible since the first part is correct. The second part is plausible sincelowering RWL will cause the void fraction to increase inside the shroud however as steamproduction is reduced (power lowering due to effective heating of the relatively cold feedwaterand eliminating the potential for high core inlet subcooling) void fraction will decrease back toits original value.The "C" distractor is plausible since RC-1 has the step to place the recirc pumps at minimumspeed if they are not tripped. The second part is plausible since it is correct.The "D" distractor is plausible since RC-1 has the step to place the recirc pumps at minimumspeed if they are not tripped. The second part is plausible since lowering RWL will cause thevoid fraction to increase inside the shroud however as steam production is reduced (powerlowering due to effective heating of the relatively cold feedwater and eliminating the potentialfor high core inlet subcooling) void fraction will decrease back to its original value.A. Correct - See description above. B. Incorrect - See description above. C. Incorrect - See description above. D. Incorrect - See description above.

References:

NONEK/A:295037 SCRAM Condition Present and Reactor Power Above APRM Downscale orUnknown EK3. Knowledge of the reasons for the following responses as they apply to SCRAMWednesday, January 21, 2015 2:26:55 PM 196 ILT-09 SRO NRC EXAMCONDITION PRESENT AND REACTOR POWER ABOVE APRM DOWNSCALE ORUNKNOWN : (CFR: 41.5 / 45.6)

EK3.03 Lowering reactor water level . . . . . . . . . . . . . . . . . . . . . . . . 4.1* 4.5*LESSON PLAN/OBJECTIVE:EOP-CP3-LP-20327, Level Power Control (CP-3), Ver 4.0, EO 201.089.A.02 References used to develop this question:31EO-EOP-017-2, ATWS Level Control, Ver 1334AB-C71-001-2, Scram Procedure, Ver 12.4 Modified from HLT Database Q#G2.4.9-001 Original QuestionWhich ONE of the choices below completes the following statements concerning an ATWScondition on Unit 1? IAW CP-3, the LOWEST listed Reactor power that would require RWL to be intentionallylowered is __________ RTP.

The basis for the EOP step below (lowering RWL) is to __________ . A. 8%; DECREASE core inlet subcooling of the incoming feedwater B. 8%; INCREASE void fraction inside the core shroud C. 6%; DECREASE core inlet subcooling of the incoming feedwater D. 6%; INCREASE void fraction inside the core shroud Wednesday, January 21, 2015 2:26:55 PM 197 ILT-09 NRC EXAM 6 1.295038EA1.01 001/10007D11/200.030.A.08/MOD/P-EP/BOTH/295038EA1.01/1/1/H/3/ARB/ELJThe Unit 1 SS suspects an abnormal offsite radiation release is occurring.The following Main Stack annunciators are illuminated:oOFF GAS VENT RADIATION HIGH-HIGH (601-412)oOFF GAS VENT RADIATION HIGH (601-418)The SS has directed 73EP-EIP-018-0, Prompt Offsite Dose Assessment (PODA), to be performed.The current normal average daily site dose rate is 1.0 E-3 mR/hr.The result from the PODA is a peak TEDE dose rate of 1.0 E-1 mR/hr.Based on the above conditions,The Main Stack Effluent Accident Range Gas Monitors (KAMANs) __________ . IAW 73EP-EIP-018-0, a radioactive release from the Main Stack __________ in progress.have automatically started;is have automatically started;is NOT are in standby;isare in standby;is NOT A.B.C.D.Description:Radiation monitors are used to monitor, indicate, record, and annunciate radiation levels atvarious points from gasses flowing through the system. The following areas are sampled: TwoPre-Treatment Monitors (gamma compensated Ion Chambers) sample just prior to the holdupvolume. Two Post-Treatment Monitors (Scintillation detectors) sample from the discharge ofthe first carbon bed in each train and just prior to the after filters. Two Stack Monitors(Scintillation detectors) sample the Off-Gas flow going up the stack and will activate the MainStack Effluent Accident Range Gas Monitor on a Hi-Hi signal. The sample probes are arranged across the stack to ensure a representative sample is obtained. Two Carbon vault monitors(Geiger-Mueller) sample for buildup of radiation due to long-lived particulates. Any combination of Inoperable, Downscale, or Hi-Hi-Hi radiation signals, simultaneously inboth trip channels of the Post-Treatment Monitors, will cause the Main Stack Inlet Valve2N62-F057 to prevent additional release. Cooler Condenser and Moisture Separator,2N62-F030A/B, and Holdup volume 2N62-F085 Loop Seal Drain valves close to prevent anWednesday, January 21, 2015 2:26:55 PM 198 ILT-09 NRC EXAMinadvertent release, since these valves drain to Radwaste instead of the Main Condenser. There are two gamma sensitive Scintillation detectors in the Main Stack Radiation MonitoringSystem. A gas sample is drawn through an Isokinetic Probe which is located high enough in themain stack vent stream to assure representative sampling. The sample passes through twoshielded chambers where the radiation level of the vent gas is measured. The detectors havethree alarms: Downscale-loss of power causes alarm, Hi-alarm only, and Hi-Hi - starts MainStack Kaman and isolates normal Main Stack Radiation monitors.IAW 73EP-EIP-018-0, Prompt Offsite Dose Assessment:NOTE:The current calculated daily average site dose rate is E-03 mR/hr19.IF the peak TEDE dose rate (mR/hr) value is an order of magnitude (10 times) higher thanthe current calculated daily average AND an emergency has been declared, THEN notify theEmergency Director a radioactive release is in progress.20.IF the peak TEDE Dose Rate value exceeds 0.057 mR/hr (5.7 E-02 mR/hr), THEN notify theEmergency Director for possible emergency classification declaration or upgrade AND notify theaffected Unit Shift Supervisor for possible EOP Actions.K/A JUSTIFICATION:This question satisfies the K/A statement by requiring the applicant to monitor the Main Stackgas monitoring system by observing the Pre-Treatment & Post-Treatment Radiation Monitorsand based on the associated release rate alarm determine if the Main Stack has been isolated from Unit 2, thereby reducing the high off-site release rate. The "B" distractor is plausible since the first part is correct. The second part is plausible if theapplicant remembers the value of 0.57 mrem/hr associated with the RR-Radioactivity ReleaseControl flowchart (entry condition) and determines 0.1 mrem/hr is less than the entry condition.

A radioactive release occurs at 10 X normal values therefore would be occurring at 0.01mrem/hr.The "C" distractor is plausible if the applicant remembers that the Kamans do not automaticallyshutdown and thinks that they must be manually started. The applicant could also think about theoperation of the Post Treatment Monitors and think that since they need a Hi-Hi-Hi alarm to cause an isolation that the Kamans need a Hi-Hi-Hi alarm also. The second part is plausiblesince it is correct.The "D" distractor is plausible if the applicant remembers that the Kamans do not automaticallyshutdown and thinks that they must be manually started. The applicant could also think about theoperation of the Post Treatment Monitors and think that since they need a Hi-Hi-Hi alarm to cause an isolation that the Kamans need a Hi-Hi-Hi alarm also. The second part is plausible ifthe applicant remembers the value of 0.57 mrem/hr associated with the RR-RadioactivityRelease Control flowchart (entry condition) and determines 0.1 mrem/hr is less than the entry condition. A radioactive release occurs at 10 X normal values therefore would be occurring at0.01 mrem/hr.Wednesday, January 21, 2015 2:26:56 PM 199 ILT-09 NRC EXAMA. Correct - See description above. B. Incorrect - See description above.

C. Incorrect - See description above.

D. Incorrect - See description above.

Wednesday, January 21, 2015 2:26:56 PM 200 ILT-09 NRC EXAM

References:

NONEK/A:295038 High Off-Site Release RateEA1. Ability to operate and/or monitor the following as they apply to HIGH OFF-SITERELEASE RATE : (CFR: 41.7 / 45.6) EA1.01 Stack-gas monitoring system: Plant-Specific . . . . . . . . . . 3.9 4.2LESSON PLAN/OBJECTIVE:D11-PRM-LP-10007, Process Radiation Monitoring, Ver 5.0, EO 200.030.A.08References used to develop this question:NMP-EP-104-F07, Offsite Dose Assessment Hatch Prompt Offsite Dose Assessment, Ver 1.034AR-601-412-1, OFF GAS VENT RADIATION HIGH-HIGH, Ver 4.434AR-601-418-1, OFF GAS VENT RADIATION HIGH, Ver 3.334SO-N62-001-1, Off Gas System, Ver. 19.3 Modified from HLT Database Q#LT-200030-009 Original QuestionThe Main Stack Effluent Accident Range Monitors (KAMANS) will automatically start assoon as the Main Stack Rad Monitors exceed the __________ setpoint. Once the KAMANS have started, they will remain in service until __________ . A. Hi; they are manually secured B. Hi; the Main Stack Rad Monitors readings are below the trip setpoint C. Hi-Hi; they are manually secured D. Hi-Hi; the Main Stack Rad Monitors readings are below the trip setpoint Wednesday, January 21, 2015 2:26:56 PM 201 ILT-09 NRC EXAM 6 2.300000G2.2.44 001/03501P51/P70/200.025.A.02/MOD/P-AB/BOTH/300000G2.2.44/2/1/H/2/JSC/ELJUnit 2 is operating at 80% RTP when 34AB-P51-001-2, Loss of Instrument or Service AirSystem or Water Intrusion into the Service Air System, is entered due to lowering air pressure. The following conditions exist,o2P52-F565, Rx Bldg Inst N2 To Non-Int Air El. 185 Isol Vlv, Danger Tagged CLOSEDoSCRAM VLV PILOT AIR HDR PRESS HIGH/LOW (603-131) is illuminatedoCRD HYD TEMP HIGH (603-140) is illuminatedBased on the above conditions and IAW 34AB-P51-001-2, A Reactor scram __________ required to be inserted at this time.If the crosstie between Unit One and Unit Two Service Air Systems CANNOT be opened, the __________ MSIVs will drift close as air pressure continues to lower.is;Inboardis;Outboardis NOT;Inboardis NOT;Outboard A.B.C.D.Description:

The loss of air is a complex situation for the plant in that many components fail in different waysthroughout the plant. Based on the indications given, air pressure will be less than 70 psig based on the annunicator SCRAM VLV PILOT AIR HDR PRESS HIGH/LOW (603-131) setpoint isreceived when the scram air header pressure is >75 psig or <70 psig. This is confirmed due to theCRD HYD TEMP HIGH annunciator being received. The 34AB-P51-001-2 requires a scram as follows: 1)SCRAM VLV PILOT AIR HDR PRESS HIGH/LOW (603-131) COINCIDENT with CRD HYD TEMP HIGH (603-140). 2)Scram pilot valve air header pressure less than or equal to 50 PSIGas indicated locally on 2C11-R013. A loss of Instrument Air results in a difficult transient for operators even if all equipmentoperates as intended. Abnormal responses from many systems and components can occur simultaneously because a large number of components are supplied by Instrument Air. Identifying the affected components, their failure modes, and the resultant effect on systemoperation and system interactions is a complicated task. Valves may fail closed, open, or as-is;Wednesday, January 21, 2015 2:26:56 PM 202 ILT-09 NRC EXAMcontrollers may fail with a maximum or minimum demand signal or may lockup with thepre-event demand output.

Although most components are designed to fail in a safe position,some manual actions may be required to override or bypass component failures in order tominimize the severity of the subsequent transients.

A gradual loss of Instrument Air is also a difficult transient for operators. In that condition,components fail in a random sequence, depending on the rate of air pressure decrease in variousportions of the system and the different pressure requirements for operating individualcomponents. The random sequence of failures makes it more difficult for the operator to identifyand diagnose the problems. Depending on the particular failure sequence, the type and severityof subsequent plant transients will vary in a non-predictable way. Symptoms and Indications of an Instrument Air Failure in excess of Nitrogen Backupcapabilities (Nitrogen backup valve 2P52-F565 is dangered tagged out). There are many othersymptoms and indications but only one discussed here. Outboard Main Steam Isolation Valvesdrift closed (Inboard MSIVs are supplied by Drywell Pneumatic System).K/A JUSTIFICATION:This question satisfies the K/A statement by requiring the applicant to be able to diagnose theresultant effect on the operation of the affected unit (MSIVs) with the inability to open the crossconnect between the units.The "A" distractor is plausible since the first part is correct. The second part is plausible if theapplicant thinks that the Inboard MSIVs are supplied with Plant Air vice Drywell Pneumatics.The "C" distractor is plausible if the applicant thinks that along with the current annunciatorsthat Scram Pilot Valve Air Header pressure less than or equal to 50 PSIG is required to also exist concurrently before the plant is required to be scrammed. The second part is plausible if theapplicant thinks that the Inboard MSIVs are supplied with Plant Air vice Drywell Pneumatics.The "D" distractor is plausible if the applicant thinks that along with the current annunciatorsthat Scram Pilot Valve Air Header pressure less than or equal to 50 PSIG is required to also existconcurrently before the plant is required to be scrammed. The second part is plausible since it iscorrect.A. Incorrect - See description above. B. Correct - See description above. C. Incorrect - See description above. D. Incorrect - See description above.

References:

NONEWednesday, January 21, 2015 2:26:56 PM 203 ILT-09 NRC EXAMK/A:300000 Instrument Air System (IAS)G2.2.44 Ability to interpret control room indications to verify the status and operation of asystem, and understand how operator actions and directives affect plant and system conditions. (CFR: 41.5 / 43.5 / 45.12) . . . . . . . . . . 4.2 4.4REPLACED THE BELOW K/A AFTER PHONE CONVERSATION WITH CHIEFEXAMINER PHIL CAPEHART ON 3/27/2014.G2.2.42 Ability to recognize system parameters that are entry-level conditions forTechnical Specifications. (CFR: 41.7 / 41.10 / 43.2 / 43.3 / 45.3) . . . . . . . . 3.9 4.6LESSON PLAN/OBJECTIVE:P51-P52-P70-Plant Air-LP-03501, Plant Air Systems, Ver 3.0, EO 200.025.A.02References used to develop this question:34AB-P51-001-2, Loss of Instrument or Service Air System or Water Intrusion into the ServiceAir System, Ver 4.9Modified from HLT Database which was used on HLT-4 NRC Exam Q#47 Original QuestionUnit 2 is operating at 100% power.o2P52-F565, "Rx Bldg Inst N2 To Non-Int Air El 185 Isol Vlv", has been tagged in theclosed position.oUnit 2 experiences a loss of all Unit 2 Station Service Air Compressors.oThe air cross-tie valve between Unit 1 and Unit 2 cannot be opened due to a bent stem.Which one of the following predicts the final MSIV positions with respect to the availability of apneumatic supply?A. The Inboard and Outboard MSIVs will remain OPEN. B. The Inboard and Outboard MSIVs will eventually drift CLOSED. C. The Inboard MSIVs will remain OPEN;The Outboard MSIVs will eventually drift CLOSED. Wednesday, January 21, 2015 2:26:56 PM 204 D. ILT-09 NRC EXAMThe Inboard MSIVs will eventually drift CLOSED;The Outboard MSIVs will remain OPEN.Wednesday, January 21, 2015 2:26:56 PM 205 ILT-09 NRC EXAM 6 3.400000K4.01 001/00901P42/200.014.A.06/BANK/SYS-I/BOTH/400000K4.01/2/1/H/3/ARB/ELJUnit 2 is operating at 30% RTP. oTwo (2) RBCCW pumps are runningoOne (1) RBCCW pump is Danger tagged out of serviceThe following sequence of events occur: o11:00 - A loss of a 600V bus results in One (1) RBCCW pump being de-energizedo11:03 - RBCCW system pressure decreases below the RBCCW pump auto start setpointo11:05 - Power is restored to the 600V busBased on the above plant conditions,After power is restored to the 600V bus, the Non-Essential Load Lockout __________REQUIRED to be depressed prior to the restart of the associated RBCCW pump. Manipulation of the RBCCW pump control switch at panel 2H11-P650 __________REQUIRED to start the RBCCW pump that was de-energized. is; is is; is NOT is NOT; is is NOT; is NOT A.B.C.D.Wednesday, January 21, 2015 2:26:56 PM 206 ILT-09 NRC EXAMDescription:

2 Pumps are powered from 600 VAC Bus "C" (R23-S003). 1 Pump is powered from 600 VACBus "D" (R23-S004). Standby Pump auto-starts if system pressure decreases to 90 psig with thepump control switch in AUTO. The RBCCW Pumps cannot be started following anundervoltage trip on the associated 600 VAC bus until the Non-essential Load LockoutPushbutton is depressed on H11-P652. This feature provides overload protection for theemergency diesel generators. Once this pushbutton is depressed, the pump control switch willbe required to be placed into the off position, the returned to auto position and then the pumpwill restart if a low pressure condition exists. If a low pressure condition does not exist, thepump can be started by placing the switch to start. Either condition requires the Non-essentialLoad Lockout Pushbutton to be depressed, then manual actions with the control switch. K/A JUSTIFICATION:This question satisfies the K/A statement by requiring the applicant to know the design feature ofthe pump breaker/start logic in which the Non-essential load lockout must be depressed to allow the auto start of the standby pump. The applicant must also know the design feature ofmanipulating the standby pump control switch (reset trip) which allows the standby pump to autostart. The "B" distractor is plausible since the first part is correct. The second is plausible if theapplicant thinks RBCCW operates like PSW. On a LOSP, PSW pumps will trip andsubsequently re-energize following a loading sequence once the EDGs tie to the bus. Theapplicant may also think they will automatically start since a low pressure start signal is present. The "C" distractor is plausible if the applicant remembers that the non-essential load lockout does not have to be depressed for all equipment powered from the 600V Emergency Bus and thinks that the RBCCW pump is one of them. The second part is plausible since it is correct. The "D" distractor is plausible if the applicant remembers that the non-essential load lockoutdoes not have to be depressed for all equipment powered from the 600V Emergency Bus andthinks that the RBCCW pump is one of them. The second is plausible if the applicant thinksRBCCW operates like PSW. On a LOSP, PSW pumps will trip and subsequently re-energize following a loading sequence once the EDGs tie to the bus. The applicant may also think theywill automatically start since a low pressure start signal is present.A. Correct - See description above. B. Incorrect - See description above. C. Incorrect - See description above. D. Incorrect - See description above. Wednesday, January 21, 2015 2:26:56 PM 207 ILT-09 NRC EXAM

References:

NONEK/A:400000 Component Cooling Water System (CCWS)K4. Knowledge of CCWS design feature(s) and or interlocks which provide for thefollowing: (CFR: 41.7) K4.01 Automatic start of standby pump . . . . . . . . . . . 3.4 3.9LESSON PLAN/OBJECTIVE:P42-RBCCW-LP-00901, Reactor Building Closed Cooling Water Ver. 3.0, EO 200.014.A.06 References used to develop this question:34AR-650-239-2, RBCCW Pumps Disch Press Low, Ver. 2.2 Bank question from HLT Database bank Q# 400000K4.01 005Wednesday, January 21, 2015 2:26:56 PM 208 ILT-09 NRC EXAM 6 4.600000AK1.02 001/03601FPS/200.092.A.01/BANK/P-AB/BOTH/600000AK1.02/1/1/F/2/JSC/ELJWhich ONE of the following plant fire locations REQUIRES a manual reactor scram IAW 34AB-X43-001-2, "Fire Procedure", when a major fire exists.Turbine Building Unit 2 East Cableway Unit Auxilary Transformer Oil Storage Tank Room Intake Structure A.B.C.D.Wednesday, January 21, 2015 2:26:56 PM 209 ILT-09 NRC EXAMDescription:OIL STORAGE TANK ROOM-De-energize local electrical equipment as necessary.-For a major fire, enter 34AB-C71-001-2, Scram Procedure, AND SCRAM the reactor.Of the listed fire locations, only Major Oil fires require the unit to be scrammed.K/A JUSTIFICATION:This question satisfies the K/A statement by requiring the applicant to have the knowledge of theoperational implications of having a major fire in the Plant (oil fire) and the required actions tobe taken IAW 34AB-X43-001-2, Fire Procedure.The "A" distractor is plausible since the Fire Procedure has actions concerning a fire in theTurbine Building Unit 2 East Cableway if a scram has occurred. The team will have to performsafe shutdown actions listed in its appropriate attachment. The "B" distractor is plausible since the unit has to be shutdown per 34GO-OPS-014-2, FastReactor Shutdown. The Fire Procedure has actions concerning a fire in the Unit AuxilaryTransformer if a scram has occurred. The team will have to perform safe shutdown actionslisted in its appropriate attachment. The "D" distractor is plausible since the unit has to be shutdown per 34GO-OPS-014-2, FastReactor Shutdown if PSW header pressure cannot be maintained > 60 psig or temperatures startto increase on PSW cooled components. The Fire Procedure has actions concerning a fire in the Intake Structure if a scram has occurred. The team will have to perform safe shutdown actionslisted in its appropriate attachment. A. Incorrect - See description above. B. Incorrect - See description above. C. Correct - See description above. D. Incorrect - See description above. Wednesday, January 21, 2015 2:26:56 PM 210 ILT-09 NRC EXAM

References:

NONEK/A:600000 Plant Fire On SiteAK1 Knowledge of the operation applications of the following concepts as they apply toPlant Fire On Site: AK1.02 Fire Fighting . . . . . . . . . . . . . . . . . . . . . . 2.9 3.1LESSON PLAN/OBJECTIVE:X43-FPS-LP-03601, Fire Protection System, Ver 5.0, EO 200.092.A.01References used to develop this question:34AB-X43-001-2, Fire Procedure, Ver 14.2Bank Question originally used on HLT 5 NRC exam Q#64Wednesday, January 21, 2015 2:26:56 PM 211 ILT-09 NRC EXAM 6 5.700000G2.2.44 001/02706S11/200.116.A.01/BANK/P-AB/BOTH/700000G2.2.44/1/1/H/3/ARB/ELJUnit 1 and Unit 2 are operating at 100% RTP. Following a grid disturbance, the following conditions/alarms exist: o4160 Bus 1E Voltage Low, (652-122)o4160 Bus 1F Voltage Low, (652-222) o4160 Bus 1G Voltage Low, (652-322)o4160 Bus 2E Voltage Low, (652-122)o4160 Bus 2F Voltage Low, (652-222)o4160 Bus 2G Voltage Low, (652-322)oAll 4160 VAC buses indicate 3820 VACoVoltage in the 230 KV switchyard is 231 KVoThe load dispatcher reports that these conditions will exist for 4 hour4.62963e-5 days <br />0.00111 hours <br />6.613757e-6 weeks <br />1.522e-6 months <br />sIAW 34AB-S11-001-0, Operation With Degraded Voltage, and the existing conditions, The 4160 VAC Emergency Bus voltages are ________ the MINIMUM ACCEPTABLEvoltage. One (1) hour later, __________ required to be supplying power to an Emergency Bus on each Unit. less than; ONLY one EDG is less than; NO EDGs are greater than; ONLY one EDG is greater than; NO EDGs are A.B.C.D.Description:34AB-S11-001-0, "Operation With Degraded Voltage," section 1.0 states that "Normalminimum voltage with either Unit in Modes 1, 2, or 3 is 233kV. This means that the switchyardvoltage is less than the normal minimum voltage level. Step 4.4 states "IF the 4160 VAC bus voltages CANNOT be maintained above 3825VAC (TechSpec), the following action will be taken:". This means that the 4160 VAC Emergency Busvoltages are less than the minimum acceptable voltages. Wednesday, January 21, 2015 2:26:56 PM 212 ILT-09 NRC EXAMStep 4.4.3 states "IF the 4160 VAC Bus voltages are NOT RESTORED to acceptable levelsWITHIN 30 minutes, perform the following to maintain 4160V 1E emergency bus voltage. (Two handed operations will be necessary):

4.4.3.1 Start the 1R43-S001A D/G, using the start switch, panel 1H11-P652. 4.4.3.2 Start the 2R43-S001A D/G, using the start switch, panel 2H11-P652K/A JUSTIFICATION:This question satisfies the K/A statement by requiring the applicant to interpret control roomindications for 4160 V Emergency bus alarms/voltages along with 230 KV switchyard voltagesand then determine if the minimum voltage still exists and the number of EDGs to be placed inservice IAW the abnormal procedure. The "B" distractor is plausible since the first part is correct. The second part is plausible if theapplicant remembers the TS requirement to restore 4160 VAC bus voltages to acceptable levelswithin 1 hour1.157407e-5 days <br />2.777778e-4 hours <br />1.653439e-6 weeks <br />3.805e-7 months <br /> (stated in 34AB-S11-001-0 step 4.4). The applicant then thinks he has 30 minutesmore to start a EDG on each Unit for a total of 1.5 hours5.787037e-5 days <br />0.00139 hours <br />8.267196e-6 weeks <br />1.9025e-6 months <br />. The procedure has both the 1 hour1.157407e-5 days <br />2.777778e-4 hours <br />1.653439e-6 weeks <br />3.805e-7 months <br /> and the 30 minute requirement being performed simultaneously.The "C" distractor is plausible if the applicant remembers the TS requirement for minimumvoltage on starting the EDGs is > 3740 V and < 4243 V. This requirement is checked usingseveral Surveillance Requirements (SRs) in TS. These include 3.8.1.2, 3.8.1.5, 3.8.1.7, 3.8.1.9, 3.8.1.10, 3.8.1.13, 3.8.1.17, and 3.8.1.18. The second part is plausible since it is correct. The "D" distractor is plausible if the applicant remembers the TS requirement for minimumvoltage on starting the EDGs is > 3740 V and < 4243 V. This requirement is checked usingseveral Surveillance Requirements (SRs) in TS. These include 3.8.1.2, 3.8.1.5, 3.8.1.7, 3.8.1.9,3.8.1.10, 3.8.1.13, 3.8.1.17, and 3.8.1.18. The second part is plausible if the applicantremembers the TS requirement to restore 4160 VAC bus voltages to acceptable levels within 1 hour1.157407e-5 days <br />2.777778e-4 hours <br />1.653439e-6 weeks <br />3.805e-7 months <br /> (stated in 34AB-S11-001-0 step 4.4). The applicant then thinks he has 30 minutes more tostart a EDG on each Unit for a total of 1.5 hours5.787037e-5 days <br />0.00139 hours <br />8.267196e-6 weeks <br />1.9025e-6 months <br />. The procedure has both the 1 hour1.157407e-5 days <br />2.777778e-4 hours <br />1.653439e-6 weeks <br />3.805e-7 months <br /> and the 30minute requirement being performed simultaneously.A. Correct - See description above. B. Incorrect - See description above. C. Incorrect - See description above. D. Incorrect - See description above. Wednesday, January 21, 2015 2:26:56 PM 213 ILT-09 NRC EXAM

References:

NONEK/A:700000 Generator Voltage and Electric Grid DisturbancesG2.2.44 Ability to interpret control room indications to verify the status and operation of asystem, and understand how operator actions and directives affect plant and system conditions. (CFR: 41.5 / 43.5 / 45.12) . . . . . . . . . . . . . . . . 4.2 4.4LESSON PLAN/OBJECTIVE:S11-LP-02706-02, Basic Grid Operating Concepts, Ver. 2.0, EO 200.116.A.01 References used to develop this question:34AB-S11-001-0, Operation With Degraded Voltage, Ver. 4.0 Original Question-question HLT bank Q# 700000G2.4.45 001Wednesday, January 21, 2015 2:26:56 PM 214 ILT-09 NRC EXAM 6 6.G2.1.23 001/00201N21/026.029.A.01/MOD/SYS-I/BOTH/G2.1.23/3/F/3/JSC/ELJUnit 2 is performing Low Pressure Feedwater Injection IAW 34SO-N21-007-2, Condensate andFeedwater System, during a Reactor startup. The following conditions exist:oOne Condensate Pump is runningoOne Condensate Booster Pump runningoCondensate/Feed system is lined up for injectionoTurbine Bypass valves are controlling RPV pressureo2N21-F165, Cleanup Recirc FCV, is throttled open to maintain RWLBased on the above conditions, As RPV pressure increases, 2N21-F165 will be throttled in the __________ direction.IAW 34SO-N21-007-2, to avoid excessive vibration on the piping returning to the MainCondenser, the MAXIMUM throttled position allowed for 2N21-F165 is __________ . close;55%close;70%open;55%open;70% A.B.C.D.Description:2N21-F165, Cleanup Recirc valve, is located downstream of the RFPs and is used for multiplepurposes. It is used for short/long cycle cleanup prior to injecting FW into the RPV during a startup. Water is cleaned up first in short cycle then long cycle. While performing long cyclecleanup, N21-F006A/B, FW Isolation s moving water through the demins. Water is directedback to the condenser through 2N21-F165. From long cycle, feedwater is injected to the RPV by opening the FW Isolation Valves. Discharge pressure is controlled by throttling 2N21-F165 and2N21-F111, Startup Level Control Valve. If 2N21-F165 is throttled close (less directed back tocondenser), FW pressure will increase until greater than RPV pressure. This d/p will cause flow to RPV and RWL will increase. Wednesday, January 21, 2015 2:26:56 PM 215 ILT-09 NRC EXAMPer Caution in 34SO-N21-007-2, Condensate and Feedwater Sytem-2N21-F165, Cleanup Recircvalve, must not be opened more than 70% with only a Condensate pump running or more than55% with a Condensate Booster Pump running to avoid excessive vibration on the piping returning to the Main Condenser. K/A JUSTIFICATION:This question satisfies the K/A statement by requiring the applicant to understand how to controlRWL during a reactor startup while at a low power condition. This step is performed per 34SO-N21-007-2 section 7.1.6, Low Pressure Feedwater Injection and is an integrated part of areactor startup.The "B" distractor is plausible since the first part is correct. The second part is plausible if theapplicant thinks 70% is the limit since this is the limit if only a Condensate pump is operating.The valve has a lower limit with a CBP running to prevent excessive vibration from occurring.The "C" distractor is plausible if the applicant thinks 2N21-F165, Cleanup Recirc Valve, issimiliar to the operation of 2N21-F111, Startup Level Control Valve. Both valves are used inconjunction with each other during startups while at low power. If 2N21-F111 is throttled in the close direction as RPV pressure increases, RWL will decrease since the valve is in line with theflowpath. The second part is plausible since it is correct.The "D" distractor is plausible if the applicant thinks 2N21-F165, Cleanup Recirc Valve, issimiliar to the operation of 2N21-F111, Startup Level Control Valve. Both valves are used inconjunction with each other during startups while at low power. If 2N21-F111 is throttled in the close direction as RPV pressure increases, RWL will decrease since the valve is in line with theflowpath. The second part is plausible if the applicant thinks 70% is the limit since this is theWednesday, January 21, 2015 2:26:56 PM 216 ILT-09 SRO NRC EXAMflowpath. The second part is plausible if the applicant thinks 70% is the limit since this is thelimit if only a Condensate pump is operating. The valve has a lower limit with a CBP running toprevent excessive vibration from occurring.

A. Correct - See description above. B. Incorrect - See description above. C. Incorrect - See description above. D. Incorrect - See description above. Wednesday, January 21, 2015 2:26:56 PM 217 ILT-09 NRC EXAM

References:

NONEK/A:2.1 Conduct of Operations G2.1.23 Ability to perform specific system and integrated plant procedures during allmodes of plant operation. (CFR: 41.10 / 43.5 / 45.2 / 45.6) . . . . . . . . . . . . 4.3 4.4LESSON PLAN/OBJECTIVE:N21-CNDFW-LP-00201, Condensate and Feedwater System, Ver 9.1, 026.029.A.01References used to develop this question:34SO-N21-007-2, Condensate and Feedwater System, Ver 52.1 Modified from HLT Database Q#LT-026029-006 Original QuestionUnit 2 Startup is in progress with one Condensate and one Condensate Booster pump running. The Condensate and Feedwater System is in a Long Cycle Cleanup mode with the Cleanup Recirc CV, 2N21-F165, 76% opened. Which ONE of the choices below completes the following statement? IAW 34SO-N21-007-2, "Condensate & Feedwater System", Section 7.3.3, "CondensateSystem Long Cycle Startup", the required procedure action for the above conditions is to immediately __________ for the cleanup recirc valve. A. increase the valve position by 24%. B. decrease the valve position by 6%.

C. decrease the valve position by 21%. D. decrease the valve position by 76%. Wednesday, January 21, 2015 2:26:56 PM 218 ILT-09 NRC EXAM 6 7.G2.1.40 001/04502F15/300.048.A.03/MOD/P-NORM/BOTH/G2.1.40/3/F/3/ARB/ELJUnit 2 is performing fuel movement in the RPV. IAW 34FH-OPS-001-0, Fuel Movement Operation, The Reactor Mode Switch __________ REQUIRED to be LOCKED in the REFUELposition. The MINIMUM RPV water level for fuel movement to continue is __________ above the top of the irradiated fuel assemblies seated in the RPV. is; 21 feet is; 23 feet is NOT; 21 feet is NOT; 23 feet A.B.C.D.Wednesday, January 21, 2015 2:26:56 PM 219 ILT-09 NRC EXAMDescription:IAW 34FH-OPS-001-0, Fuel Movement Operation, Attachment 1 step 1.2.6 states "ReactorMode Switch is locked in the REFUEL position and key removed". Step 1.2.7 states "Reactorwater level is at least 23 feet above the top of the irradiated fuel assemblies seated in the RPV". Also in Limitations, steps 5.2.2 & 5.2.3 state:5.2.2 Fuel movements in the reactor vessel may be performed only WHEN the ReactorMode switch is LOCKED in the REFUEL position.5.2.3 Reactor Vessel water level shall be maintained

>23' and Fuel Pool Water level shallbe maintained >21 feet, above the top of the fuel assemblies seated in the Vessel andFuel Pool. Fuel Pool level readings can be obtained from 1T24-R001 and2T24-R001, Fuel Pool level indicators, located in the Fuel Pools.K/A JUSTIFICATION:This question satisfies the K/A statement by requiring the applicant to know the Refueling administrative requirements for the position of the Reactor Mode switch and the minimum requirement for water level for fuel movement to continue. The "A" distractor is plausible since the first part is correct. The second is plausible if theapplicant remembers that 21 feet is the requirement for fuel pool level not the requirement forfuel assemblies seated in the RPV. The "C" distractor is plausible if the applicant remembers the mode switch must be in the Refuelposition and thinks the only time the mode switch is locked is when it is locked in the Shutdown position due to INOP Nuclear Instruments. The second is plausible if the applicant remembersthat 21 feet is the requirement for fuel pool level not the requirement for fuel assemblies seatedin the RPV. The "D" distractor is plausible if the applicant remembers the mode switch must be in the Refuelposition and thinks the only time the mode switch is locked is when it is locked in the Shutdown position due to INOP Nuclear Instruments. The second is plausible since it is correct. A. Incorrect - See description above. B. Correct - See description above. C. Incorrect - See description above. D. Incorrect - See description above. Wednesday, January 21, 2015 2:26:56 PM 220 ILT-09 NRC EXAM

References:

NONEK/A:2.1 Conduct of Operations G2.1.40 Knowledge of refueling administrative requirements. (CFR: 41.10 / 43.5 / 45.13) . . . . . . . . . . . . . . . . . 2.8 3.9LESSON PLAN/OBJECTIVE:F15-RF-LP-04502, Refueling, Ver. 4.0, EO 300.048.A.03 & EO 300.048.A.01References used to develop this question:34FH-OPS-001-0, Fuel Movement Operation, Ver. 25.4 Modified from HLT Database Q#G2.1.40-002 Original QuestionIAW 34FH-OPS-001-0, Fuel Movement Operation, which ONE of the choices below is therequired Reactor Mode Switch position and the MINIMUM Reactor Pressure Vessel (RPV)water level for fuel movement to occur in the RPV? Fuel movement in the RPV will be performed when the Reactor Mode Switch is in the

__________ position.The MINIMUM RPV water level for fuel movement to continue is __________ feetabove the top of the irradiated fuel assemblies seated in the RPV. A.

locked REFUEL; 23B.SHUTDOWN; 21C.SHUTDOWN; 23D.locked REFUEL;

21Wednesday, January 21, 2015 2:26:56 PM 221 ILT-09 NRC EXAM 6 8.G2.2.21 001/00701E11/100.035.A.03/BANK/P-NORM/BOTH/G2.2.21/3/H/3/JSC/ELJGiven the following:

1E11-F017A, RHR Injection valve, was declared INOPERABLE for preventative maintenance (PM).

Following the PM, operators performed the stroke test on 1E11-F017A IAW 34SV-E11-002-1,RHR Valve Operability. The stroke test data is shown below: IAW 34SV-E11-002-1, To time 1E11-F017A OPEN, the NPO will START the stopwatch when the __________ . Based on the above data, 1E11-F017A __________ be IMMEDIATELY declared OPERABLE. control switch is placed to OPEN; cancontrol switch is placed to OPEN; can NOTred light FIRST illuminates; can red light FIRST illuminates; can NOTA.B.C.D.Wednesday, January 21, 2015 2:26:56 PM 222 ILT-09 NRC EXAMDescription:

34SV-E11-002-1, RHR Valve Operability, step 4.3.5 states: Full-stroke time is that time intervalfrom initiation of the actuating signal to the end of the actuation cycle. Valves will be timedfrom WHEN the switch is positioned UNTIL either the green light EXTINGUISHES (open) ORthe red light EXTINGUISHES (close). Step 7.7.2.2 states: Valves with OPERATING times thatdo NOT meet the CALCULATED ALLOWABLE time will be immediately retested ORdeclared inoperable. They can not be immediately declared OPERABLE. K/A JUSTIFICATION:This question satisfies the K/A statement by requiring the applicant to know the requirements toreturn a component to operable status. The valve is inoperable prior to re-testing and the applicant has to make the determination to restore it to operable based on surveillanceperformance.The "A" distractor is plausible since the first part is correct. The second is plausible if theapplicant thinks that since the valve meet the Maximum Time Limit, it can be immediately declared operable. The valve must be immediately re-tested.The C" distractor is plausible if the applicant thinks timing occurs when the red light firstilluminates and does not know the procedure requirement. The second is plausible if theapplicant thinks that since the valve meet the Maximum Time Limit, it can be immediatelydeclared operable. The valve must be immediately re-tested.The "D" distractor is plausible if the applicant thinks timing occurs when the red light firstilluminates and does not know the procedure requirement. The second part is correct. A. Incorrect - See description above. B. Correct - See description above. C. Incorrect - See description above. D. Incorrect - See description above. Wednesday, January 21, 2015 2:26:56 PM 223 ILT-09 NRC EXAM

References:

NONEK/A:2.2 Equipment Control G2.2.21 Knowledge of pre- and post-maintenance operability requirements. (CFR: 41.10 / 43.2) . . . . . . . . . . . . . . . . . . . . . . . . . . 2.9 4.1LESSON PLAN/OBJECTIVE:E11-RHR-LP-00701, Residual Heat Removal System, Ver 9.1, EO 100.035.A.03LT-LP-30005, Technical Specifications, Ver 10.1, EO 300.006.A.23 References used to develop this question:34SV-E11-002-1, RHR Valve Operability, Ver 20.1Bank question used on 2011 HLT 6 NRC exam question #69Wednesday, January 21, 2015 2:26:56 PM 224 ILT-09 NRC EXAM 6 9.G2.2.36 001/03001T46/030.002.A.06/MOD/P-NORM/BOTH/G2.2.36/3/F/3/ARB/ELJUnit 1 and Unit 2 are operating at 100% RTP. On your Control Room tour during turnover, younote the change in status in the Standby Gas Treatment System (SBGT) as shown below: With the above status of SBGT, Troubleshooting on the feeder breaker to __________ will impact the LCO for the SBGTSystem if the troubleshooting activity resulted in tripping the feeder breaker to the MCC. 2R24-S012, Rx. Bldg. MCC 2B 2R24-S013, Rx. Bldg. MCC 2A 2R24-S014, Rx. Bldg. MCC 2D 2R24-S015, Rx. Bldg. MCC 2F A.B.C.D.Wednesday, January 21, 2015 2:26:57 PM 225 ILT-09 NRC EXAMDescription:The following table lists the power supplies for U1 & U 2 SBGT Systems:Component Power Supply SBGT Fan 2T46-C001A 600/208 VAC Essential MCC 2C (2R24-S011) SBGT Fan 2T46-C001B 600/208 VAC Essential MCC 2B (2R24-S012) SBGT Fan 1T46-C001A 600/208 VAC Essential MCC 1C (1R24-S011) SBGT Fan 1T46-C001B 600/208 VAC Essential MCC 1B (1R24-S012) With SBGT 2A "Danger" tagged out of service and inoperable, a 7 day Required ActionStatement (TS 3.6.4.3 Condiation B) exists. With Maintenance performing troubleshooting activities on the feeder breaker for 2R24-S012, Rx. Bldg. MCC 2B, which 2R24-S012 is thepower supply to SBGT 2B, if this breaker trips, both Unit 2 SBGT Systems will be inoperable. TS 3.6.4.3 Condition E, Required Action E.1 would then be entered requiring LCO 3.0.3 immediately. K/A JUSTIFICATION:This question satisfies the K/A statement by requiring the applicant to determine the potentialeffect (analyze) of troubleshooting (maintenance activity) a power source on the LCO for theSBGT System. The "B" distractor is plausible if the applicant remembers that this MCC is located in the Unit 2Reactor Building (same elevation as 2R24-S012) and believes this bus is the power supply to SBGT 2B and concludes that troubleshooting on this bus will effect the LCO for SBGT. The "C" distractor is plausible if the applicant remembers that this MCC is located in the Unit 2Reactor Building (same elevation as 2R24-S012) and believes this bus is the power supply toSBGT 2B and concludes that troubleshooting on this bus will effect the LCO for SBGT.The "D" distractor is plausible if the applicant remembers that this MCC is located in the Unit 2Reactor Building (same elevation as SBGT 2B) and believes this bus is the power supply toSBGT 2B and concludes that troubleshooting on this bus will effect the LCO for SBGT. A. Correct - See description above. B. Incorrect - See description above. C. Incorrect - See description above. D. Incorrect - See description above.

References:

NONEK/A:Wednesday, January 21, 2015 2:26:57 PM 226 ILT-09 NRC EXAM 2.2 Equipment ControlG2.2.36 Ability to analyze the effect of maintenance activities, such as degraded powersources, on the status of limiting conditions for operations. (CFR: 41.10 / 43.2 / 45.13) . . . . . . . . . 3.1 4.2 LESSON PLAN/OBJECTIVE:T46-SBGT-LP-03001, Standby Gas Treatment System, Ver. 6.0, EO 030.002.A.06 & EO 300.010.C.01 References used to develop this question:34SO-R23-001-2, 600V/480V AC System, Ver. 8.0 U1 TS 3.6.4.3, Standby Gas Treatment (SGT) System, Amendment 256 U2 TS 3.6.4.3, Standby Gas Treatment (SGT) System, Amendment 135 Modified for Plant Hatch from 2010 Oyster Creek NRC Exam Q#68 Original QuestionWednesday, January 21, 2015 2:26:57 PM 227 ILT-09 NRC EXAMWednesday, January 21, 2015 2:26:57 PM 228 ILT-09 NRC EXAM 7 0.G2.3.11 001/20312EOP101/013.054.A.07/BANK/P-EOP/BOTH/G2.3.11/3/F/3/JSC/ELJWhich ONE of the following is the BASIS for restarting the Turbine Building (TB) Ventilationwhen executing 31EO-EOP-014-2, "SC Secondary Containment Control - RR RadioactivityRelease Control"? Restarting the TB Ventilation __________ AND assures a release from the TB VentilationSystem is monitored prior to exiting the __________ . preserves personnel accessibility; Main Stack preserves personnel accessibility; Reactor Building Stack maintains equipment operability; Main Stack maintains equipment operability; Reactor Building Stack A.B.C.D.Wednesday, January 21, 2015 2:26:58 PM 229 ILT-09 NRC EXAMDescription:Continued personnel access to the turbine building may be essential for responding toemergencies or transients which may degrade into emergencies. The turbine building is not anair-tight structure. A radioactivity release inside the turbine building would limit personnelaccess and eventually lead to an unmonitored ground level release. Operation of the turbine building ventilation system: helps preserves turbine building accessibility, AND assures thatradioactivity in turbine building areas is discharged through a monitored release point. (Discharged to the reactor building stack). K/A JUSTIFICATION:This question satisfies the K/A statement by requiring the applicant to know where the TurbineBuilding exhaust release point is occurring and the reason why restarting the ventilation isrequired.The "A" distractor is plausible since the first part is correct. The second part is plausible if theapplicant remembers that SBGT system discharges to the Main Stack and since the TurbineBuilding Ventilation will be processing the TB atmosphere, that it will discharge to the MainStack as well. The "C" distractor is plausible if the applicant thinks that since there is equipment that may beoperated in the Turbine Building then equipment operability is the reason. The second part is plausible if the applicant remembers that SBGT system discharges to the Main Stack and sincethe Turbine Building Ventilation will be processing the TB atmosphere, that it will discharge tothe Main Stack as well. The "D" distractor is plausible if the applicant thinks that since there is equipment that may beoperated in the Turbine Building then equipment operability is the reason. The second part is plausible since it is correct. A. Incorrect - See description above. B. Correct - See description above. C. Incorrect - See description above. D. Incorrect - See description above. Wednesday, January 21, 2015 2:26:58 PM 230 ILT-09 NRC EXAM

References:

NONEK/A:2.3 Radiation Control G2.3.11 Ability to control radiation releases. (CFR: 41.11 / 43.4 / 45.10) . . . . . . . . . . . . . 3.8 4.3LESSON PLAN/OBJECTIVE:EOP-SCRR-LP-20325, Secondary Containment / Radioactivity Release Control, Ver 2.1 ,EO 201.082.A.01 Reference(s) used to develop this question:31EO-EOP-014-2, Secondary Containment Control - Radioactivity Release Control, Ver 11Bank question used on 2011 NRC Exam Q#70Wednesday, January 21, 2015 2:26:58 PM 231 ILT-09 NRC EXAM 7 1.G2.3.13 001/30008ADMRAD/M30008.003/BANK/P-NORM/BOTH/G2.3.13/3/F/3/ARB/ELJTwo NPOs are required to enter a "Locked High Radiation" area room to perform a tagout. The NPOs will be accompanied by a Radiation Protection (RP) Technician (Tech). IAW 62RP-RAD-016-0, Control Of High Radiation Areas, which ONE of the choices belowcompletes the following statements? The keys to the "Locked High Radiation" area room can be issued from __________ . After exiting the Locked High Radiation Area, the door can be verified secure by __________ . the RP Office ONLY one of the NPOs the RP Office ONLY the RP Tech ONLY either the the RP Office OR the Work Control Center; one of the NPOs either the the RP Office OR the Work Control Center; the RP Tech ONLY A.B.C.D.Wednesday, January 21, 2015 2:26:58 PM 232 ILT-09 NRC EXAMDescription:

62RP-RAD-016-0, Step 5.2.3.3 & 5.2.3.4 states that for Very High Radiation Area and LockedHigh Radiation Area keys, the keys will ONLY be issued to RP technicians. Upon exitingLocked High Radiation Area doors, concurrent verification is required assuring that thedoor/padlock is secured AND locked. This will be performed by the RP technician holding thekey and a second worker that will act as the verifier. For entry in to Very High Radiation Areas,RP personnel only will verify that the door used during the entry is secured, AND locked. K/A JUSTIFICATION:This question satisfies the K/A statement by requiring the applicant to understand theradiological procedures for access into locked high radiation areas and the requirements for keycontrol and door verification. The "B" distractor is plausible since the first part is correct. The second is plausible if theapplicant thinks about the requirements for a Very High Radiation Area door and thinks these are the requirements for a Locked High Radiation area. Very High requires RP personnel to verifythe door. The "C" distractor is plausible if the applicant thinks that since the Work Control Centerprovides the work package for performing work that you could check out the high radiation doorkey to perform work in High radiation areas as well. The second part is plausible since it is correct. The "D" distractor is plausible if the applicant thinks that since the Work Control Centerprovides the work package for performing work that you could check out the high radiation doorkey to perform work in High radiation areas as well. The second is plausible if the applicantthinks about the requirements for a Very High Radiation Area door and thinks these are the requirements for a Locked High Radiation area. Very High requires RP personnel to verify thedoor. A. Correct - See description above. B. Incorrect - See description above. C. Incorrect - See description above. D. Incorrect - See description above. Wednesday, January 21, 2015 2:26:58 PM 233 ILT-09 NRC EXAM

References:

NONEK/A:2.3 Radiation Control G2.3.13 Knowledge of radiological safety procedures pertaining to licensed operatorduties, such as response to radiation monitor alarms, containment entry requirements, fuel handling responsibilities, access to locked high-radiation areas, aligning filters, etc. (CFR: 41.12 / 43.4 / 45.9 / 45.10) . . . . . . . . . . . . . . . . . . 3.4 3.8LESSON PLAN/OBJECTIVE:LT-LP-30008, Radiation Control Adminstration Procedures And Instrumentation, LOLT-30008.003 Reference(s) used to develop this question:62RP-RAD-016-0, Control Of High Radiation Areas, Ver. 33.0 Bank question from HLT Database Q#G2.3.13-001 used on Hatch 2011-301 NRC Exam Q#72 Wednesday, January 21, 2015 2:26:58 PM 234 ILT-09 NRC EXAM 7 2.G2.3.14 001/30004ADMIN/LT-30008.002/BANK/P-NORM/BOTH/G2.3.14/3/F/2/JSC/ELJUnit 1 is operating at 100% RTP.

3 4SV-E51-002-1, RCIC Pump Operability, will be performed within the next hour.IAW 34SV-E51-002-1, the __________ (RCIC) diagonal will be posted as a __________ during the RCIC run.Southwest;Radiation Area ONLYSouthwest;High Radiation AreaNorthwest;Radiation Area ONLYNorthwest;High Radiation AreaA.B.C.D.Wednesday, January 21, 2015 2:26:58 PM 235 ILT-09 NRC EXAMDescription:IAW 34SV-E51-002-1, step 5.1.8 states "The number of personnel in the RCIC Room AndTorus Area will be limited during testing due to High Radiation Areas AND the potential for a high energy line break in these rooms." Step 6.1 states "At least one hour PRIOR to running thisequipment, notify Health Physics to post the necessary locations as High Radiation Areas." A High Radiation Area is an area with radiation dose rates >100 mr/hr. Unit 1 RCIC is located in the Southwest diagonal while Unit 2 RCIC is located in the Northwestdiagonal. K/A JUSTIFICATION:This question satisfies the K/A statement by requiring the applicant to know the effects thatrunning RCIC will have on the radiation levels in the RCIC diagonal and the required posting requirements.The "A" distractor is plausible since the first part is correct. The second part is plausible if theapplicant remembers that this area is already a Radiation Area without RCIC in operation. The "C" distractor is plausible if the applicant thinks about the Unit 2 location vice the Unit 1location. This would be correct if asking for Unit 2. The second part is plausible if the applicantremembers that this area is already a Radiation Area without RCIC in operation.The "D" distractor is plausible if the applicant thinks about the Unit 2 location vice the Unit 1location. This would be correct if asking for Unit 2. The second part is plausible since it is correct.A. Incorrect - See description above. B. Correct - See description above. C. Incorrect - See description above. D. Incorrect - See description above. Wednesday, January 21, 2015 2:26:58 PM 236 ILT-09 NRC EXAM

References:

NONEK/A:2.3 Radiation Control G2.3.14 Knowledge of radiation or contamination hazards that may arise during normal,abnormal, or emergency conditions or activities.

(CFR: 41.12 / 43.4 / 45.10) . . . . . . . . . . . . . . . 3.4 3.8LESSON PLAN/OBJECTIVE:LT-LP-30008, Radiation Control Administrative Procedures and Instrumentation, EO LT-30008.002 References used to develop this question:34SV-E51-002-1, RCIC Pump Operability, Ver 26.1Original Question-question HLT bank #G2.3.14 003Wednesday, January 21, 2015 2:26:58 PM 237 ILT-09 NRC EXAM 7 3.G2.4.21 001/00701RHR/006.005.A.02/MOD/SYS-I/BOTH/G2.4.21/3/H/3/ARB/ELJUnit 2 is in a forced outage with RHR Loop "B" in Shutdown Cooling Mode. oRPV pressure is 115 psigSubsequently, a leak occurs in the Drywell resulting in Drywell pressure increasing to 1.9 psig. Based on the above conditions and with NO operator actions, RHR Loop "B" __________ REMAIN in the Shutdown Cooling Mode. HPCI __________ AUTOMATICALLY start and inject into the RPV. will;will will;will NOT will NOT;will will NOT;will NOT A.B.C.D.Wednesday, January 21, 2015 2:26:58 PM 238 ILT-09 NRC EXAMDescription:Phil, this was question 5 of 10 of the previously submitted questions. Changes wereincorporated based on your ES-401-9 comments. 2E11-F015A/B will automatically close if a PCIS Group 2 signal is received (3 inches RWL or1.85 psig Drywell pressure) while in the SDC mode.

HPCI will receive an isolation signal due to low steam supply pressure at 134 psig.K/A JUSTIFICATION:This question satisfies the K/A statement by requiring the applicant to understand the status ofthe RHR Systems during SDC (core cooling & heat removal) operation along with TorusCooling (containment condition) operation and then addessing the safety status, i.e. still in service providing function or isolated and not providing function. The "A" distractor is plausible if the applicant thinks about the SDC valves F008/F009 will onlyclose on low RWL at 3 inches or high RPV pressure at 138 psig and thinks SDC will remain inservice based on the fact that neither one of these conditions are met. F015A/B acts as a Group 2 valve only when F008 and F009 are open. Therefore the normal operation of the F015A/B wouldreceive an open signal on the receipt of a LOCA signal. The second part is plausible if theapplicant thinks about the setpoint for RCIC isolating on Low RPV pressure at 95 psig and thinks this is the HPCI setpoint.The "B" distractor is plausible if the applicant thinks about the SDC valves F008/F009 will onlyclose on low RWL at 3 inches or high RPV pressure at 138 psig and thinks SDC will remain inservice based on the fact that neither one of these conditions are met. F015A/B acts as a Group 2valve only when F008 and F009 are open. Therefore the normal operation of the F015A/B would receive an open signal on the receipt of a LOCA signal. The second part is plausible since it iscorrect.The "C" distractor is plausible since the first part is correct. The second part is plausible if theapplicant thinks about the setpoint for RCIC isolating on Low RPV pressure at 95 psig andthinks this is the HPCI setpoint.A. Incorrect - See description above. B. Incorrect - See description above. C. Incorrect - See description above. D. Correct - See description above.

References:

NONEK/A:Wednesday, January 21, 2015 2:26:58 PM 239 ILT-09 NRC EXAM 2.4 Emergency Procedures / PlanG2.4.21 Knowledge of the parameters and logic used to assess the status of safety functions,such as reactivity control, core cooling and heat removal, reactor coolant system integrity,containment conditions, radioactivity release control, etc. (CFR: 41.7 / 43.5 / 45.12) . . . . . . . . . . . . . . . . . 4.0 4.6LESSON PLAN/OBJECTIVE:E11-RHR-LP-00701, Residual Heat Removal System, Ver. 9.1, EO 006.005.A.02 Reference(s) used to develop this question:34SO-E11-010-2, Residual Heat Removal System, Ver. 40.4 34SO-E41-001-2, High Pressure Coolant Injection (HPCI) System, Ver 28.334SO-E51-001-2, Reactor Core Isolation Cooling (RCIC) System, Ver 25.1Modified from HLT Database Q#205000G2.4.21-001Original QuestionUnit 2 is in a forced outage with the following conditions: At 11:00oRHR Loop "A" in Torus Cooling modeoRHR Loop "B" in Shutdown Cooling (SDC) modeoReactor pressure20 psig LOWERING at 2 psig/minute due to RHR SDC oTorus temperature98°F LOWERING at 0.3°F/minute At 11:05 RPS MG set 2A tripsWhich ONE of the following completes both statements? (Assume NO Operator action) RHR __________ will REMAIN in its' current lineup. At 11:20 a Tech Spec Mode change __________ have occurred. A. Loop A; will NOT B. Loop A; will C. Loop B; will NOTWednesday, January 21, 2015 2:26:58 PM 240 ILT-09 NRC EXAMD. Loop B; will Wednesday, January 21, 2015 2:26:58 PM 241 ILT-09 NRC EXAM 7 4.G2.4.32 001/21401EP/H-EP-001.042.A.05/MOD/P-EP/BOTH/G2.4.32/3/H/2/JSC/ELJUnit 2 was operating at 100% RTP when a loss of electrical power results in a loss of asignificant number of Main Control Room annunciators (P603, P602 & P601) with a Reactorscram. oAn Alert Emergency classification is declaredIAW NMP-EP-111-002, Emergency Notification Network Communicator Instructions - Hatch,The LATEST time that the INITIAL Plant Page Announcement can be made is __________ .Without additional direction from the Emergency Director, the operator __________REQUIRED to repeat the Plant Page Announcement for the next two (2) hours. 14 minutes;is 14 minutes;is NOT 29 minutes;is 29 minutes;is NOT A.B.C.D.Description:The loss of annunciators concurrent with a significant transient (scram) is an Alert emergencybased on SA4.Once an Alert emergency is declared, the following events have to occur:

IAW NMP-EP-111-002, section III, INSTRUCTIONS & STANDARD ANNOUNCEMENTFOR ALERT EMERGENCY:states "The appropriate emergency tone and announcement mustWednesday, January 21, 2015 2:26:58 PM 242 ILT-09 NRC EXAMbe made as soon as possible, but not to exceed 15 minutes after the initial emergencydeclaration"IAW NMP-EP-111-F12, 6.0 b. states "After the first two (2) hours, repeat the announcement as directed by the ED, SM, or SS and track time of announcement. K/A JUSTIFICATION:This question satisfies the K/A statement by requiring the applicant to know the announcementrequirements for the emergency associated with a loss of annunciators.The "B" distractor is plausible since the first part is correct. The second part is plausible if theapplicant thinks that they need direction from the Emergency Director as required if theemergency lasts longer than 2 hours2.314815e-5 days <br />5.555556e-4 hours <br />3.306878e-6 weeks <br />7.61e-7 months <br />. NMP-EP-111-F12 states "After the first two (2) hours,repeat the announcement as directed by the ED, SM, or SS and track time of announcement."The operator does not need further direction to repeat the announcement for the first two hoursIAW NMP-EP-111-F12.The "C" distractor is plausible if the applicant thinks about the followup announcement timerequirement of repeating the announcement every 30 minutes instead of the required 15 minutelimit. The second part is plausible since it is correct. The "D" distractor is plausible if the applicant thinks about the followup announcement timerequirement of repeating the announcement every 30 minutes instead of the required 15 minute limit. The second part is plausible if the applicant thinks that they need direction from theEmergency Director as required if the emergency lasts longer than 2 hours2.314815e-5 days <br />5.555556e-4 hours <br />3.306878e-6 weeks <br />7.61e-7 months <br />. NMP-EP-111-F12states "After the first two (2) hours, repeat the announcement as directed by the ED, SM, or SSand track time of announcement." The operator does not need further direction to repeat theannouncement for the first two hours IAW NMP-EP-111-F12.A. Correct - See description above. B. Incorrect - See description above. C. Incorrect - See description above. D. Incorrect - See description above.

References:

NONEK/A:2.4 Emergency Procedures / PlanG2.4.32 Knowledge of operator response to loss of all annunciators. Wednesday, January 21, 2015 2:26:58 PM 243 ILT-09 NRC EXAMLESSON PLAN/OBJECTIVE:EP-SS-LP-21401, Operations Fundamentals, Ver 4.0, EO H-EP-001.042.A.05References used to develop this question:NMP-EP-111-F12, Emergency Page Announcement Instructions, Ver 1.0NMP-EP-111-002, Emergency Notification Network Communicator Instructions-Hatch, Ver 1.0Original Question-HLT 7 NRC exam q#75Unit 2 was operating at 100% RTP when a loss of electrical power results in a loss of asignificant number of Main Control Room annuciators and a reactor scram. An ALERT Emergency classification is declared.

Which ONE of the choices below completes the following statement? The LATEST time from the ALERT declaration that the INITIAL Plant Page Announcementcan be made and still meet the requirements of NMP-EP-111-002, Emergency NotificationNetwork Communicator Instructions - Hatch, is _________ . Without additional direction from the Emergency Director, the operator is REQUIRED tomake __________ . A. 15 minutes; the INITIAL Plant Page Announcement, followed by a repeat Plant Page Announcement approximately 30 minutes later B. 15 minutes; ONLY the Initial Plant Page AnnouncementC. 30 minutes; the INITIAL Plant Page Announcement, followed by a repeat Plant Page Announcement approximately 30 minutes later D. 30 minutes; ONLY the Initial Plant Page Announcement Wednesday, January 21, 2015 2:26:58 PM 244 ILT-09 NRC EXAM 7 5.G2.4.49 001/01501MSRFW/200.050.B.02/MOD/P-AB/BOTH/G2.4.49/3/F/3/ARB/ELJUnit 2 is at 100% RTP when a loss of Feedwater Heating event occurs. IAW 34AB-N21-001-2, Loss Of Feedwater Heating,The Immediate Operator Action is to depress the __________ and reactor power willINITIALLY be reduced and maintained __________ below the steady state power level priorto the feedwater temperature reduction. Individual Recirc Flow Control LOWER FAST pushbuttons; 3% to 5% Individual Recirc Flow Control LOWER FAST pushbuttons; >20% Master Recirc Flow Control LOWER FAST pushbutton; 3% to 5% Master Recirc Flow Control LOWER FAST pushbutton; >20% A.B.C.D.Wednesday, January 21, 2015 2:26:58 PM 245 ILT-09 NRC EXAMDescription:IAW 34AB-N21-001-2 step 3.1 states "Maintain Reactor power BELOW the steady statepower level prior to the feedwater temperature reduction, via recirc, using the MasterRecirc Flow Control LOWER FAST pushbutton, per 34SO-B31-001-2 AND 34GO-OPS-005-2." K/A JUSTIFICATION:This question satisfies the K/A statement by requiring the applicant to know (without reference)the Immediate operator action for a loss of Feedwater Heating which is to lower reactor power using the appropriate controls (Master Recirc Flow Control LOWER FAST) to a specified value(the steady state power level prior to the feedwater temperature reduction). The "A" distractor is plausible since the Individual Recirc Flow Control LOWER FASTpushbuttons will reduce recirc speeds but are not required to be used IAW 34SO-N21-001-2.

The second part is plausible since it is correct. The "B" distractor is plausible since the Individual Recirc Flow Control LOWER FASTpushbuttons will reduce recirc speeds but are not required to be used IAW 34SO-N21-001-2. The second part is plausible if the applicant remembers the subsequent actions of reducing morethan 20% pre-event power level which 65% RTP would meet. The "D" distractor is plausible since the first part is correct. The second part is plausible if theapplicant remembers the subsequent actions of reducing more than 20% pre-event power level which 65% RTP would meet. A. Incorrect - See description above. B. Incorrect - See description above. C. Correct - See description above. D. Incorrect - See description above.

References:

NONEK/A:2.4 Emergency Procedures / Plan2.4.49 Ability to perform without reference to procedures those actions that requireimmediate operation of system components and controls.

(CFR: 41.10 / 43.2 / 45.6) . . . . . . . . . . 4.6 4.4Wednesday, January 21, 2015 2:26:58 PM 246 ILT-09 NRC EXAMLESSON PLAN/OBJECTIVE:N22-MSRFW-LP-01501, Moisture Separator Reheaters And Feedwater Heaters, Ver. 5.2 EO 200.050.B.02 Reference(s) used to develop this question:34AB-N21-001-2, Loss Of Feedwater Heating, Ver. 7.9 34GO-OPS-005-2, Power Changes, Ver. 28.3 Modified from HLT Database Q#295014G2.1.25-001 & Q#NRC2009302-075Q#NRC2009302-075 was used on 2009 Hatch NRC Exam Q#75 Original Question Q#295014G2.1.25-001 Unit 2 is at 100% power when a loss of feedwater heating event occurs. Which ONE of the following completes the following statement IAW 34AB-N21-001-2, "LossOf Feedwater Heating" Section 3.0 Immediate Operator Actions?Maintain Reactor power BELOW _______ using ______ .A. 65%; recirc B. 65%; control rods C. the steady state power level prior to the feedwater temperature reduction; recirc D. the steady state power level prior to the feedwater temperature reduction; control rods Original Question Q#NRC2009302-075 Unit 2 is at 100% RTP when a loss of feedwater heating event occurs. oReactor power is reduced IAW 34AB-N21-001-2, "Loss Of Feedwater Heating".oReactor power is STABLE at 75% RTP.Which ONE of the choices below completes the following statements? IAW 34AB-N21-001-2, the OATC was INITIALLY directed to depress the __________ toWednesday, January 21, 2015 2:26:58 PM 247 ILT-09 NRC EXAMlower reactor power. IAW 34GO-OPS-005-2, Power Changes, the HIGHEST listed Final Feedwater Temperatureat 75% RTP which would REQUIRE the Final Feedwater Temperature ReductionCumulative Usage to be tracked is __________ . Reference Provided A. Master Recirc Flow Control LOWER FAST pushbutton; 390°F B. Master Recirc Flow Control LOWER FAST pushbutton; 380°F C. Individual Recirc Flow Control LOWER FAST pushbuttons; 390°F D. Individual Recirc Flow Control LOWER FAST pushbuttons; 380°F Wednesday, January 21, 2015 2:26:58 PM 248 ILT-09 NRC EXAMThe auto closure of the recirc pump discharge valve is initiated when RPV pressure decreases to370 psig and a LOCA signal (from either Div I or Div II isolation logic) is presentsimultaneously. The valve cannot be opened if a LOCA signal is present, regardless of reactorpressure. An Alert emergency is declared based on the Fission Product Barrier Evaluation Chart-Modes1-2-3. Based on the 8.5 psig in the drywell, the plant has a loss of the RCS barrier.The conditions to declare a Notification of an Unusual event have also been met based onSU5-RCS Leakage. Wednesday, January 21, 2015 2:27:02 PM347 ILT-09 NRC EXAM JUSTIFICATION:

The must have detailed procedure knowledge of NMP-EP-110-GL02, HNP EALs - ICs,Threshold Values and Basis, and recoginition of the specific plant conditions to classify theappropriate emergency. Declaration of an emergency is above the RO knowledge level.K/A JUSTIFICATION:This question satisfies the K/A statement by requiring the applicant to be able to diagnoseindications of a failure of both Recirc pump seals in one loop and be able to respond to thesituation by isolating the leak based on annunciator response procedures and then be able toclassify the emergency.The "A" distractor is plausible if the applicant thinks about the annunciator REACTOR PRESSLOW 500 PSIG (601-314) which has a setpt of 425 psig. At this pressure, LPCI and CS valvesopen to lineup for injection. Injection will occur as soon as pressure lowers below the shutoffhead of each pump. The answer is plausible also if RPV pressure were below 370 psig. The second part is plausible since it is correct. The "B" distractor is plausible if the applicant thinks about the annunciator REACTOR PRESSLOW 500 PSIG (601-314) which has a setpt of 425 psig. At this pressure, LPCI and CS valvesopen to lineup for injection. Injection will occur as soon as pressure lowers below the shutoffhead of each pump. The answer is plausible also if RPV pressure were below 370 psig. The second part is plausible since a Notification of an Unusual Event (SU5) conditions have beenmet however it is not the highest emergency level classification.The "D" distractor is plausible since the first part is correct. The second part is plausible since aNotification of an Unusual Event (SU5) conditions have been met however it is not the highestemergency level classification.A. Incorrect - See description above. B. Incorrect - See description above. C. Correct - See description above. D. Incorrect - See description above. Wednesday, January 21, 2015 2:27:03 PM348 ILT-09 NRC EXAM

References:

NONE K/A:2.4 Emergency Procedures / Plan G2.4.31 Knowledge of annunciator alarms, indications, or response procedures. (CFR: 41.10 / 45.3) . . . . . . . . . . . 4.2 4.1SRO only because of link to 10CFR55.43(b)(5): Assessment of facility conditions and selection of appropriate procedure, recalling the action in the body of procedure and whento take the action. LESSON PLAN/OBJECTIVE:B31-RRS-LP-00401, Reactor Recirculation System, Ver 10.5, EO 004.001.A.07References used to develop this question:34AR-601-314-2, REACTOR PRESS LOW 500 PSIG, Ver 2.4You have completed the test!Wednesday, January 21, 2015 2:27:03 PM34 9